family

¡Supera tus tareas y exámenes ahora con Quizwiz!

What psych medication is known to cause aplastic anemia and agranulocytosis in a small number of patients. In addition, these patients need to avoid alcohol, grapefruit juice, and they should not drive

Carbamazepine

Treatment or generalized convulsives, simple partial, and complex partial seizures are treated with?

Carbamazepine, phenytoin, and valproic acid

Supraventricular Tachycardia Tx

Carotid Massage/Adenosine --> Beta Blocker or Verapamil --> Flecainide

What medication inhibits the metabolism of levodopa to 3-0-methyldopa?

Catecholamine-0-methyl-transferase

A newborn baby with some pyrexia developed a sudden onset of seizures and gradual depressed level of consciousness, on the tenth day of birth, and was brought to the neonatal intensive care unit of the hospital. The delivery had been uneventful after a prolonged rupture of membranes. The child seemed all right at birth. The mother reported normal pregnancy except that she had some blisters around the genital area with some pain during micturition towards delivery time. An EEG in the baby showed fronto-temporal focal abnormalities. What is the most common cause of this condition??

Caused by a type II virus

Treatment of bacterial meningitis in children 3- adults 55 years of age?

Cefotaxmine and vancomycin

Treatment of neisseria gonorrhoeae (yellow/milky/serous discharge)

Ceftriaxone with concurrent treatment of chlamydia

Diarrhea plus vesicular rash

Celiac

...

(1) (2) (3) (4) (5)

A 38-year-old woman is going through a divorce and simultaneously filing bankruptcy. She is very stressed about her financial situation and failed marriage. One day, after a particularly long crying spell, she notices a sudden onset of extreme difficulty swallowing at dinnertime. She has difficulty swallowing both solids and liquids. She feels that the food is sticking in her throat. She ignores it. She has numerous bouts of these episodes of difficulty with swallowing. She notices that when she lies down, undigested food comes up. Finally, she sees her doctor. On questioning by her doctor, she insists she has no heartburn. She has not seen any blood when she regurgitates nor has she vomited blood. Manometry is performed which shows an absence of normal peristalsis and an elevated LES pressure. What is the most likely diagnosis? 1 Achalasia 2 Scleroderma 3 Esophageal varices 4 Gastroesophageal reflux disease 5 Mallory-Weiss syndrome

(1) Achalasia is characterized by dysphagia. There is poor peristalsis in achalasia. This patient has a characteristic history for achalasia. The onset is during a period of stress. The symptoms consist of dysphagia, complaints of food sticking in the throat, and regurgitation. Scleroderma can present with dysphagia. However, manometry would show a decreased LES pressure, not an elevated LES pressure as the case here. In addition, scleroderma can be associated with heartburn. Esophageal varices usually present with hematemesis. Gastroesophageal reflux disease would present with heartburn. If anything, the LES pressure would be lower than normal, rather than elevated. Mallory-Weiss syndrome is an esophageal tear. This would present with hematemesis.

6) A 26-year-old Caucasian man went on a tour to Asia. He traveled to India, Sri Lanka, and Hong Kong over a period of 10 days and then returned to the USA. About 1 week later, he started feeling tired and complained of malaise. Two days later he develops a low grade fever associated with nausea, vomiting, and right upper quadrant pain. His roommate thinks his eyes look yellow and insists on taking him to the ER. The patient has no significant past medical history and is on no medications. He is a non-smoker and drinks alcohol only occasionally. On examination, he has a temperature of 100.4 F; pulse is 84/minute, and BP 120/80 mm Hg. His sclera is icteric, and he has no lymphadenopathy. Lungs are clear, and heart sounds normal. Abdominal exam reveals an enlarged liver 3 cm below the subcostal margin, with mild tenderness in the right upper quadrant. There is no evidence of ascites, caput medusae, or splenomegaly. Neurological exam shows a fully oriented patient without any deficits. Labs reveal a Hb of 14g/dL, WBC of 13,000/uL, platelets 350,000/uL, ALT 3200 U/L, AST 2650 U/L, alkaline phosphatase 675 U/L, total bilirubin 10mg/dL, and direct bilirubin 7mg/dL. Blood cultures are drawn and hepatitis profile sent. Ultrasound of the liver is pending. Which of the following would you consider the most likely diagnosis? 1 Hepatitis A 2 Hepatitis B 3 Hepatitis C 4 Hepatitis D 5 Alcoholic hepatitis

(1) Hepatitis A is a common cause of acute hepatitis all over the world caused by Hepatitis A virus (HAV), an RNA virus. It is more prevalent in the developing countries like India, due to overcrowded areas with poor hygiene and sanitation and in the lower socio-economic strata. It is spread by the fecal-oral route and has an incubation period of 2-6 weeks. The prodrome is characterized by malaise, fatigue, fever, nausea and vomiting, and right upper quadrant pain, as in this patient. Labs show liver enzymes to be higher than 1000 mg/dL and direct hyperbilirubinemia. ALT is higher than AST. Leukocytosis may be seen. Detection of serum IgM anti-HAV is diagnostic and remains positive for 3-6 months. IgG anti-HAV appears after 1 month of disease and may persist for years. Presence of IgG alone indicates previous exposure to HAV, immunity to recurring HAV infection, and noninfectivity. The disease is self limiting, and treatment is supportive. Almost all patients recover fully, and fulminant hepatitis is rare.

26) What is used to prepare the Hepatitis B vaccine in the United States? Answer Choices 1 Hepatitis B surface antigen (HBsAg) 2 Hepatitis B core antigen (HBcAg) 3 Hepatitis B toxoid 4 Hepatitis B e antigen (HBeAg) 5 Inactivated hepatitis B virus

(1) Hepatitis B surface antigen (HBsAg) is the antigen which is used for hepatitis B(HB) vaccination. In the United States, Hepatitis B vaccines use recombinant DNA technology to express HBsAg in yeast. It is then purified by biochemical and biophysical separation techniques. Hepatitis B vaccines licensed in the United States are formulated to contain 10-40 μg of HBsAg protein/ml. The vaccine is administered intramuscularly in 3 doses, the schedule being 0, 1, and 6 months. Infants of mothers with HBsAg status being negative or unknown, should receive the first dose of HB monovalent vaccine at birth and the second dose between 1-2 months and the third dose between 6-18 months. Infants born to HBsAg positive mothers should receive HB vaccine and HB immunoglobulin within 12 hours of birth. The second dose is administered at 1-2 months of age. The final dose in the immunization series should not be administered before 24 weeks of age.

19) A 62-year-old woman has an unexplained weight loss. She has a vague, non-descriptive pain in her stomach accompanied by diarrhea. When her son sees that she has become yellow, he insists that she seek medical attention. Numerous tests are done, including endoscopic retrograde pancreatography, and she is told that she has pancreatic cancer. The tests reveal that the pancreatic cancer is located at the head of the pancreas. What tumor marker is most likely to be found? 1 CA 19-9 (carbohydrate antigen 19-9) 2 AFP (alpha-fetoprotein) 3 CALLA (common acute lymphoblastic leukemia associated antigen) 4 PSA (prostate specific antigen) 5 Gastrin

(1) Pancreatic carcinoma can present with abdominal pain, jaundice, and weight loss. Jaundice would be seen with pancreatic carcinoma located at the head of the pancreas because it obstructs the common bile duct. CA 19-9 is carbohydrate antigen (CA) 19-9. This marker is a sensitive marker in patients with pancreatic carcinoma. This marker can be elevated in other GI cancers as well as some other medical conditions.

7) A 56-year-old man presents with a 24-hour history of abdominal pain, nausea and vomiting, fever, and clammy skin. Home treatment included acetaminophen and fluids; treatment offered little relief. His past medical history includes gallstones and congestive heart disease. His medications include thiazide diuretics. On clinical exam, his skin is cool and clammy, he has abdominal tenderness in the upper quadrants, his heart rate is 100, and his blood pressure is 110/70. What is his diagnosis? Answer Choices 1 Acute pancreatitis 2 Chronic pancreatitis 3 Pancreatic carcinoma 4 Pancreatic abscess 5 Insulinoma

(1) Pancreatitis is an inflammation or infection of the pancreas. Symptoms include abdominal pain (mainly located in the upper abdomen) nausea, vomiting, weakness, sweating, anxiety, fever, clammy skin, and mild jaundice. General examination may show a low blood pressure and a heart rate above 90. Tests should include (those that demonstrate pancreatic enzyme release) elevated serum amylase, elevated urine amylase, and elevated serum lipase. Abdominal ultrasound and CT scan will demonstrate an enlarged pancreas. Blood test may show an elevated white blood cell count, elevated blood glucose, and decreased serum calcium. Treatment is aimed at supportive measures such as fluid replacement by intravenous (IV) infusion, pain relief by analgesics, and withholding food or fluid by mouth to restrict pancreatic activity that makes symptoms worse. Most cases resolve within a week. However, some cases can be life threatening.

27) A 30-year-old man presents to the hospital complaining of a 1-week history of painful tongue. He has also been having diarrhea and forgetting things more easily. He recently emigrated from India and lives alone on a maize-based diet. On examination you note that the tip and margins of his tongue are a bright scarlet color. He also has a symmetrical erythematous rash on his forearms. What is the most likely diagnosis? 1 Vitamin A deficiency 2 Pyridoxine deficiency 3 Vitamin C deficiency 4 Niacin deficiency 5 Vitamin E deficiency

(4) Niacin deficiency causes pellagra, which is characterized by: A symmetrical dermatitis, usually on parts of the body exposed to sunlight Scarlet glossitis and stomatitis Diarrhea Mental aberrations such as memory impairment, depression, and dementia. These may appear alone or in combination. Causes include inadequate dietary intake, especially in patients with corn-based diets or alcoholism. Dietary sources include legumes, yeast, meat, and enriched cereal products.

16) The patient is a 47-year-old male who presents to an urgicare center with 18 hours of abdominal pain, nausea, vomiting, and chills. He is a single construction worker, denies smoking, and has at least a 10-year history of drinking 2-4 alcoholic beverages daily. A series of lab work is performed on the patient to evaluate his abdominal pain prior to abdominal imaging. What laboratory results would be most indicative of the patient suffering from acute pancreatitis? 1 serum amylase 310 U/L and serum lipase 760 U/L 2 serum amylase 250 U/L and serum lipase 110 U/L 3 serum aspartate aminotransferase 32 U/L and serum alanine aminotransferase 29 U/L 4 serum white blood cell count 14,000/mm3 and serum total bilirubin 1.8 mg/dl-acute cholecystitis 5 serum aspartate aminotransferase 120 U/L and serum alanine aminotransferase 40 U/L

(1) The correct answer is a serum amylase of 310 U/L and a serum lipase of 760 U/L, as these levels are significantly elevated, which is indicative of acute pancreatitis. The normal range for serum amylase is 30-220 U/L, and the normal range for serum lipase is 0-160 U/L. Both levels being significantly elevated is typically seen in a patient with acute pancreatitis. Serum lipase is both more sensitive and more specific than serum amylase for diagnosis of acute pancreatitis, but more so when the serum lipase is at least 3 times the normal level (as is with this case). However, various other biliary and intestinal diseases can also alter these lab results.

36) A 65-year-old man has a 2-day history of diffuse, colicky abdominal pain originating in the RUQ (right upper quadrant). It is not aggravated by food or activity. There has been nausea but no vomiting. His skin and eyes have turned yellow in the last several days. His urine has been dark brown, and his stools have been white and chalky. He stopped drinking alcohol about 2 years ago and has been sober since. There is no history of drug abuse. He smokes 2 packs of cigarettes a day. He had gallstones diagnosed at the time he stopped drinking, but he refused surgery. There is a history of previously diagnosed but untreated hypertension. He denies temperature elevation, but has felt "warm." There was an episode of "shaking chills" earlier today. When he walks, he gets dizzy. Vital signs are: T=103 F (oral); P=115/min; BP=100/65 mmHg; R=32/min (labored); Pulse Ox=90% (room air). He is mildly icteric. Abdomen is slightly protuberant, and there is tenderness and rebound tenderness restricted to the RUQ. No surgical scars are present. Chest examination is within normal limits except for scattered wheezes and rhonchi. Cardiac examination reveals mild cardiomegaly. On rectal examination, pale soft stool is obtained, guaiac negative. Good sphincter tone is present. Mild atrophy of the testicles is noted. The remainder of the physical examination, including neurological, is within normal limits. What is the most likely cause of this man's primary problem? 1 Common bile duct stones 2 Smoking 3 Renal calculi 4 Hypertension 5 Alcohol abuse

(1) The most common cause of cholangitis is common bile duct (CBD) stones with obstruction of the CBD. The subsequent development of edema together with the presence of stones produces biliary stasis. Regardless of the source or bacteria, when bacterial infection occurs, cholangitis occurs. Smoking, although not healthy, has yet to be reported as causing biliary stasis, a precursor of cholangitis. Renal calculi could mimic this patient's abdominal pain but is unlikely to produce jaundice. Finally, although malignant severe hypertension is associated with multiple end organ failure, the gallbladder has not been reported as one of the end organs. Liver failure is associated with jaundice and hypotension; it is usually a terminal event, a state this patient has yet to reach.

29) A 60-year-old male presents with a 6-month history of dysphagia to solids, regurgitation of undigested food, and halitosis. He denies a decrease in appetite, abdominal pain, weight loss, or change in bowel habits. His past medical history is significant for a total hip replacement. His lab work is as follows: Hemoglobin: 14.0g/dl Hematocrit: 43% White cell count: 6.0x109/L Platelets: 300x109/L Sodium: 135 mEq/L Potassium : 4.0 mEq/L Urea: 6 mg/dL Creatinine: 0.6 mg/dL Glucose: 98 mg/dL Chloride: 100 mEq/L HCO3: 25 mEq/L His liver panel was normal. The diagnosis can be determined initially by which of the following techniques? 1 Barium Swallow 2 Esophageal manometry 3 MRI of the head and neck 4 Radionuclide scanning 5 Upper endoscopy

(1) The patient may have a Zenker's diverticulum, which may predispose the patient to an aspiration pneumonia. Zenker's diverticulum is an outpouching of the posterior pharyngeal wall immediately above the upper esophageal sphincter. Collection of food particles in the pouch results in halitosis and also the symptom of regurgitation of undigested food particles. The diagnosis is likely to be Zenker's diverticulum since the patient has dysphagia to solids, regurgitation of undigested food particles, and halitosis. Barium swallow is the diagnostic procedure of choice. The best method to determine the diagnosis of a diverticulum is with a Barium Swallow initially, as it is least invasive. Then, as part of the work up before surgery (to reduce the size of the diverticulum), an upper endoscopy is used to rule out an esophageal cancer.

34) A 54-year-old woman presents to the emergency department with nausea, vomiting, right-upper quadrant abdominal pain, fever, and jaundice starting 4 hours ago. During the last 6 months, she suffered several bouts of upper-abdominal pain accompanied by nausea, vomiting, and occasional jaundice, for which medical attention was sought. Her past medical history includes hyperlipidemia, for which she first took simvastatin; she switched to cholestyramine because of side effects. Vital signs on admission are as follows: BP 110 / 80 mm Hg, HR 90 bpm, RR 20 rpm, temperature 38.1°C (100.6 F). She is alert and oriented, and mildly jaundiced. Her right-upper abdomen is diffusely tender to palpation. An upper abdominal ultrasound is performed and reveals a thickening and calcifications of the gallbladder wall, but there are no signs of air within the peritoneal cavity or the bile ducts. What is the most appropriate next step in management? 1 Intravenous hydration and antibiotics 2 Endoscopic papillotomy 3 Give analgesics and refer for elective cholecystectomy 4 Magnetic resonance cholangiography to look for choledocholithiasis 5 Emergency cholecystectomy

(1) The presence of fever, jaundice, and right-upper quadrant pain defines Charcot's triad, which is the classical presentation of acute cholangitis. Reynolds' pentad consists of Charcot's triad plus sepsis/shock and mental status changes. 95% of patients presenting with these syndromes have common duct stones. In most cases, there is a favorable response to conservative treatment, which consists of interrupting oral feeding, analgesia, intravenous hydration, and antibiotics. The following antibiotic schemes can be used: - A third-generation cephalosporin + an amino glycoside - Piperacillin or ampicillin + metronidazole + an aminoglycoside - Monotherapy with imipenem, meropenem, mezlocillin, ampicillin-sulbactam, ticarcillin-clavulanate, or piperacillin-tazobactam After clinical improvement, cholecystectomy is undertaken in the first few days after the initial hospitalization in order to avoid recurrence. Surgery is warranted because the recurrence rate is as high as 10% per year in patients whose gallbladders are not removed. The use of cholestyramine is another risk factor for recurrence in this patient, and gallbladder calcifications (porcelain gallbladder) are thought to pose a risk of malignization and are considered an indication for cholecystectomy.

25) A young American couple is planning to travel to central Africa in 2 months for a 9-10 month stay. They have come to the travel clinic to ask if they need any immunizations. Both have had all of the regular immunizations, including Hepatitis B and tetanus toxoid, and they are up to date. However, there is no record of any hepatitis A vaccination and no history of past disease. What is the best approach for protecting this couple from getting hepatitis A during their trip? 1 Hepatitis A vaccine administration 2 Immunoglobulin administration 3 To draw blood to check for past infection 4 No protection necessary 5 To chlorinate the water

(1) Travelers from industrialized countries are likely to be susceptible to infection with hepatitis A virus (HAV) and should receive prophylactic hepatitis A vaccine before traveling to the risk areas (the areas outside of Australia, Western Europe, Japan, New Zealand, and North America, except Mexico). Hepatitis A is the most common vaccine-preventable infection of travelers. Although the disease is rarely fatal, most infected persons become quite ill and many are unable to work for several weeks or months. Hepatitis A may be acquired from fecal-contaminated food or water, or from direct contact with infected individuals; person-to-person transmission is particularly common among children and between sexual partners. 2 monovalent hepatitis A vaccines are licensed in the United States for people aged more than 12 years and less than 40 years of age. A full vaccination series includes 2 doses, first dose to be administered before travel and giving the second dose 6-12 months (Havrix) or 6-18 months (Vaqta) after the first. All hepatitis A vaccines should be administered intramuscularly.

39) A 65-year-old woman presents with a 2 month history of rectal bleeding, initially thought to be hemorrhoidal in nature. She has a history of hypertension, but is otherwise healthy. Examination is notable for a large circumferential tumor 2 cm from the anal verge. The hemoglobin is 7 gm/dl. A biopsy performed several days later shows anal melanoma. Which of the following is true concerning anal melanoma? 1 Anal melanoma is the most common of all anal cancers 2 A positive S100 stain makes the diagnosis 3 It is a slow growing tumor 4 This tumor metastasizes infrequently 5 The prognosis is good following abdominoperineal resection

(2) Anal melanoma is a rare cancer and accounts for 2% of all anal cancers. Anal melanoma is an aggressive malignancy easily confused with benign hemorrhoids. The presentation is usually that of intermittent rectal bleeding. Sometimes these lesions are not pigmented and may be difficult to identify grossly. Immunohistochemical study of positive S100 stain makes the diagnosis. Unfortunately, melanoma of the anal canal is an aggressive disease. Melanomas commonly metastasize through the lymphatic and hematogenous routes, typically to the liver, lung, bone, and brain. If metastases occur in the GIT, the most common sites include the small bowel, colon, and anorectum. Commonly, the patients have distant disease with metastasis to the lungs and liver. Wide local excision is recommended as primary therapy for primary anorectal melanoma. Comparative studies of the survival of patients who underwent APR with those who underwent wide local excision revealed no statistically significant advantage at all disease stages. Therefore, APR should only be performed when local excision is not possible or for palliative purposes. The prognosis for anorectal melanoma is poor, irrespective of surgical treatment performed. Squamous cell carcinomas are the most common type of anal cancer.

3) A 63-year-old woman presents with acute onset of abdominal pain. She describes it as a steady deep discomfort in the left lower quadrant. Initially she was constipated, but is now experiencing diarrhea. On physical examination, she has a temperature of 38°C. The abdomen is tender in the LLQ with guarding and rebound tenderness. She has positive fecal occult blood. What is the best test to determine her most likely diagnosis? 1 Abdominal ultrasound 2 Abdominal CT 3 Barium enema 4 Upper GI series 5 Colonoscopy

(2) Diverticulitis is a common cause of left lower quadrant abdominal pain that is associated with fecal occult blood. It is best diagnosed with abdominal CT. Findings of sigmoid diverticula, a thickened colonic wall and inflammation within the pericolonic fat suggest the diagnosis. While an ultrasound is less expensive and non-invasive, it is less specific and operator dependent. Barium enema and colonoscopy will both demonstrate diverticula, but there is risk of perforation with these modalities. An upper GI is not indicated in this patient with lower abdominal pain.

28) A 30-year-old woman presents with small wounds on the sides of her mouth for the past 2 weeks. She recently adopted a strict vegetarian diet. On examination, you note she has pale conjunctivae, a magenta tongue, and macerated lips in addition to the angular stomatitis. What is the most likely diagnosis? 1 Thiamine deficiency 2 Riboflavin deficiency 3 Vitamin K deficiency 4 Vitamin D deficiency 5 Iron deficiency

(2) In Riboflavin or Vitamin B2 deficiency, patients present with angular stomatitis and cheilosis. On examination, they are pale, have atrophic glossitis, and the tongue may appear magenta. They may also have a sebaceous dermatosis with greasy material in their nasolabial folds, alae nasi, and genitals. Causes include inadequate dietary intake. Dietary sources of riboflavin include milk, cheese, meat, and enriched cereal products.

2) A 56-year-old female visits her rheumatologist complaining of heartburn and dyspepsia. She was diagnosed with osteoarthritis 4 years earlier and for the past 18 months has been managing pain with naproxen. An upper gastrointestinal endoscopy shows several areas of gastric irritation and a single ulcer about 3 mm in diameter. The gastroenterologist suggests that the patient be tested for Helicobacter pylori infection. What is the gold standard for diagnosis of this infection? 1 Culture 2 Histology 3 Antigen test 4 Western blot serology 5 Urease breath test

(2) The long term use of nonsteroidal antiinflammatory drugs (NSAIDS) in treatment of arthropathies carries about a 4-fold increase in the relative risk of duodenal or gastric ulcers. This risk is multifactorial; one risk factor for development of gastric ulcers in patients taking NSAIDS is infection with Helicobacter pylori. Histologic examination of gastric biopsies obtained during endoscopy is the gold standard for diagnosis of this infection based on sensitivity and specificity (refer to the table). This approach has the added benefit of allowing simultaneous assessment of the pathology of the lesion, which can rule out neoplasms. Culture of H. pylori is difficult and requires considerable microbiologic expertise. However, indicators in the cultivation medium can detect urease. Invasive procedures can be preceded by the urease breath test as a screening for the presence of the organism. Helicobacter serology by ELISA is limited to screening and detection of a past or present infection. An antigen test is also available that detects H. pylori protein in the stool.

30) A 32-year-old female is referred to you by a psychiatrist because of decreased vibratory sensation. Her friend had insisted that she see a psychiatrist because she has been 'depressed' and 'slow' as of late. She has no prior or family history of depression and denies fever, weight loss and has no history of a major life event. Leukocytes count: 6,600/μL Segmented neutrophils: 65% Hemoglobin: 8.1g/dL Mean corpuscular volume: 116 fl Platelet count: 145,000 /μL Serum glucose: 106 mg/dl Sodium: 146 mmol/L Potassium : 3.9 mmol/L Chloride: 108 mmol/L Bicarbonate: 23 mmol/L Total bilirubin: 1.8 mg/dL Which of the following is a possible finding that may account for this patient's features? Answer Choices 1 Alcoholism 2 Chronic atrophic gastritis 3 Low serum ferritin level 4 Menorrhagia 5 Methotrexate therapy

(2) The patient has vitamin B12 deficiency. The suggestive features are depressive symptoms, decreased vibratory sensation, vitiligo, smooth tongue, anemia with macrocytic RBC, evidence of hemolysis (high bilirubin and LDH), and a low vitamin B12 level. A normal red cell folate level and the presence of neurologic manifestations rule out folate deficiency as the cause of the macrocytic anemia. Of all the listed options, the most likely cause is chronic atrophic gastritis. This is seen in patients with pernicious anemia. They have deficiency of intrinsic factor, which is necessary for the absorption of vitamin B12 from the gut. The gastritis is of the 'A' type and affects the body and fundus but spares the antrum. They also have anti-IF antibody, and some have anti-parietal cell antibody. Treatment is by parenteral vitamin B12 supplementation. With correction, reticulocytosis will be noted after a week, and hemoglobin will start to rise after about 6 weeks; there may not be a full recovery from the neurologic manifestations, however.

Cilastozol contraindication

CHF

CHADS

CHF, HTN, Age > 75, Diabetes, Stroke; 2+ score = anticoagulate

12) A 46-year-old woman presents with a 2-month history of heartburn, epigastric discomfort, nausea, and occasional vomiting. She has a history of hyperlipidemia, controlled with diet and exercise, as well as asthma, for which she takes inhalers as needed. She takes no other medications, including over-the-counter analgesics. Family history is noncontributory. On exam she is afebrile, BP120/70 mm Hg, pulse 74/min, and SPO2 92%. Lungs are clear, and she has minimal epigastric tenderness. Otherwise, physical examination is unremarkable. She is advised by her physician to take lansoprazole once daily, which provides only partial relief. Endoscopy is then recommended, which shows a duodenal ulcer. Biopsy reveals infection with Helicobacter pylori. What would be the recommended regimen at this time? 1 Lansoprazole, amoxicillin, and metronidazole twice daily for 2 weeks 2 Pantroprazole, amoxicillin, and clarithromycin twice daily for 2 weeks 3 Omeprazole, tetracycline, and clarithromycin twice daily for 2 weeks 4 Bismuth, metronidazole, and lansoprazole twice daily for 2 weeks 5 Bismuth, metronidazole, and tetracycline 4 times daily for 2 weeks

(2) There are several regimens recommended for H. pylori infection, which is an important cause of peptic ulcer disease and should be treated if found associated with symptoms. The choice of the regimen depends on considerations such as cost, side effects, and ease of administration. Allergy to one of the medications, as well as intolerance, should also be taken into account. Any proton pump inhibitor (PPI) with amoxicillin 1000 mg twice daily and clarithromycin 500 mg twice daily for 2 weeks or PPI with metronidazole 500mg twice daily and clarithromycin 500mg twice daily for 2 weeks are recommended.

23) A 38-year-old man presents with sudden onset of acute upper abdominal pain since the previous night associated with nausea, several episodes of vomiting, and weakness. The pain is mostly in the epigastric region with constant, severe, and steady radiation to the back. He also has a low grade fever since this morning without any chills. He denies diarrhea or dysuria. His past history is significant for hypertension, for which he takes amlodipine 10 mg daily and enalapril 5 mg daily. He has smoked half a pack of cigarettes daily for the past 12 years. He initially denies drinking alcohol except on the weekends occasionally, but on further and repeated questioning he says he drinks 3-4 beers daily and had been drinking continuously for the last 2 days with his friends while watching sports on TV. Family history is unremarkable. On examination he has a temperature of 100.6° F, pulse 106/minute, BP 150/92 mm Hg, and respiratory rate is 20/minute. There is no pallor, icterus, cyanosis, or lymphadenopathy. Mucus membranes are dry, and skin is somewhat clammy. Lungs are clear, and heart sounds normal except for sinus tachycardia. Abdominal exam reveals diffuse tenderness in the epigastric and right as well as left upper quadrants. There is some distension and mild guarding in the upper abdomen. Bowel sounds are hypoactive, but there is no ascites or hepatosplenomegaly. Rectal exam is normal. Labs reveal Hb 15g%, WBC 14,500/uL, platelets 400,000/uL, AST 42 U/L, ALT 36 U/L, AP 26 U/L, amylase 3600 U/L, lipase 546 U/L, BUN 25 mg/dL, creatinine, 1.5 mg/dL, bilirubin 1.2 mg/dL, and random blood sugar 110 mg/dL. Which of the following is one of the predictors of acute pancreatic necrosis if present at diagnosis along with 2 other factors? 1 Age over 50 years 2 WBC count more than 16,000/uL 3 Blood glucose over 180mg/dL 4 Serum LDH over 300 U/L 5 AST more than 200 U/L

(2) This patient is suffering from acute alcoholic pancreatitis. The 2 most common causes of acute pancreatitis are gallstones and alcohol. Other causes include hypertriglyceridemia, hypercalcemia, abdominal trauma, ERCP, and drugs like valproic acid, azathioprine, mercaptopurine, didanosine, thiazides, tetracyclines etc. Assessment of severity is done by either Ranson's criteria on admission and at 48 hours or by the Acute Physiology and Chronic Health (APACHE) II scoring system. Ranson's criteria include: (1) Age more that 55 years (2) WBC count more than 16,000/Ul (3) Blood glucose more than 200mg/dL (4) Serum LDH over 350 U/L (5) AST over 250 U/L.

A 52-year-old woman was noted on yearly examination to have a microcytic anemia. She has a long history of constipation and describes occasional hemorrhoidal bleeding, but otherwise denies any melena. She has no prior surgical history. Her only medical problem is an elevated cholesterol level that is controlled by diet. Her pulse is 92 BPM, blood pressure is 140/78 mm Hg, respiration rate is 14/min, and temperature is 98.7°F. Upon physical examination, no cervical or supraclavicular adenopathy is present and her head, eyes, ears, neck, and throat are normal. Her lungs are clear with equal breath sounds. Her heart has a regular pulse rate and rhythm, no murmurs, rubs or gallops. Abdominal exam reveals no organomegaly or masses. Rectal exam is notable for guaiac positive stool without any masses. Neurological examination is normal. Here are her labs: WBC/7,400 µL; Hemoglobin/10.2 g/dL; Hematocrit/30.4 g/dL; Platelets/245,000; Na-/138 mEq; K/3.8 mEq/L; Cl-/100 mEq/L; HCO3/27 mEq/L Subsequent studies reveal a sigmoid colon carcinoma. What should be the first treatment modality for this disease? 1 Radiation 2 Surgery 3 Chemotherapy 4 Observation 5 Chemotherapy followed by radiation

(2) Treatment of a sigmoid colon carcinoma consists at the very least of surgical therapy, and depending on the pathologic characteristics of the lesions, the decision would be where chemotherapy would be beneficial. Neither preoperative chemotherapy nor radiation have been shown to have any advantage over surgery in the treatment of colon carcinoma. Pre-operative chemotherapy and radiation may be beneficial in some patients with rectal carcinoma.

40) An 8-month-old female infant presents with a 2-day history of increasing irritability and decreased appetite. She has also had some diarrhea and low grade fever. On exam she is afebrile, very fussy, and hard to console; however, she appears alert and active. She lies with her hips and knees flexed, crying harder with any movement. A few petechiae are noted on her skin. Leg X-rays are done to look for a possible fracture that shows a pencil-thin cortex and a ground glass appearance of the bones. Further history reveals that she has been given evaporated milk since birth to save money, and she has not yet been started on solids. What vitamin deficiency would most likely cause this infant's symptoms? 1 Vitamin B6 2 Vitamin C 3 Vitamin A 4 Vitamin B2 5 Vitamin E

(2) Vitamin C is involved in cellular oxidation through reduction reactions; it is required for the normal growth and maturation of cells. It helps form the ground substance among cells of the capillary walls, collagen, and osteoid tissue. Vitamin C is abundantly present in citrus fruits, spinach, cauliflower, liver, and kidney. Scurvy is characterized by pinpoint spots on the skin (especially on the thighs and legs) called petechiae, spongy gums, poor healing of bruises, joint pain, bleeding from mucous membranes, and thin hair. It can occur in infants whose mothers have a dietary deficiency in vitamin C and also in infants fed unsupplemented evaporated milk. Infants deficient in the vitamin may present with fever, diarrhea, appetite loss, poor weight gain, and protein depletion. Irritability, generalized tenderness causing pseudoparalysis, frog position of the legs, peripheral edema, swelling of gums, scorbutic beads in the ribs, and petechial hemorrhages can also be seen. Bone involvement is typical in infantile scurvy. Radiographic changes include ground glass appearance of the bones, pencil-thin cortex, a zone of calcified cartilage at the metaphysis, and a zone of rarefaction proximal to this. Administration of 100 to 200 mg/day of ascorbic acid for 1 week will promote quick healing. The daily requirement of vitamin C for infants 7 to 12 months-old is 50 mg.

37) A 32-year-old woman presents with a pins and needles sensation on the sides and the bottom of her feet, as well as severe burning sensation in the feet. The burning sensation manifests primarily at night. Examination reveals a profoundly cachectic but alert patient with atrophy of muscles of the leg. Examination of the eye indicates that she also has nystagmus. Further questioning reveals that she has a history of homelessness and chronic alcoholism. What vitamin deficiency is most commonly associated with chronic alcoholism? 1 Vitamin C 2 Vitamin B1 3 Niacin 4 Vitamin E 5 Vitamin B6

(2) Wernicke's encephalopathy is a result of the inadequate intake or absorption of thiamine (Vitamin B1) coupled with continued carbohydrate ingestion. It is most common among chronic alcoholics who suffer from poor nutrition. It is therefore imperative that all chronic alcoholics receive a dose of thiamine before they receive any glucose solution IV. It is also seen in patients on dialysis and during hyperemesis gravidarum of pregnancy. Clinical features appear acutely and include ataxia, confusion, nystagmus, partial ophthalmoplegia, apathy, confusion, severe memory loss, as well as autonomic dysfunction.

1) A 36-year-old Jewish man complains of abdominal pain and diarrhea, as well as a 2-day history of a low-grade fever. He is initially treated with fluids and antibiotics, but his symptoms worsen. On evaluation, he is diagnosed with Crohn's disease. He is a nonsmoker but has alcoholic drinks on alternate days and had undergone appendectomy 2 years back. His mother was diagnosed with irritable bowel syndrome (IBS) 10 years back. What in his history is a risk factor for Crohn's disease? 1 His age 2 Alcohol consumption 3 His Jewish ancestry 4 Family history of IBS 5 Appendectomy

(3) Crohn's disease (CD) is more common in the Jewish population than others. It is more common in whites than blacks. Environmental factors may play a role in its etiopathogenesis because African-Americans seem to have the same risk as whites, but African blacks have a lower risk. The age of onset is between 15 to 30 years with a second peak between 60 to 80 years. CD is an inflammatory bowel disease affecting any part of the GIT from mouth to anus. The most commonly affected region is the ileocecal region. Symptoms include abdominal pain, diarrhea (which may be bloody), vomiting, fever, and weight loss. Extraintestinal symptoms such as arthritis, uveitis, etc., may also develop. Smoking, not alcohol, is a risk factor for CD. Family history of IBS is unrelated to CD; however, family history of CD itself is a risk factor. Appendectomy neither increases the risk nor is protective in CD.

14) A 30-year-old man presents with severe nausea and vomiting that started about 1 hour ago. Prior to falling ill, he had been at a party where he enjoyed pudding along with other food. Physical examination revealed normal temperature with mild diffuse tenderness of the abdomen. The organism isolated is a Gram-positive coccus that occurs in grape-like clusters, is catalase and coagulase positive, and forms golden yellow colony on agar. What toxin released by the causative organism is responsible for the patient's symptoms? 1 Toxic shock syndrome toxin (TSST-1) 2 Exfoliatin toxin 3 Enterotoxin 4 Leukocidin 5 Alpha toxin

(3) Enterotoxin produced by Staphylococcus aureus is an exotoxin responsible for the patient's condition. The clinical presentation and the laboratory findings are suggestive of staphylococcal food poisoning. A short incubation period (1-6 hours) with predominant emesis suggests Staphylococcal food poisoning. S. aureus organisms are Gram-positive cocci that occur in grape-like clusters. They are catalase and coagulase positive, and they form golden yellow colonies on agar. Staphylococcal food poisoning results from ingestion of preformed enterotoxins on food contaminated with S. aureus. Bacteria growing in carbohydrates and meat products produce enterotoxins that, upon ingestion, diffuse into the circulation and cause emesis by stimulating the vomiting center in the central nervous system.

5) A 14-year-old girl presents with a 4-day history of flatulence, foul smelling stools, and abdominal distention. Her appetite has been decreased as well. She has not seen any blood in her stools. She returned from a 2-week camping trip in the mountains of the western United States 1 week ago. Others in her expedition group are asymptomatic. Her physical examination reveals a well-developed, well-nourished adolescent with slight abdominal distention and tenderness; otherwise, everything is within normal limits. What is the most likely cause of her symptoms? 1 Shigella flexneri 2 Staphylococcus aureus 3 Giardia lamblia 4 Salmonella typhimurium 5 Rotavirus

(3) Giardia lamblia is a flagellate protozoon that exists in trophozoite and cyst forms. The infective form is the cyst. Infection is found in the small intestine and the biliary tract. Symptomatic infections cause a wide array of clinical disease, ranging from asymptomatic infections to occasional acute watery diarrhea to protracted illness characterized by flatulence, foul-smelling stools, abdominal pain and distention, and anorexia. Humans are the main reservoir of infection, but organisms can infect dogs, cats, beavers, and other animals. Contaminated water supplies, mountain streams, and person-to-person contact are major sources for infection. The incubation period is 1 to 4 weeks. The most widely used diagnostic tests include direct smear examination of stool specimens for trophozoites or cysts and immunofluorescence antibody testing of stool specimens or duodenal fluid. Treatment includes correction of any dehydration or electrolyte abnormalities present. Metronidazole is the drug of choice. Other alternatives include furazolidone, albendazole, and nitazoxanide.

32) A 45-year-old woman presents for initial history and physical at her new primary care provider's office. She is currently asymptomatic, but based on her sexual history with multiple partners, you decide to obtain screening hepatitis lab tests in addition to other screening tests. The following results are obtained: Hep A Ab: negative HBcAb: positive HBeAg: negative HBsAg: positive HBsAb: positive Hep C Ab: positive Hep D Ab: negative What is the most likely diagnosis based on these results? 1 She has not been infected with or been vaccinated against Hepatitis B virus but has been exposed to Hepatitis C 2 She has been vaccinated against Hepatitis B in the past but is not currently infected with Hepatitis B nor exposed to Hepatitis C 3 She has had Hepatitis B infection in the past, is currently infected with Hepatitis B, and has been exposed to Hepatitis C 4 She has had Hepatitis B infection in the past but is not currently infected and has been exposed to Hepatitis C 5 She has been vaccinated against Hepatitis B but has not been infected with it and has been exposed to Hepatitis C

(3) HBcAb, HBsAg, and HBsAb become positive with Hepatitis B virus infection, but over time HBsAg will clear from the serum only if the patient is not chronically infected. These results demonstrate chronic infection, as all three are positive. She also shows exposure to Hepatitis C, as the surface antibody is positive. The remaining answers are incorrect because vaccination alone will cause only the HBsAb to become positive. All Hepatitis B results would be negative if the patient had not had Hepatitis B virus infection or been vaccinated for Hepatitis B virus. Also, only HBsAb would be positive if the patient was never infected but had received the vaccine.

9) A 53-year-old man 2-day history of jaundice and malaise. His history is significant for mucosal candidiasis, for which the patient is on oral ketoconazole 200 mg daily for the past 3 weeks. He occasionally has headaches, and sometimes takes paracetamol in a daily dose of 1 gram to achieve headache relief. Two weeks before the appearance of symptoms, the patient was treated with flucloxacillin for respiratory tract infection. Except for the appearance of jaundice and malaise, the patient denies the presence of any other symptoms, and the remainder of his personal history is unremarkable. Physical examination reveals a mildly jaundiced patient, 180 centimeters tall, 82 kilograms in weight. His blood pressure is 110/86 mmHg, and the remainder of his general physical examination revealed no abnormalities. Laboratory analyses reveal the presence of hyperbilirubinemia and elevated serum transaminases, normal alkaline phosphatase and γ-Glutamyl transferase levels. Other routine laboratory analyses reveal no abnormalities. Serological testing does not reveal the presence of antibodies against human immuno-deficiency, hepatitis A, C, D and E, or viruses in patient's serum. Also, HBsAg, anti-HBs or anti-HBc antibodies are not present in patient's serum. Anti-LKM-1, antinuclear, anti-thyroid, antimitochondrial, and anti-smooth muscle antibodies are absent. What is the most likely cause of liver dysfunction in this patient? 1 Paracetamol-induced hepatitis 2 Primary sclerosing cholangitis 3 Ketoconazole caused hepatitis 4 Flucloxacillin-induced hepatitis 5 Primary autoimmune hepatitis

(3) Ketoconazole is an antifungal imidazole used in treating systemic mycoses. The common adverse effect of ketoconazole is hepatotoxicity. The severity of ketoconazole-induced hepatotoxicity is linked to the exposure level of the drug. Ketoconazole-induced hepatotoxicity is probably mediated through a reactive metabolite N- deacetyl ketoconazole (DAK). The latter appears to be the major metabolite, which is a hepatic cytotoxic. Ketoconazole administration results in a significantly dose-dependent increase in serum transaminase activities, as well as cloudy swelling, ballooning degeneration and centrilobular confluent necrosis of the hepatocytes. The histological feature ranges from acute hepatitis to confluent centrilobular, or massive necrosis.

33) A 21-year-old Caucasian female goes to a physician complaining of weakness, fatigue, palpitations, and postprandial abdominal pain with bloating, flatulence, and occasional diarrhea with voluminous stools. She received oral iron for iron-deficiency anemia 4 years ago but says she has been feeling weak again. Three months ago, pruritic papules and vesicles appeared on both elbow surfaces. Her family history is unremarkable. Vital signs are within normal ranges. On physical examination, there is noticeable skin pallor. Excoriated papules and blisters are seen on both elbows. The remainder of the examination is normal. A complete blood count shows the following: Ht 31% Hb 9.8 g/dL MCV 78 fl MCHC: 25 g/dL WBC 8.200 /mm3. w/ normal differential iron 54 µg/dL ferritin 5 µg/dL. What is displayed in the attached image from the endoscopic examination of the duodenum? 1 Ectatic lymphatic vessels below the mucosal surface 2 Large quantities of Gram-negative and Gram-positive bacteria 3 Flattened vili and a submucosal lymphocytic infiltrate 4 PAS-positive macrophages containing Gram-positive cocci 5 Submucosal infiltration with aberrant lymphocytes 6 Flagellated protozoa on the mucosal surface

(3) The hallmark of celiac sprue is an immunologic reaction to gluten. Endoscopic examination of the duodenum shows discrete but characteristic mucosal granulation (see image in the question text). Histopathology reveals villus flattening and lymphocytic proliferation in the submucosa. These mucosal alterations also produce secondary lactase deficiency. Celiac disease is a common cause of steatorrhea. The classical presentation consists of diarrhea with bulky, foul-smelling stools, abdominal distention, and weight loss. However, most patients don't develop the full-blown syndrome. The presence of a subset of these symptoms should trigger suspicion of celiac disease.

22) How is a sigmoid volvulus treated? 1 Barium enema 2 Emergency colon resection 3 Rigid proctosigmoidoscopy 4 Oral purgatives 5 Bowel stimulants

(3) When sigmoid volvulus causing large bowel obstruction is suspected from the history and physical exam, or when it is seen on an abdominal radiograph, the diagnostic and therapeutic modality is rigid proctosigmoidoscopy with decompression and untwisting of the volvulus.

4) A 39-year-old CFO of a finance company is seen for upper abdominal discomfort. She experiences heartburn, belching, and epigastric burning pain on and off for the last several months. She does say that her food habits are irregular, due to the high stress at her job, and her caffeine intake is more than she thinks is healthy. She does not smoke or drink. She has no medical problems and is on no medications. On exam she is afebrile, pulse is 78/min, BP 110/70, height is 5'6, and weight 135 pounds. Further exam reveals some epigastric tenderness but is otherwise normal. She is advised a trial of lansoprazole by her doctor and follow-up in 2 weeks. However, she says that she has been reading about her symptoms and is concerned about infection with helicobacter pylori. What is the most cost-effective initial measure to diagnose this? 1 Referral to gastroenterologist for endoscopy 2 Empiric treatment with triple therapy 3 Urea breath test 4 Rapid antibody test 5 Fecal antigen tes

(4) An office based rapid antibody test is still considered the most cost effective mode of diagnosis of infection with H. pylori, especially in a patient not treated in the past for the same. This test is simple, inexpensive, and sensitive, as well as specific. However, it is not useful as a follow-up test to confirm eradication of infection. Other tests include stool antigen testing, which is also inexpensive, noninvasive, highly sensitive, and specific, and can be used as a follow-up test as well. This infection is common in the general population. It is the cause of 50-80% of peptic ulcers, and incidence of ulcer disease in those with the infection is 20%. Testing is indicated in active or documented past history of peptic ulcer disease and gastric mucosa associated lymphoid tissue (MALT) lymphoma. Many physicians test patients with functional dyspepsia, as this patient, though it is still controversial. However, if testing is done and it is positive, treatment with triple therapy must be done.

17) A 23-year-old man presents with difficulty in walking and frequent falls for the last 3 months. He gives history of occasional alcohol consumption and regular cigarette smoking (6 per day). He also has 3-month history of increased thirst and increased frequency of urine. There is no history of fever, weight loss, and no history of any major illness in the past. He has mild palpitations on walking. On examination, pulse 86/min and blood pressure 130/80mm of Hg are noted. Abdominal, respiratory, and cardiovascular system examination is within normal limits. CNS examination shows nystagmus, loss of fast saccadic eye movements, truncal titubation, absent deep tendon reflexes, and upward plantar reflexes. There is weakness in distal limb muscles and some sensory loss of fine touch and vibrations. He has ataxic gait, but no spasticity. His CBC was within normal limits, blood sugar fasting is 120mg%, and Postprandial is 168 mg%. MRI of brain is normal This condition is associated with a defiency of what vitamin? 1 Vitamin A 2 Vitamin B1 (Thiamine) 3 Vitamin D 4 Vitamin E 5 Vitamin K

(4) Friedreich's ataxia may be associated with vitamin E deficiency. 2 types of vitamin E deficiency can occur in Friedreich's ataxia: The first is abetalipoproteinemia (Bassen-Kornzweig syndrome), which is associated with a defect in the microsomal triglyceride transfer protein. This results in the impairment of formation and secretion of VLDL in the liver and a deficiency of delivery of vitamin E to the tissues, including CNS. The second type, called ataxia with vitamin E deficiency (AVED), is due to a defect in a tocoferol transfer protein and an impaired ability to bind the vitamin E to VLDL. There are 2 forms of Friedrich's ataxia- the classic form and the form associated with Vitamin E deficiency.

A 35-year-old man presents for initial history and physical at his new primary care provider office. He is currently asymptomatic but based on his sexual history, you decide to obtain screening hepatitis lab tests in addition to other screening tests. The following results are received from the laboratory: Hep A Ab: negative HBcAb: positive HBeAg: negative HBsAg: negative HBsAb: positive Hep C Ab: negative What is your most likely diagnosis based on these results? 1 He has not been infected with or been vaccinated against Hepatitis B virus in the past 2 He has been vaccinated against Hepatitis B in the past 3 He has had Hepatitis B infection in the past and is currently infected 4 He has had Hepatitis B infection in the past but is not currently infected 5 There is not enough information to make a determination

(4) HBcAb, HBsAg, and HBsAb become positive with Hepatitis B virus infection but over time HBsAg will clear from the serum if the patient is not chronically infected as demonstrated by the results above while the others will typically remain positive. The remaining answers are incorrect because vaccination alone will cause only the HBsAb to become positive. All Hepatitis B results would be negative if the patient had not had Hepatitis B virus infection or been vaccinated for Hepatitis B virus.

31) What is the leading cause of fatal accidental poisoning among children 6 years old and under in the United States? 1 Alcohol 2 Bleach 3 Antifreeze 4 Iron pills 5 Salt

(4) Iron supplements are the leading cause of fatal accidental poisoning among children 6 years old and under in the United States. It is imperative that the healthcare provider counsel the patient to keep his/her supplements out of the reach of children.

18) A full-term neonate with an Apgar score of 9 at birth developed abdominal distension with watery diarrhea and flatulence following the initiation of milk feeding. The infant became normal when the milk feeding is terminated. What is the most likely diagnosis in this case? 1 Celiac sprue 2 Tropical sprue 3 Whipple disease 4 Disaccharidase deficiency 5 Amebic dysentery

(4) The disaccharidases, of which the most important is lactase, are located in the apical cell membrane of the villous absorptive epithelial cells, the deficiency of which leads to the incomplete breakdown of the disaccharide lactose into its monosaccharides, glucose, and galactose, leading to diarrhea from the unabsorbed lactose. Bacterial fermentation of the unabsorbed sugars leads to increased hydrogen production, which is readily measured in the exhaled air by gas chromatography. There are both hereditary and acquired forms. Histologically in both, there is no abnormality of the mucosal cells of the bowel. Malabsorption is promptly corrected when exposure to milk and milk products is terminated.

21) Your 88-year-old patient informs you that it has been over 10 years since her last colonoscopy (which was negative) and it is time for another. She reminds you that she has always kept up with all her health screening and wants you as her primary care physician to refer her to "a good gastroenterologist to make sure [she does not] have any colon cancer." You review her history and note that she has been in generally good health. She has had no weight loss, no change in her bowel habits, and no dark stool or rectal bleeding. There is no family history of colon cancer. She is eating well and continues to remain active, although slowed down a bit due to arthritis. She takes acetaminophen, an occasional ibuprofen, a multivitamin, and olmesartin/hydrochlorothiazide for her high blood pressure. Based on the most recent recommendations from the US Preventive Services Task Force, you explain to her that: 1 Fecal occult testing is adequate for colon cancer screening given her negative history. 2 CT colonography would be preferable due to the risks of colonoscopy at her age. 3 Colonoscopy should be done, but she should be aware of the increased risks of the procedure in patients over age 70. 4 Screening for colon cancer in asymptomatic patients over age 85 is not recommended because the risks outweigh the benefits. 5 Flexible signioidoscopy with fecal DNA testing would be the best screening method for her.

(4) The most recent guidelines from the US Preventive Services Task Force (USPSTF) recommend against screening for colon cancer in asymptomatic persons over 85 who have had adequate screening in the past. The Task Force states that screening should increase the number of life years gained with the least amount of risk. It is felt that for persons over 85, death rates would be higher due to the likely presence of other conditions, reducing any potential benefit screening would have on mortality. In these cases, the risks of screening would outweigh the benefits.

38) A 69-year-old edentulous alcoholic man who lives alone presents for evaluation of a shoulder wound that is not healing well. On physical examination, numerous ecchymoses are noted on the posterior aspect of his legs and thighs. Careful examination of the man's skin reveals minute hemorrhages around hair follicles and splinter hemorrhages in the nail beds. Laboratory examination is remarkable for a hemoglobin of 10 (normal 14-18 g/dL); no other hematologic abnormalities are noted. What treatment should be included in therapy? 1 Administration of factor VIII 2 Administration of iron 3 Administration of vitamin B12 4 Administration of vitamin C 5 Administration of vitamin K

(4) The patient suffers from scurvy due to a deficiency of dietary vitamin C. Absence of vitamin C leads to impaired hydroxylation of proline residues in the nascent procollagen chains, leading to weakness of blood vessel walls. Clinically, the deficiency syndrome is characterized by perifollicular hemorrhages, fragmentation of hairs, purpura, ecchymoses, splinter hemorrhages, and hemorrhages into muscle. In patients with normal dentition, gum changes (swelling, bleeding, loosening of teeth) are also noted. Without supplementation with vitamin C, death may eventually occur.

8) A 15-year-old boy presents with abdominal pain and rectal bleeding. His family history is significant for premature death in his mother and maternal grandmother from metastatic colon cancer. Both died before the age of 35 years; in both, autopsy findings included hundreds of colon polyps along with multiple primary colon cancers. Endoscopy of the boy also demonstrates extensive colonic polyp disease. A mutation in what gene caused the patient's symptoms? 1 BRCA1 2 BRCA2 3 MSH2 4 APC 5 RET

(4) The presentation of hundreds of colonic polyps, often 'carpeting' the large bowel, in multiple family members is typical for familial adenomatosis polyposis syndrome. The typical course is for polyps to develop in late childhood and the early teenage years, with transformation of 1 or more polyps to colon cancer in the third or fourth decade. Mutations in the APC gene, a known tumor suppressor gene, cause this syndrome.

13) A 63-year-old woman with a history hypertension and gastro-esophageal reflux disease (GERD) also has troubling osteoarthritis in her right knee. Her pain has not been controlled by acetaminophen (Tylenol). You decide to prescribe a non-steroidal anti-inflammatory drug (NSAID). What NSAID would be best for this patient based on the likelihood of causing the least gastrointestinal bleeding? 1 Ketorolac (Toradol) 2 Meloxicam (Mobic) 3 Indomethacin (Indocin) 4 Ibuprofen (Motrin) 5 Naproxen (Naprosyn)

(4) The risk of upper gastrointestinal (GI) bleeding varies between individual NSAIDs at the doses commonly used in the general population. Drugs that have a long half life or slow release formulation are associated with greater risk. Drugs that cause more profound inhibition of both cyclooxygenase (COX) isozymes are also associated with greater risk. Ibuprofen has the lowest reported relative risk of upper GI bleeding at 2.7.

24) A 6-week-old male infant presents with a 2-day history of vomiting after every feeding of cow's milk-based formula with iron, in the amount of 4 ounces per feeding. There has been no fever, diarrhea, or other symptoms except increased crying. The child appears alert and hungry. The mother describes the vomiting as forceful, traveling about 2 feet. Physical evaluation reveals minimal tear production with mild skin tenting. Bowel sounds are decreased. BUN was 29 mg/dl; serum sodium was 129 mg/dl; serum potassium was 3.4 mg/dl; serum chloride was 89 mg/dl; and serum bicarbonate was 34 mg/dl. What is the next step in this patient's care? 1 Exploratory laparotomy 2 Gastric aspirate for Helicobacter pylori 3 High dose intravenous methylprednisolone 4 Intravenous correction of dehydration and electrolyte imbalance with simultaneous abdominal ultrasonography 5 Trial of oral rehydration

(4) While the above scenario is highly suggestive of infantile hypertrophic pyloric stenosis (IHPS) (age 6 weeks, male, projectile vomiting with each feeding, hypochloremia with metabolic alkalosis), the need for surgery is not emergent. The child should be placed NPO, and dehydration and electrolyte abnormalities should be corrected, after which a pyloromyotomy can be performed.

15) A patient with a history of alcohol abuse comes to your office with his histopathological report asking about the "bodies found in his liver cells." What bodies is he talking about? 1 Russell bodies - protein inclusions 2 Lewy bodies - protein aggregates 3 Negri bodies - eosinophilic inclusions 4 Heinz bodies - hemoglobin precipitations 5 Mallory bodies - hyaline inclusions 6 Guarnieri bodies - intracytoplasmic inclusions

(5) Mallory bodies called alcoholic hyaline are cytoplasmic inclusions found most often in alcoholic hepatitis. They are abnormal aggregations of cytoskeletal proteins. Russell bodies are found in reactive or malignant plasma cells, not in liver cells. They represent the excess immunoglobulin.

Treatment of bacterial meningitis in a adults older than 55?

Ampicillin, Cefotaxmine and vancomycin

Treatment of Enteric Fever (Typhoid fever)

Ampicillin, and bactrim are developing resistance. Ceftriaxone or fluoroquinolone will work.

Treatment of bacteremia

Ampicillin, chloramphenicol, and bactrim are developing resistance. Ceftriaxone or fluoroquinolone will work.

20) What food sources of calcium can you suggest to a patient who is lactose-intolerant? 1 Swiss cheese 2 Brie 3 Whole milk 4 Fat-free milk 5 Tofu

(5) People who are lactose-intolerant are unable to produce lactase, an enzyme vital to the digestion of lactose. Tofu is essentially soybean curd; therefore, it is not made out of milk and does not contain lactose. Tofu will not pose a problem for those who are lactose-intolerant. Tofu, along with sardines and almonds, are wonderful non-dairy sources of calcium. The enzyme lactase breaks down the disaccharide lactose into monosaccharides, glucose, and galactose. These monosaccharides are then absorbed in the small intestines. Often, people who are lactose-intolerant who eat/drink a lactose-containing product will experience nausea, diarrhea, abdominal cramping, and excessive gas. Swiss cheese, brie, whole milk, and fat-free milk all contain lactose; they will not be properly digested by lactose-intolerant individuals.

Dilatation and impaired contraction of one or both ventricles"

...explains dilated cardiomyopathy.

What is the chances of contracting HIV from a needle stick from a known HIV patient?

0.3%

A 7-year-old girl presents because her parents are concerned about her growth. Her prenatal history was normal; she sat up when she was 6 months old, walked without assistance at 10 months, and all other milestones of the development were normal. Her immunizations are up to date, and she had no serious illness. On examination, you find a cooperative and friendly child. Except for a short stature, physical findings are normal. What will be your next diagnostic step?

Ask about parent's growth

Initial energy used for cardioversion in an atrial flutter patient

1 and only 1 direct current moderate-energy 100 Joules electric shock.

Time Fram for Charcoal use

1 hour

Tuberculosis cutoffs: 5, 10, 15

> 5 is positive in immunocomprimised >10 is positive in children, nursing homes, correctional facilities, homeless, immigrants, IV drug users, hospital workers, chronic illnes > 15 is positive in everyone else

What medication should be used to arrest the development of Parkinson's disease?

A monoamine oxidase D inhibitor (Selegiline)

Patients with dyslipidemia and significant risk of CAD whould be placed on what medication?

Aspirin 81-325 mg daily

What are abortive treatment options for migraines?

Aspirin, acetaminophen, NSAID's, isometheptene or caffeine.

How do you treat tension headaches

Aspirin, acetaminophen, or NSAIDs

Treatment of sleep apnea?

CPAP continuous positive airway pressure

Threshold for prophylactic platelet transfusion?

10,000

What level should you correct Hb to in males with chronic kidney disease?

10-12 (normal levels show adverse effects)

Treatment of hypermagnesium

10-20 mL of 10% calcium gluconate IV over 10 minutes. Saline diuresis and IV furosemide may excrete excessive magnesium.

Abortive treatment of cluster headaches

100% oxygen, ijectable forms of ergotamines or supatriptans and analgesics

H/o diabetes, MI, chest pain, CAD or Men > 45 and Women > 55: Preop Clearance

12 Lead EKG

CKD recommended BP

130/80

What is the target glucose level in a critically ill diabetic?

140-180; lower than that does not show improved mortality and has increased risk of complication

Children with normal immune status should receive how many doses of varicella vaccine?

2

A 4-month-old febrile infant presents with loss of appetite, irritability, seizures, focal sensory and motor deficits, and an acute petechial rash. On physical examination, a bulging fontanelle is noted; rectal temperature is 102.8°F. What study would be most important in this child's evaluation?

CSF analysis

How do you diagnose a Pleural effusion?

CXR AP and lateral with lateral decubiti to see if the fluid moves. US guided diagnostic thoracentesis is the gold standard

How do you diagnose pulmonary hypertension?

CXR/CT, PFT's, Echo, Right heart catheterization, labs/biopsy

After adjusting synthroid dose,due to asymptomatic abnormal TSH value, how soon should you recheck?

2-3 months

How many shots are in the tetanus series?

3-4 initial doses followed by boosters every 10 years

Treatment of nephrotic syndrome

ACE inhibitors, judicious use of diuretics

Who is most susceptible

AIDS patient with CD4 count less than 50

What is the post exposure immunization schedule for rabies?

5 injections of 1 mL IM on days 0,3,7,14, and 28.

What is the maximum starting dose of warfarin in the elderly?

5 mg

When does SABA action begin? How long does it last?

5 minutes; 4-6 hours

Size to Repair AAA

5.5 cm

Threshold for platelet transfusion in patients undergoing invasive procedure?

50,000

What age group is most at risk for child abuse?

75% of fatal child abuse involves children aged 3 years and younger.

How does sarcoidosis present?

90% with pulmonary symptoms DOE, dry nonproductive. Non-pulmonary: Skin:Erythema nodosum, lupus pernia, polyserositis; heart: conduction abnormalities; eye: uveitis; parotid gland: enlargement

How do you treat a pneumothorax?

<15% rest, cough/chest pain relief, O2 (highflow) >15% chest tube plus criteria for under 15%, possible surgery.

A 48-year-old white female comes to see you because of abnormal vaginal bleeding. Her periods are lasting 3-5 days longer than usual, bleeding is heavier, and she has experienced some intermenstrual bleeding. Her physical examination is unremarkable, except for a parous cervix with dark blood at the os and in the vagina. She has no orthostatic hypotension, and her hemoglobin level is 11.5 g/dL. A pregnancy test is negative. Which one of the following is the most important next step in management? (check one) A. Laboratory tests to rule out thyroid dysfunction B. An endometrial biopsy C. Oral contraceptives, 4 times a day for 5-7 days D. Cyclic combination therapy with conjugated estrogens (Premarin) and medroxy-progesterone (Provera) each month E. Administration of a gonadotropin-releasing hormone analog such as leuprolide acetate (Eligard Lupron Depot)

A patient over the age of 35 who experiences abnormal vaginal bleeding must have an endometrial assessment to exclude endometrial hyperplasia or cancer. An endometrial biopsy is currently the preferred method for identifying endometrial disease. A laboratory evaluation for thyroid dysfunction or hemorrhagic diathesis is appropriate if no cancer is present on an endometrial biopsy and medical therapy fails to halt the bleeding. The other options listed can be used as medical therapy to control the bleeding once the histopathologic diagnosis has been made.

Narcissistic Personality Disorder is centrally characterized by what feature?

A pattern of grandiosity

A 15-year-old white male is being evaluated after a fall down one flight of stairs. He was transported by the local rescue squad with his cervical spine immobilized. He walked briefly at the scene and did not lose consciousness. His only complaint is a mild, generalized headache. One episode of vomiting occurred shortly after the accident. No weakness or numbness has been noted. Vital signs, mental status, and neurologic findings are normal. Radiologic evaluation of the cervical spine is remarkable only for an air-fluid level in the sphenoid sinus. Which one of the following abnormalities is most likely to be associated with this radiologic finding? (check one) A. A basilar skull fracture B. An orbital floor fracture C. An epidural hematoma D. A zygomatic arch fracture E. A mandible fracture

A post-traumatic air-fluid level in the sphenoid sinus is associated with basilar skull fractures. This finding is frequently noted on cervical spine films. Orbital floor fractures may be associated with double vision, fluid in the maxillary sinus, an air-fluid level in the maxillary sinus, and diplopia. Epidural hematomas are more frequently associated with skull fractures in the area of the meningeal artery. Zygomatic arch fractures are more visible on Towne's view. Characteristic swelling and lateral orbital bruising are typically present. Mandible fractures may be associated with dental misalignment or bleeding.

Which one of the following is a major advantage of second-generation (atypical) antipsychotics compared with first-generation antipsychotics? (check one) A. Less tardive dyskinesia B. Less monitoring for major side effects C. The availability of depot (intramuscular) formulations D. Lower cost E. Simpler dosing schedules

A recent expert consensus panel endorsed the use of second-generation antipsychotics rather than first-generation drugs. Tardive dyskinesia is much less common with the use of second-generation antipsychotics. Several of the second-generation drugs require monitoring for major side effects, however. For example, clozapine, shown by studies to be the most efficacious of the new class, causes granulocytopenia or agranulocytosis, requiring weekly and later biweekly monitoring of blood counts. Both classes have depot formulations for intramuscular administration every 2-4 weeks. Oral dosing of drugs from both classes varies from 1 to 3 times daily. First-generation antipsychotics cost less than second-generation drugs.

Correlation between A1c and mean glucose

A rough guide for estimating average plasma glucose levels assumes that an 1cof 6.0% equals an average glucose level of 120 mg/dL. Each percentage point increase in 1c is equivalent to a 30-mg/dL rise in average glucose. An HbA1cof 7.0% is therefore roughly equivalent to an average glucose level of 150 mg/dL, and an HbA1c of 8.0% translates to an average glucose level of 180 mg/dL.

A 59-year-old male reports decreases in sexual desire and spontaneous erections, as well as reduced beard growth. The most appropriate test to screen for late-onset male hypogonadism is:

A serum total testosterone level is recommended as the initial screening test for late-onset male hypogonadism. Due to its high cost, a free testosterone level is recommended only if the total testosterone level is borderline and abnormalities in sex hormone-binding globulin are suspected. Follow-up LH and FSH levels help to distinguish primary from secondary hypogonadism.

Treatment of Alzheimer's Disease

A trial of acetylcholinesterase inhibitors (tacrine, donepezil, galantamine, or rivastigmine) to help with memory. Memantine is thought ot help with severe Alzheimer's

Aortic dissection presents with what on X-Ray

A widened mediastinum.

Treatment for slowing the progression of Chronic Kidney Disease

ACE inhibitors and ARBs slow the progression of renal dysfunction, particularly in proteinuric patients. Managing comorbid contions (bp>130/80) HBA1C <7.5% LDL <100mg/dL

Aortic Dissection Tx: Ascending vs. Descending

ASCENDING - Surgery DESCENDING - Beta Blocker

What type of agents need to be avoided in WPW syndrome

AV nodal blockers can cause Vfib

Treatment of perioral dermatitis

AVOID topical steroids as they will aggravate the lesions. Use topical metroidazole or erythromycin or oral minocycline, doxycycline, or tetracycline

A 28-year-old white female consults you with a complaint of irregular heavy menstrual periods. A general physical examination, pelvic examination, and Papanicolaou test are normal and a pregnancy test is negative. A CBC and chemistry profile are also normal. The next step in her workup should be: (check one) A. endometrial aspiration B. dilatation and curettage C. LH and FSH assays D. administration of estrogen E. cyclic administration of progesterone for 3 months

Abnormal uterine bleeding is a relatively common disorder that may be due to functional disorders of the hypothalamus, pituitary, or ovary, as well as uterine lesions. However, the patient who is younger than 30 years of age will rarely be found to have a structural uterine defect. Once pregnancy, hematologic disease, and renal impairment are excluded, administration of intramuscular or oral progesterone will usually produce definitive flow and control the bleeding. No further evaluation should be necessary unless the bleeding recurs. Endometrial aspiration, dilatation and curettage, and other diagnostic procedures are appropriate for recurrent problems or for older women. Estrogen would only increase the problem, which is usually due to anovulation with prolonged estrogen secretion, producing a hypertrophic endometrium.

A mother brings her 10-year-old daughter to the pediatrician's office. Over the past few weeks, the girl's teacher has noticed the child staring off into the distance and pausing in the middle of a sentence. Yesterday, the child started twitching at her desk, but it only lasted a few seconds. The child does not seem to remember any of these claims. The mother states that her pregnancy with the child was without complications, and you note that the child has achieved all growth milestones uneventfully. The child is not currently on any medications. The mother has not noticed any of these behaviors at home. She adds that she and her husband are separating. What is the most likely cause of this child's behavior?

Absence seizures

A 28-year-old male presents with the recent onset of intermittent urethral discharge accompanied by dysuria. He is heterosexual, has no prior history of a sexually transmitted infection, and acquired a new sexual partner a month ago. He has no regional lymphadenopathy or ulcers, and gentle milking of the urethra produces no discharge. Evaluation of a first-void urine specimen, however, reveals 15 WBCs/hpf. You treat him with oral azithromycin (Zithromax), 1 g in a single dose, and ceftriaxone (Rocephin), 125 mg intramuscularly. Test results for gonorrhea, Chlamydia, syphilis, HIV, and hepatitis B are negative. He returns 2 months later because his urethral discharge has persisted. He reports no relationships with a different sexual partner, and is confident that his current partner has only had sexual contact with him. You repeat the previous tests and again treat him with oral azithromycin. According to CDC testing and treatment guidelines, which one of the following drugs should be added to his treatment regimen? (check one) A. Metronidazole (Flagyl) B. Amoxicillin/clavulanate (Augmentin) C. Ciprofloxacin (Cipro) D. Trimethoprim/sulfamethoxazole (Bactrim, Septra) E. Cefixime (Suprax)

According to CDC guidelines, the initial workup for urethritis in men includes gonorrhea and Chlamydia testing of the penile discharge or urine, urinalysis with microscopy if no discharge is present, VDRL or RPR testing for syphilis, and HIV and hepatitis B testing. Empiric treatment for men with a purulent urethral discharge or a positive urine test (positive leukocyte esterase or ≥10 WBCs/hpf in the first-void urine sediment) includes azithromycin, 1 g orally as a single dose, OR doxycycline, 100 mg orally twice a day for 7 days, PLUS ceftriaxone, 125 mg intramuscularly, OR cefixime, 400 mg orally as a single dose. If the patient presents with the same complaint within 3 months, and does not have a new sexual partner, the tests obtained at his first visit should be repeated, and consideration should be given to obtaining cultures for Mycoplasma or Ureaplasma and Trichomonas from the urethra or urine. Treatment should include azithromycin, 500 mg orally once daily for 5 days, or doxycycline, 100 mg orally twice daily for 7 days, plus metronidazole, 2 g orally as a single dose.

Which one of the following is considered a contraindication to the use of beta-blockers for congestive heart failure? (check one) A. Mild asthma B. Symptomatic heart block C. New York Heart Association (NYHA) Class III heart failure D. NYHA Class I heart failure in a patient with a history of a previous myocardial infarction E. An ejection fraction <30%

According to several randomized, controlled trials, mortality rates are improved in patients with heart failure who receive beta-blockers in addition to diuretics, ACE inhibitors, and occasionally, digoxin. Contraindications to beta-blocker use include hemodynamic instability, heart block, bradycardia, and severe asthma. Beta-blockers may be tried in patients with mild asthma or COPD as long as they are monitored for potential exacerbations. Beta-blocker use has been shown to be effective in patients with NYHA Class II or III heart failure. There is no absolute threshold ejection fraction. Beta-blockers have also been shown to decrease mortality in patients with a previous history of myocardial infarction, regardless of their NYHA classification

An 8-year-old male presents with cervical lymphadenitis. He has a kitten at home and you are concerned about cat-scratch disease. Which one of the following antibiotics is most appropriate for treatment of Bartonella henselae infection? (check one) A. Azithromycin (Zithromax) B. Ceftriaxone (Rocephin) C. Amoxicillin/clavulanate (Augmentin) D. Doxycycline E. Clindamycin (Cleocin)

Azithromycin has been shown to reduce the duration of lymphadenopathy in cat-scratch disease (SOR B). Other antibiotics that have been used include rifampin, ciprofloxacin, trimethoprim/sulfamethoxazole, and gentamicin. Ceftriaxone, amoxicillin/clavulanate, doxycycline, and clindamycin are not effective in the treatment of Bartonella infection.

A 70-year-old male with a history of hypertension and type 2 diabetes mellitus presents with a 2-month history of increasing paroxysmal nocturnal dyspnea and shortness of breath with minimal exertion. An echocardiogram shows an ejection fraction of 25%. Which one of the patients current medications should be discontinued? (check one) A. Lisinopril (Zestril) B. Pioglitazone (Actos) C. Glipizide (Glucotrol) D. Metoprolol (Toprol-XL) E. Repaglinide (Prandin)

According to the American Diabetes Association guidelines, thiazolidinediones (TZDs) are associated with fluid retention, and their use can be complicated by the development of heart failure. Caution is necessary when prescribing TZDs in patients with known heart failure or other heart diseases, those with preexisting edema, and those on concurrent insulin therapy (SOR C). Older patients can be treated with the same drug regimens as younger patients, but special care is required when prescribing and monitoring drug therapy. Metformin is often contraindicated because of renal insufficiency or heart failure. Sulfonylureas and other insulin secretagogues can cause hypoglycemia. Insulin can also cause hypoglycemia, and injecting it requires good visual and motor skills and cognitive ability on the part of the patient or a caregiver. TZDs should not be used in patients with New York Heart Association class III or IV heart failure.

A 74-year-old African-American female has moderately severe pain due to osteoarthritis. However, she is also on medication for a seizure disorder. When choosing medications to manage her chronic pain, which one of the following should be used with caution because of her history of seizures? (check one) A. Salsalate (Disalcid) B. Celecoxib (Celebrex) C. Hydrocodone (Lortab) D. Oxycodone (OxyContin) E. Tramadol (Ultram)

According to the American Geriatrics Society 2002 clinical practice guidelines for management of persistent pain in older persons, tramadol has efficacy and safety similar to those of equianalgesic doses of codeine and hydrocodone. However, because of the threat of seizures (rare but potential), tramadol should be used with caution in patients with a history of seizure disorder or those taking other medications that lower seizure thresholds.

Which one of the following should be avoided when treating pain in the elderly? (check one) A. Fentanyl (Sublimaze) B. Hydrocodone C. Meperidine (Demerol) D. Morphine E. Oxycodone (OxyContin)

According to the Beers criteria, a list of drugs that should generally be avoided in older adults, meperidine should not be used in the elderly because its metabolite can accumulate and cause seizures. The other medications are not listed in the Beers criteria and are not contraindicated in the elderly

A 52-year-old white male is being considered for pharmacologic treatment of hyperlipidemia because of an LDL cholesterol level of 180 mg/dL. Before beginning medication for his hyperlipidemia, he should be screened for: (check one) A. Hyperthyroidism B. Hypothyroidism C. Addison's disease D. Cushing's disease E. Pernicious anemia

According to the Summary of the National Cholesterol Education Program (NCEP) Adult Treatment Panel III Report of 2001, any person with elevated LDL cholesterol or any other form of hyperlipidemia should undergo clinical or laboratory assessment to rule out secondary dyslipidemia before initiation of lipid-lowering therapy. Causes of secondary dyslipidemia include diabetes mellitus, hypothyroidism, obstructive liver disease, chronic renal failure, and some medications.

A 3-year-old boy was visiting his grandparents when he found a bottle of phenobarbital belonging to his grandfather; the bottle had just been refilled with 50 tablets. The boy was found with 1 tablet in his mouth and 3 more missing. He was immediately rushed to the emergency department within 30 minutes. He appears frightened, but otherwise well. Which of the following gastrointestinal decontamination modalities would be most appropriate for this patient? Syrup of ipecac Gastric lavage Activated charcoal Cathartics Whole bowel irrigation

Activated charcoal Activated charcoal (AC) in a multiple dose regimen should only be considered if a patient has ingested a life-threatening amount of phenobarbital, carbamazepine, dapsone, quinine, or theophylline.

What are the symptoms of and acute exacerbation of COPD

Acute changes in patients baseline SOB, cough, and sputum production

A 72-year-old male with COPD presents to the emergency department with an acute exacerbation marked by increased sputum production and shortness of breath. His oxygen saturation is 88% on room air and he has diffuse inspiratory and expiratory wheezes bilaterally. In addition to oxygen and bronchodilators, which one of the following is most appropriate for this patient? (check one) A. No additional treatments B. Systemic corticosteroids only C. Inhaled corticosteroids only D. Systemic corticosteroids and antibiotics E. Inhaled corticosteroids and antibiotics

Acute exacerbations of COPD are very common, with most caused by superimposed infections. Supplemental oxygen, antibiotics, and bronchodilators are used for management. Systemic corticosteroids, either oral or parenteral, have been shown to significantly reduce treatment failures and improve lung function and dyspnea over the first 72 hours, although there is an increased risk of adverse drug reactions.

What kind of kidney damage is caused by antibiotics?

Acute interstitial nephritis

A 5-year-old Hispanic boy with acute lymphoblastic leukemia presents with fever and rash. He is currently on induction chemotherapy that includes dexamethasone. He was exposed to a friend with varicella 2 weeks ago; he has never had the varicella vaccination and his mother cannot recall that he ever had a varicella infection. Blood tests prior to initiation of chemotherapy showed the absence of antibody to the varicella virus. He now has 10 small vesicles and several red macules on his face and chest. Temperature is 38.5° C. Exam is otherwise normal. What is the most appropriate immediate treatment for this patient? Observation Varicella-zoster immune globulin 125 units/10 kg IM Varicella vaccine 0.5 mL IM Acyclovir 500 mg/m2 IV every 8 hours Acyclovir 200 mg/m2 PO 5 times a day

Acyclovir 500 mg/m2 IV every 8 hours High doses of IV acyclovir (500 mg/m2 IV every 8 hours) should be administered to immunocompromised patients with primary varicella infections due to the high risk of disseminated disease and complications from infection. To prevent primary varicella infection in susceptible patients who have been exposed to the virus, varicella-zoster immune globulin (VZIG) needs to be administered within 96 hours of exposure and sooner, if possible, for maximum effectiveness. Varicella vaccination should not be administered to persons receiving high dose systemic steroids, and it should not be given concurrently with VZIG.

Treatment of Herpes simplex virus

Acyclovir, valacyclovir. Keratitis is treated with trifluridine.

Treatment of severe hypomagnesemia

Administer magnesium sulfate solution (1-2 g) IV followed by an infusion of 6g of magnesium sulfate over 1 L of fluids for 24 hours

A 30-year-old mother with full term pregnancy developed chickenpox 3 days before delivery. She delivered a term female infant weighing 3.2 kg with a good cry. Systemic examination is normal, and neonatal reflexes are intact. What is the appropriate step for prevention of varicella infection in this infant? Isolate the infant from the mother Observe the infant for development of chickenpox Start intravenous acyclovir Administer varicella vaccine Administer varicella zoster immune globulin (Vari ZIG)

Administer varicella zoster immune globulin (Vari ZIG) Infants born to mothers who have contracted chickenpox 5 days before or 2 days after delivery are at risk of severe varicella infection. Maternal viremia occurs about 48 hours prior to the onset of rash. The rash in the infant usually appears by the end of first week or the early part of second week of life. As the mother has not developed a significant antibody response, the infant receives a large dose of virus without maternal varicella-zoster virus (VZV) specific antibodies. These infants should receive 1 vial of varicella zoster immune globulin (Vari ZIG) as soon as possible after birth, ideally within 3 days and a maximum of up to 10 days. Each vial contains 250 mg solution for intramuscular injection. Although neonatal varicella may occur in 50% of these neonates despite receiving vari ZIG, it is usually mild. Perinatally acquired varicella can be life-threatening, so the infant should be treated with acyclovir 10 mg/kg every 8 hours intravenously, but only when the lesions develop. Treatment is continued for 10 days. Infants with neonatal varicella who receive prompt antiviral therapy have an excellent prognosis.

A 30-year-old primipara mother develops chicken pox near the time of delivery; 4 days later she delivers a healthy infant. What is the best line of action to protect the infant from varicella infection? Isolate the infant Administer varicella zoster immunoglobulin Administer oral acyclovir Administer varicella zoster immunoglobulon and oral acyclovir soon after birth Administer varicella zoster immunoglobulin soon after birth and start IV acyclovir when the infant develops the varicella lesions

Administer varicella zoster immunoglobulin soon after birth and start IV acyclovir when the infant develops the varicella lesions All newborns whose mothers develop varicella 5 days before to 2 days after delivery should receive 1 vial (125 units) of VariZIG intramuscularly. Since perinatally-acquired varicella can be a life-threatening condition, the infant should be treated with acyclovir 10 mg/kg/8 hours intravenously when the lesions develop. Infants who develop severe community-acquired varicella infection, especially those who develop complications like pneumonia, encephalitis, or hepatitis, should also receive intravenous acyclovir.

Rare complication of using thioureas

Aganulocytosis

You are writing a prescription for amoxicillin for a 6-year-old female with acute otitis media. Her mother has had an anaphylactic reaction to penicillin in the past and is concerned that she may have passed this trait down to her daughter. You reassure her that this is not usually the case but warn her about potential signs of an allergic reaction. Which one of the following is the most concerning early symptom of a dangerous drug reaction? A) Tachycardia and elevated blood pressure B) Small, bright, erythematous macules diffusely over the trunk C) Pruritus around the mouth and on the palms of the hands and soles of the feet D) Eczematous patches in the antecubital and popliteal fossae E) Diarrhea with blood on the tissue paper (check one) A. Tachycardia and elevated blood pressure B. Small, bright, erythematous macules diffusely over the trunk C. Pruritus around the mouth and on the palms of the hands and soles of the feet D. Eczematous patches in the antecubital and popliteal fossae E) Diarrhea with blood on the tissue paper

Allergic reactions to medications have four primary mechanisms, referred to as Gell and Coombs classifications. The most frequent forms are type I reactions, which are immediate and mediated through IgE, and type IV reactions, which are delayed and mediated through T-cell hypersensitization. Severe type I reactions are often referred to as anaphylaxis and are the most likely to be life threatening with very little warning. Recognition of the early signs of anaphylaxis is the first step in preventing such catastrophes. Anaphylactic reactions result from a massive release of histamine and start with pruritus around the mouth, on the scalp, and on the palms and soles; flushing of the face and neck, with rhinitis and conjunctivitis; angioedema of the oral mucosa, especially of the pharynx and larynx; severe urticaria; dyspnea and bronchospasm (especially in known asthmatics); and hypotension. A delay in lifesaving therapy during this phase will result in full shock, hypotension, and death. Type IV reactions usually result in benign, diffuse erythematous macules on the trunk and proximal extremities, often referred to as a drug rash. These reactions infrequently become more severe and rarely are life threatening. In severe cases the lesions become painful and palpable, and may involve blistering, mucositis, and ecchymosis.

Treatment of BPH

Alpha adrenergic agonists and 5 alpha-reductase inhibitors Balloon dilation, microwave irradiation and stent placement

Which DM II medication delays absorption of carbs in the intestines sometimes producing GI symptoms?

Alpha-Glucosidase inhibitors (acarbose, miglitol)

Which one of the following benzodiazepines has the shortest half-life? (check one) A. Flurazepam (Dalmane) B. Alprazolam (Xanax) C. Clorazepate (Tranxene) D. Diazepam (Valium) E. Clonazepam (Klonopin)

Alprazolam (Xanax) has a half-life of about 12 hours, versus 25 hours for clonazepam and 50 hours for flurazepam, clorazepate, and diazepam.

A 60-year-old male is recovering from a non-Q-wave myocardial infarction. He has a 40-pack-year smoking history, currently smokes a pack of cigarettes per day, and has a strong family history of coronary artery disease. Studies ordered by the cardiologist showed no indication for any coronary artery procedures. His BMI is 27.5 kg/m 2 and his blood pressure is 130/70 mm Hg. Laboratory tests reveal a fasting blood glucose level of 85 mg/dL, a total cholesterol level of 195 mg/dL, and an LDL-cholesterol level of 95 mg/dL. Which one of the following secondary prevention measures would be LEAST likely to improve this patient's cardiovascular outcome? (check one) A. A weight reduction diet B. A β-blocker C. A statin D. An antiplatelet agent E. Smoking cessation

Although dietary management may be appropriate, a weight reduction diet is not likely to improve this patient's cardiovascular outcome. In fact, even if this person were obese, there is insufficient evidence that weight reduction would decrease his cardiovascular mortality (SOR C). There is good evidence that the other options, even β-blockers in a patient with normal blood pressure, are indicated. All of these measures have evidence to support their usefulness for secondary prevention of coronary artery disease (SOR A).

A 62-year-old male has been taking omeprazole (Prilosec) for over a year for gastroesophageal reflux disease. He is asymptomatic and has had no problems tolerating the drug, but asks you about potential side effects, as well as the benefits of continuing therapy. It would be most accurate to tell him that omeprazole therapy is associated with which one of the following? (check one) A. A decreased rate of hip fracture B. Decreased vitamin B12 absorption C. A reduced likelihood of pneumonia D. A reduced likelihood of Clostridium difficile colitis E. An increased likelihood of iron deficiency anemia

Although proton pump inhibitors are the most effective treatment for patients with asymptomatic gastroesophageal reflux disease, there are several potential problems with prolonged therapy. Omeprazole is associated with an increased risk of community-acquired pneumonia and Clostridium difficile colitis. Omeprazole has also been shown to acutely decrease the absorption of vitamin B 12 , and it decreases calcium absorption, leading to an increased risk of hip fracture. The risk for Clostridium difficile colitis is also increased.

A 54-year-old female takes levothyroxine (Synthroid), 0.125 mg/day, for central hypothyroidism secondary to a pituitary adenoma. The nurse practitioner in your office orders a TSH level, which is found to be 0.1 mIU/mL (N 0.5-5.0). Which one of the following would you recommend? (check one) A. Decrease the dosage of levothyroxine B. Increase the dosage of levothyroxine C. Order a free T4 level D. Order a TRH stimulation test E. Repeat the TSH level in 3 months

Although uncommon, pituitary disease can cause secondary hypothyroidism. The characteristic laboratory findings are a low serum free T4 and a low TSH. A free T4 level is needed to evaluate the proper dosage of replacement therapy in secondary hypothyroidism. The TSH level is not useful for determining the adequacy of thyroid replacement in secondary hypothyroidism since the pituitary is malfunctioning. In the initial evaluation of secondary hypothyroidism, a TRH stimulation test would be useful if TSH failed to rise in response to stimulation. It is not necessary in this case, since the diagnosis has already been made.

Which medications can be given to help with the symptoms associated with MS?

Amantadine and pemoline can improve fatigue Baclofen and diazepam improve spasticity

Amiodarone (Cordarone) is most useful for which one of the following? (check one) A. Prophylactic perioperative use for emergency surgery B. Primary prevention of nonischemic cardiomyopathy C. Treatment of atrial flutter D. Treatment of multi-focal premature ventricular contractions following acute myocardial infarction E. Treatment of sustained ventricular tachyarrhythmias in patients with poor hemodynamic stability

Amiodarone is one of the most frequently prescribed antiarrhythmic medications in the U.S. It is useful in the acute management of sustained ventricular tachyarrhythmias, regardless of hemodynamic stability. Amiodarone is appropriate first-line treatment for atrial fibrillation only in symptomatic patients with left ventricular dysfunction and heart failure. It has a very limited role in the treatment of atrial flutter. The only role for prophylactic amiodarone is in the perioperative period of cardiac surgery. The use of prophylactic antiarrhythmic agents in the face of "warning dysrhythmias" or after myocardial infarction is no longer recommended. Prophylactic amiodarone is not indicated for primary prevention in patients with nonischemic cardiomyopathy.

What medications may help control deep, constant, aching pain?

Amitriptyline, nortriptyline, desipramine, gabapentin, or fluphenaine

Otits Media Tx

Amoxicillin is first line Amoxicillin + Clav

A 1-year-old boy is brought in by his dad with a "finger infection." About 10 days ago, the child was bit by a pet rat on his index finger. They did not think anything of it until few days ago, which is when the finger became swollen and the child started to appear unwell. He presents with increased temperatures, decreased appetite, and vomiting. His vital signs are normal, except for temperature, which was 102°F. He was hydrated with IV fluids, and empiric antibiotics were started. Blood cultures were taken, and the causative organism was identified as S. minus. The child is getting moderate oral intake at this time. What is the most appropriate treatment for this infection?

Amoxicillin/clavulanic acid

What helps in differentiating amphetamine from cocaine intoxication?

Amphetamines lasts 4 to 6 hours, and those from cocaine rarely last more than 2 hours.

What antibiotics should be used for a child under four weeks with possible sepsis?

Ampicillin and cefotaxime (claforan); gentamicin and ampicillin

Treatment with which one of the following antihypertensive medications may mimic the effects of primary hyperparathyroidism? (check one) A. Amlodipine (Norvasc) B. Doxazosin (Cardura) C. Hydrochlorothiazide D. Lisinopril (Prinivil, Zestril) E. Metoprolol (Lopressor, Toprol-XL)

An elevated level of parathyroid hormone (or a level that is in an unexpected "normal" range) in a patient with an elevated calcium level generally indicates a diagnosis of primary hyperparathyroidism. However, these laboratory findings may also occur with lithium or thiazide use, tertiary hyperparathyroidism associated with end-stage renal failure, or familial hypocalciuric hypercalcemia, and a medical and family history should be obtained to assess these possibilities. The other medications listed do not cause hypercalcemia.

Which one of the following, when confirmed with a repeat test, meets the diagnostic criteria for diabetes mellitus? (check one) A. A fasting blood glucose level of 120 mg/dL B. A 2-hour value of 180 mg/dL on an oral glucose tolerance test C. A random glucose level of 180 mg/dL in a patient with symptoms of diabetes mellitus D. A positive urine dipstick for glucose E. A hemoglobin A1c of 7.0%

An international expert committee issued a report in 2009 recommending that a hemoglobin A1c level ≥6.5% be used to diagnose diabetes mellitus. Other criteria include a fasting plasma glucose level ≥126 mg/dL, a random glucose leve l≥200 mg/dL in a patient with symptoms of diabetes, or a 2-hour oral glucose tolerance test value ≥200 mg/dL. While a urine dipstick may be used to screen for diabetes, it is not a diagnostic test.

What is a pathological finding for sleep apnea?

An obstructed airway for more than 10 seconds that occurs more than 7-10 times per hour.

A preterm infant develops abdominal distention and bilious vomiting. Meconium has not been passed since birth, which occured 2 days ago. Physical examination finds a distended abdomen. Rectal examination indicates increased sphincter tone and watery stool appears after the removal of examining finger. Plain radiograph shows absence of intra-intestinal air in the pelvis and a barium enema is inconclusive. What is the most appropriate next step in evaluating the likely diagnosis? Full-thickness rectal biopsy Trial with stool softeners Follow-up X-ray Anorectal manometry Repeat barium enema

Anorectal manometry The clinical presentation is strongly suggestive of Hirschsprung's disease. It is a congenital condition which is characterized by the absence of ganglion cells in the submucosal (Meissner's) and myenteric plexus of the distal bowel. The failure of the ganglionic cells to fully migrate caudally during embryonic life is believed to be the cause. In Hirschsprung's disease anorectal manometry shows no recto-sphincteric inhibition reflex on rectal distension. Anorectal manometry is non-invasive and has good accuracy of detection; therefore, it is the diagnosis of choice in infants.

Growth hormone secretion is abnormally high and white blood cell counts are low due to bone marrow hypoplasia. are seen in what disease?

Anorexia

You see a 22-year-old female who sustained a right knee injury in a recent college soccer game.She is a defender and executed a sudden cutting maneuver. With her right foot planted and her ankle locked, she attempted to shift the position of her body to stop an oncoming ball and felt her knee pop. She has had a moderate amount of pain and swelling, which began within 2 hours of the injury, but she is most concerned about the loss of knee hyperextension. Which one of the following tests is most likely to be abnormal in this patient? (check one) A. Anterior drawer B. Lachman C. McMurray D. Pivot shift

Anterior cruciate ligament (ACL) tears occur more commonly in women than in men. The intensity of play is also a factor, with a much greater risk of ACL injuries occurring during games than during practices. The most accurate maneuver for detecting an ACL tear is the Lachman test (sensitivity 60%-100%, mean 84%), followed by the anterior drawer test (sensitivity 9%-93%, mean 62%) and the pivot shift test (sensitivity 27%-95%, mean 62%) (SOR C). McMurray's test is used to detect meniscal tears.

Which one of the following is an appropriate rationale for antibiotic treatment of Bordetella pertussis infections? (check one) A. It delays progression from the catarrhal stage to the paroxysmal stage B. It reduces the severity of symptoms C. It reduces the duration of illness D. It reduces the risk of transmission to others E. It reduces the need for hospitalization

Antibiotic treatment for pertussis is effective for eradicating bacterial infection but not for reducing the duration or severity of the disease. The eradication of infection is important for disease control because it reduces infectivity. Antibiotic treatment is thought to be most effective if started early in the course of the illness, characterized as the catarrhal phase. The paroxysmal stage follows the catarrhal phase. The CDC recommends macrolides for primary treatment of pertussis. The preferred antimicrobial regimen is azithromycin for 3-5 days or clarithromycin for 7 days. These regimens are as effective as longer therapy with erythromycin and have fewer side effects. Children under 1 month of age should be treated with azithromycin. There is an association between erythromycin and hypertrophic pyloric stenosis in young infants. Trimethoprim/sulfamethoxazole can be used in patients who are unable to take macrolides or where macrolide resistance may be an issue, but should not be used in children under the age of 2 months. Fluoroquinolones have been shown to reduce pertussis in vitro but have not been shown to be clinically effective (SOR A).

Which drug class is likely to produce confusion in the elderly?

Anticholinergics

Treatment of Parkinson's Disease

Anticholinergics and levadopa

How do you treatm Venous thromoembolism?

Anticoagulation Heparin or LMWH--> Warfarin for 3-12 months Inferior vena cava filter Thrombolytic therapy Surgery

Which one of the following anticonvulsant medications is preferred for the treatment of mania or hypomania in patients with bipolar disorder? (check one) A. Phenytoin (Dilantin) B. Phenobarbital C. Valproic acid (Depakene) D. Gabapentin (Neurontin) E. Clonazepam (Klonopin)

Anticonvulsant medications are used in the treatment of various psychiatric disorders. Valproic acid is FDA-approved for the treatment of manic episodes associated with bipolar disorder. It has been shown in controlled studies to be significantly more effective than placebo. The initial dosage is 750 mg daily given in divided doses, and most individuals require between 1000 and 2500 mg daily. Carbamazepine has also been used to treat mania and is an alternative for individuals who cannot tolerate lithium or valproic acid. Clonazepam is used in the treatment of panic attacks, and gabapentin is used to treat anxiety. Both phenytoin and gabapentin are also used to treat peripheral neuropathy. The primary use of phenobarbital is as an anticonvulsant. Ref: Valproate and other anticonvulsants for psychiatric disorders.

How should neuropathic pain be managed?

Anticonvulsants (Gabapentin or pregabalin) duloxetine (cymbalta) or lidocaine topical patch. Methdaone can be used but needs to be monitored closely.

Treatment of Atopic dermatitis

Antihistamines, topical corticosteriods. Avoid systemic steroids. Hydration and topical emollients are beneficial. Ultraviolet B light is effective

Which antiviral medications can be used to treat Influenza?

Antiviral medicines that are used to treat or prevent the flu include oseltamivir (Tamiflu®) and zanamivir (Relenza®). Must be given within 1-2 days of onset

A 24-year-old female has a history of mood swings over the past several months, which have created marital and financial problems, in addition to jeopardizing her career as a television news reporter. You have made a diagnosis of bipolar disorder, and she has finally accepted the need for treatment. However, she insists that you choose a drug that "won't make me fat."

Apirazole

A 24-year-old primigravida has nausea and vomiting associated with pregnancy. Which one of the following is recommended by the American Congress of Obstetricians and Gynecologists (ACOG) as first-line therapy? (check one) A. Droperidol (Inapsine) B. Ondansetron (Zofran) C. Prochlorperazine D. Metoclopramide (Reglan) E. Doxylamine (Unisom) and vitamin B6

Approximately 10% of women with nausea and vomiting during pregnancy require medication. Pharmacologic therapies that have been used include vitamin B6 , antihistamines, and prokinetic agents, as well as other medications. Randomized, placebo-controlled trials have shown that vitamin B6 is effective for this problem. The combination of vitamin B 6 and doxylamine was studied in more than 6000 patients and was associated with a 70% reduction in nausea and vomiting, with no evidence of teratogenicity. It is recommended by the American Congress of Obstetricians and Gynecologists as first-line therapy for nausea and vomiting in pregnancy. A combination pill was removed from the U.S. market in 1983 because of unjustified concerns about teratogenicity, but the medications can be bought separately over the counter. In rare cases, metoclopramide has been associated with tardive dyskinesia, and the FDA has issued a black-box warning concerning the use of this drug in general. The 5-HT3 -receptor antagonists, such as ondansetron, are being used for hyperemesis in pregnancy, but information is limited. Droperidol has been used for this problem in the past, but it is now used infrequently because of its risks, particularly heart arrhythmias.

A 6-month-old healthy infant presents with an inability to sleep through the night. He has 3 - 4 nighttime awakenings, and his parents spend a long time getting him to go back to sleep. Both parents work and their sleep continues to remain disrupted, which affects their work performance the next day. What best describes the physiology of sleep in this patient?

Approximately 25% - 50% of 6 - 12-month-olds, and 30% of 1-year-olds, have problematic night waking

While evaluating a stroke patient, you ask him to stick out his tongue. At first he is unable to do this, but a few moments later he performs this movement spontaneously. This defect is known as: (check one) A. apraxia B. agnosia C. expressive (Broca's) aphasia D. astereognosis

Apraxia is a transmission disturbance on the output side, which interferes with skilled movements. Even though the patient understands the request, he is unable to perform the task when asked, but may then perform it after a time delay. Agnosia is the inability to recognize previously familiar sensory input, and is a modality-bound deficit. For example, it results in a loss of ability to recognize objects. Aphasia is a language disorder, and expressive aphasia is a loss of the ability to express language. The ability to recognize objects by palpation in one hand but not the other is called astereognosis.

Treatment of a phimosis

Asymptomatic-Leave it alone, will gradually get larger Symptomatic-Circumcision

A 45-year-old male sees you for a routine annual visit and is found to have atrial fibrillation, with a ventricular rate of 70-75 beats/min. He is otherwise healthy, and a laboratory workup and echocardiogram are normal. Which one of the following would be the most appropriate management? (check one) A. Aspirin, 325 mg daily B. Warfarin (Coumadin), with a target INR of 2.0-3.0 C. Clopidogrel (Plavix), 75 mg daily D. Amiodarone (Cordarone), 200 mg daily E. Observation only

Atrial fibrillation is the most common arrhythmia, and its prevalence increases with age. The major risk with atrial fibrillation is stroke, and a patient's risk can be determined by the CHADS 2 score. CHADS stands for Congestive heart failure, Hypertension, Age >75, Diabetes mellitus, and previous Stroke or transient ischemic attack. Each of these is worth 1 point except for stroke, which is worth 2 points. A patient with 4 or more points is at high risk, and 2-3 points indicates moderate risk. Having ≤1 point indicates low risk, and this patient has 0 points. Low-risk patients should be treated with aspirin, 81-325 mg daily (SOR B). Moderate-or high-risk patients should be treated with warfarin. Amiodarone is used for rate control, and clopidogrel is used for vascular events not related to atrial fibrillatio

A 75-year-old African-American male with no previous history of cardiac problems complains of shortness of breath and a feeling of general weakness. His symptoms have developed over the past 24 hours. On physical examination you find a regular pulse with a rate of 160 beats/min. You note rales to the base of the scapula bilaterally, moderate jugular venous distention, and hepatojugular reflux. His blood pressure is 90/55 mm Hg; when he sits up he becomes weak and diaphoretic and complains of precordial pressure. An EKG reveals atrial flutter with 2:1 block. Management at this time should include: (check one) A. intravenous digoxin B. intravenous verapamil (Calan, Isoptin) C. amiodarone (Cordarone) D. electrical cardioversion E. insertion of a pacemaker

Atrial flutter is not ordinarily a serious arrhythmia, but this patient has heart failure manifested by rales, jugular venous distention, hepatojugular reflux, hypotension, and angina. Electrical cardioversion should be performed immediately. This is generally a very easy rhythm to convert. Digoxin and verapamil are appropriate in hemodynamically stable patients. A pacemaker for rapid atrial pacing may be beneficial if digitalis intoxication is the cause of atrial flutter, but this is unlikely in a patient with no previous history of cardiac problems. Amiodarone is not indicated in this clinical situation.

A 27-year-old white male has been in rehabilitation for C6 complete quadriplegia. His health had been good prior to a diving accident 2 months ago which caused his paralysis. The patient has been catheterized since admission and his recovery has been steady. His vital signs have been normal and stable. The nurse calls and tells you that for the past hour the patient has experienced sweating, rhinorrhea, and a pounding headache. His heart rate is 55/min and his blood pressure is 220/115 mm Hg. His temperature and respirations are reported as normal. There has been no vomiting and his neurologic examination is unchanged. The most likely diagnosis is: (check one) A. Cluster headache B. Autonomic hyperreflexia C. Sepsis D. Intracranial hemorrhage E. Progression of the spinal cord lesion

Autonomic hyperreflexia is characterized by the sudden onset of headache and hypertension in a patient with a lesion above the T6 level. There may be associated bradycardia, sweating, dilated pupils, blurred vision, nasal stuffiness, flushing, or piloerection. It usually occurs several months after the injury and has an incidence as high as 85% in quadriplegic patients. Frequently, it subsides within 3 years of injury, but it can recur at any time. Bowel and bladder distention are common causes. Hypertension is the major concern because of associated seizures and cerebral hemorrhage. Cluster headaches have a constant unilateral orbital localization. The pain is steady (non-throbbing) and lacrimation and rhinorrhea may be part of the syndrome. Sepsis is usually manifested by chills, fever, nausea, and vomiting. Common signs include tachycardia and hypotension rather than bradycardia and hypertension. Signs and symptoms of intracranial hemorrhage vary depending upon the site of the hemorrhage, but the unchanged neurologic status and the lack of a history of hypertension decrease the likelihood of this diagnosis. There are no neurologic findings or history which suggest progression of the patient's lesion at C6

Treatment of erythema multiforme

Avoid precipitating substances and control herpes outbreak with acyclovir

How should opioid-induced nausea be treated?

Avoid rapid infusion of large doses of medication. Compazine, ondansetron or haloperidol as nasuea is mediate in the chemoreceptor trigger zone via dopamine and serotonin systems.

What is the treatment for contact dermatitis?

Avoid the offending agent, wet dressing with Burrow's solution (aluminum acetate in water) and topical corticosteroids

What is the treatment for rosacea?

Avoidance or precipitating factors and using sunscreen; antibiotics (metronidazole, doxy); clonidine or non-selective B-blocker

Treatment of chlamydia (watery discharge)

Azithromycin

Severe Preeclampsia Requirements

BP > 160/110, Proteinuria > 5g/24h, Thrombocytopenia w/ Proteinuria < 100,000, Liver enzyme abnormalities, AMS

Who is the flu vaccine contraindicated in?

Babies under 6 month, people with egg allergies, patients with a hx of Guillain Barre Syndrome.

The most common cause of abnormal vaginal discharge in a sexually active 19-year-old female is (check one) A. Candida albicans B. Trichomonas vaginalis C. Staphylococcus D. Group B Streptococcus E. Bacterial vaginosis

Bacterial vaginosis (BV) is the most common cause of acute vaginitis, accounting for up to 50% of cases in some populations. It is usually caused by a shift in normal vaginal flora.

What are triggers for an acute COPD exacerbation?

Bacterial-H.Flu, S.pneumo Viral-Influenza, parainfluenza, rhinovirus Smoking, CHF, allergens

TB infection infects what part of the lungs?

Bases (first time infected)

A 9-month-old, former low birthweight premature infant born at 29 weeks gestation is being seen for a well child visit. He stayed in the NICU for 3 months after birth where he spent 2 weeks on ventilator support for mild bronchopulmonary dysplasia and antibiotics. He had normal head ultrasound studies and vision exams. He spent the rest of the time feeding and growing until discharge. Initially, he had some intermittent apnea, which ceased a few weeks prior to discharge. He continues to gain weight and is eating and doing relatively well according to his mother, who is a single 18-year-old. She is going to school part time and relies on some school daycare and a sister who also helps with child care. So far he has had 1 cold and ear infection. His mother wonders about his development, as he is noticeably behind his cousins who are crawling and cruising well, though his development has been progressing. She has many questions and concerns, however. What developmental screening test would be best for this patient? Ages and Stages Questionnaire Bayley Infant Neurodevelopmental Screen Denver II screening Parents Evaluations of Developmental Status Child Behavioral Inventory

Bayley Infant Neurodevelopmental Screen The Bayley Infant Neurodevelopmental Screens (BINS) is a developmental screening tool that identifies general developmental delay in high risk preterm, low birth weight, or low socioeconomic status population that is practitioner-administered. The risk for neurodevelopmental morbidities increases with the rising number of high risk, low birth weight premies that survive. There is an increased awareness between the potential long-term alterations of growth and development and certain perinatal interventions, such as oxygen administration and postnatal steroids. These findings have led to the recognition of the need to improve standardization and comparability of methodology and data collection.

A 68-year-old African-American female with primary hypothyroidism is taking levothyroxine (Synthroid), 125 μg/day. Her TSH level is 0.2μU/mL (N 0.5-5.0). She has no symptoms of either hypothyroidism or hyperthyroidism. Which one of the following would be most appropriate at this point? (check one) A. Continuing levothyroxine at the same dosage B. Increasing the levothyroxine dosage C. Decreasing the levothyroxine dosage D. Discontinuing levothyroxine E. Ordering a free T 4

Because of the precise relationship between circulating thyroid hormone and pituitary TSH secretion, measurement of serum TSH is essential in the management of patients receiving levothyroxine therapy. Immunoassays can reliably distinguish between normal and suppressed concentrations of TSH. In a patient receiving levothyroxine, a low TSH level usually indicates overreplacement. If this occurs, the dosage should be reduced slightly and the TSH level repeated in 2-3 months' time. There is no need to discontinue therapy in this situation, and repeating the TSH level in 2 weeks would not be helpful. A free T4 level would also be unnecessary, since it is not as sensitive as a TSH level for detecting mild states of excess thyroid hormone.

A 3-month-old male infant presented for his first well-child visit. He was born at 32 weeks gestation and experienced hyperbilirubinemia in the nursery, requiring phototherapy and exchange transfusion. He has had rhinorrhea without fever for 2 days. Physical examination revealed a normal-appearing infant at the 25th percentile for height and weight. Vital signs were normal. There was mild clear rhinorrhea without respiratory distress. 2 types of immunizations against rotavirus are available: RotaTeq and Rotarix. Should you administer rotavirus vaccine at this point? No, since the child has an upper respiratory infection No, since the child was born prematurely Begin either immunization series now Initiate the RotaTeq series No, since the rotavirus series was not initiated at 2 months of age

Begin either immunization series now Rotavirus is the most common cause of gastroenteritis and diarrhea under 2 years of age, especially between 3 and 15 months of age. 2 types of immunizations are available: Rotarix, which is administered orally at 2 and 4 months, and RotaTeq, which is given orally at 2, 4, and 6 months. Either series can be initiated after 6 weeks and before 15 weeks. Both immunizations are approximately 75% effective in reducing the number of cases of gastroenteritis due to rotavirus, which account for nearly all severe cases and hospitalizations due to rotavirus.

First line modes of therapy for urge urinary incontinence?

Behavioral therapy (pelvic muscular contractions) and anticholinergic therapy

What is melanosis coli

Benign spotting of colonic mucosa resulting from anthraquinone laxative abuse

Treatment for psychomotro agitation in alcohol withdrawal?

Benzodiazepenes

Flumazenil treats what substance overdose?

Benzodiazepine overdose. It is a competitive antagonist of the gamma-aminobutyric acid (GABA) receptor.

Treatment of sleep terror and sleepwalking?

Benzodiazepines

Treatment of hyperthyroidism (thyrotoxicosis) symptoms of tachycardia, tremor, diaphoresis, anxiety, palpitations

Beta blockers, specifically propranolol

Which one of the following therapeutic agents is most appropriate for daily use in the prevention of migraine headache? (check one) A. Dihydroergotamine (D.H.E. 45) B. Amitriptyline (Elavil) C. Sumatriptan (Imitrex) D. Aspirin/caffeine/butalbital (Fiorinal) E. Acetaminophen/hydrocodone bitartrate (Vicodin)

Beta-adrenergic blockers, antidepressants, anticonvulsants, calcium channel blockers, NSAIDs, and serotonin antagonists are the major classes of drugs used for preventive migraine therapy. All of these medications result in about a 50% reduction in the frequency of headaches. The other drugs listed are useful for the treatment of acute migraine, but not for prevention.

Which one of the following has been shown to decrease mortality late after a myocardial infarction? (check one) A. Nitrates B. Beta-blockers C. Digoxin D. Thiazide diuretics E. Calcium channel antagonists

Beta-blockers and ACE inhibitors have been found to decrease mortality late after myocardial infarction. Aspirin has been shown to decrease nonfatal myocardial infarction, nonfatal stroke, and vascular events. Nitrates, digoxin, thiazide diuretics, and calcium channel antagonists have not been found to reduce mortality after myocardial infarction.

When do night terrors tend to occur?

Between 18 months and 6 years of age

Which one of the following classes of diabetes medications acts primarily by stimulating pancreatic insulin secretion? (check one) A. Biguanides, such as metformin (Glucophage) B. Thiazolidinediones, such as pioglitazone (Actos) C. DPP-4 inhibitors, such as sitagliptin (Januvia) D. Sulfonylureas, such as glipizide (Glucotrol) E. Amylin analogs, such as pramlintide (Symlin)

Biguanides and thiazolidinediones are insulin sensitizers that decrease hepatic glucose production and increase insulin sensitivity. Sulfonylureas and meglitinides stimulate pancreatic insulin secretion, while DPP-4 inhibitors prevent GLP-1 breakdown and slow the breakdown of some sugars. GLP-1 mimetics stimulate insulin secretion, suppress glucagon secretion, and promote β-cell production. Amylin analogs act with insulin to delay gastric emptying and they also inhibit glucagon release.

Buzz word for sarcoidosis

Bilateral hilar adenopathy seen on CXR. Biopsy with noncaseating granulomas

Which bacterial infection is associated with a large increase in lymphocytes?

Bordetella pertussis

A 63-year-old white male sees you for an initial visit and is accompanied by his daughter, who is a patient of yours and scheduled the visit. The father recently relocated to be near the daughter after his wife died. He has well-controlled type 2 diabetes mellitus, but is otherwise healthy. Referring to the copy of the medical records they brought with them, the daughter notes that her father has received influenza vaccine in 3 of the past 5 years, but she can find no documentation that he ever had "the pneumonia vaccine." She asks if he should receive it at this visit. You advise them that he should receive pneumococcal vaccine: (check one) A. annually, along with influenza vaccine B. now and a repeat dose every 5 years C. every 5 years starting at age 65 D. now and a repeat dose once at age 68 E. only once, at age 65

Both the CDC and the American Academy of Family Physicians recommend that all adults over the age of 65 receive a single dose of pneumococcal polysaccharide vaccine. Immunization before the age of 65 is recommended for certain subgroups of adults, including institutionalized individuals over the age of 50; those with chronic cardiac or pulmonary disease, diabetes mellitus, anatomic asplenia, chronic liver disease, or kidney failure; and health-care workers. It is recommended that those receiving the vaccine before the age of 65 receive an additional dose at age 65 or 5 years after the first dose, whichever is later.

Treatment of Botulism

Botulism Antitoxin through the CDC. Respiratory failure may warrant ventilation

A 6-month-old white male is brought to your office because he has "blisters" in his diaper area. On examination, you find large bullae filled with cloudy yellow fluid. Some of the blisters have ruptured and the bases are covered with a thin crust. Which one of the following is most appropriate in the management of this condition? (check one) A. Rinsing diapers with a vinegar solution B. A topical antifungal agent C. Penicillin D. Trimethoprim/sulfamethoxazole (Bactrim, Septra)

Bullous impetigo is a localized skin infection characterized by large bullae; it is caused by phage group II Staphylococcus aureus. Cultures of fluid from an intact blister will reveal the causative agent. The lesions are caused by exfolatin, a local toxin produced by the S. aureus, and develop on intact skin. Complications are rare, but cellulitis occurs in <10% of cases. Strains of Staphylococcus associated with impetigo in the U.S. have little or no nephritogenic potential. Systemic therapy should be used in patients with widespread lesions. With the emergence of MRSA, trimethoprim/sulfamethoxazole and clindamycin are options for outpatient therapy. Intravenous vancomycin can be used to treat hospitalized patients with more severe infections.

Which antidepressant is associate with modest weight loss?

Bupropion

A 74-year-old male presents with a 4-day history of diarrhea that he had initially thought was "a 24-hour virus." He states that the onset of his illness included nausea, one episode of vomiting, and profuse diarrhea. He has felt feverish and has been having abdominal cramps. He does not recall eating anything unusual and has not traveled recently. On examination he appears uncomfortable, but in no real distress. His oral temperature is 37.1°C (98.8°F), blood pressure 134/82 mm Hg, and pulse rate 100 beats/min. He has lost 4 kg (9 lb) since his last visit 2 months earlier. His abdomen is soft, with hyperactive bowel sounds and mild diffuse tenderness on palpation. A CBC and basic metabolic profile are normal. Which one of the following is the most likely cause of this patient's illness? (check one) A. Norwalk-like virus (Norovirus) B. Shigella C. Campylobacter D. Escherichia coli O157:H7 E. Staphyloccocus aureus

Campylobacter jejuni is one of the most common causes of bacterial foodborne illnesses, estimated to affect 1 million Americans annually. Undercooked or improperly handled chicken is most often implicated as the source; surveys have demonstrated that between 20% and 100% of all retail chicken sold in the United States is contaminated. The infection is generally isolated and sporadic, occurs more frequently at the extremes of age, is most common during the summer months, and affects males disproportionately. Symptoms typically begin 2-5 days following exposure. Diarrhea is the predominant symptom, with a lesser degree of nausea and vomiting. Up to 10 days is required for full recovery. While Escherichia coli O157:H7 and Shigella may cause a similar illness, both generally present with bloody diarrhea. E. coli O157:H7 is most often transmitted in contaminated undercooked beef, and Shigella is usually spread in a fecal-oral pattern or via contaminated water. The peripheral WBC count is typically increased substantially in shigellosis. Staphylococcus aureus produces an enterotoxin in food that causes the onset of nausea, vomiting, and diarrhea within hours of ingestion and clears within 24-48 hours. Norovirus is a very common cause of acute viral gastroenteritis, usually with more vomiting than diarrhea. It spreads person to person, and patients usually recover within 24 hours.

A 3-year-old girl presents with a 1-day history of irritability and weakness in her legs. Neurologic exam reveals an ascending symmetrical paralysis with cranial neuropathy. A lumbar puncture is performed and cerebrospinal fluid is found to have a normal glucose level, <10 leukocytes/mm3, and elevated protein. Medical history shows the child recently recovered from a mild diarrheal illness. What organism is triggering this syndrome?

C. jejuni

What lab test must be ordered for a patient taking Clozapine

CBC with absolute neutrophil count every week for the first 6 months

Prophylactic treatment of Histoplasmosis in a patient with HIV

CD4 count less than 100 with itraconazole

Prophylactic treatment of PJP in a patient with HIV

CD4 count less than 200 with bactrim

Prophylactic treatment of Cytomegalovirus in a patient with HIV

CD4 count less than 50

Prophylactic treatment of Candida in a patient with HIV

CD4 count less than 500 with fluconazole

A 4-year-old boy presents with erratic movements of his left arm and leg. These movements have been present since birth and occur on the left side. They have not been getting worse. The boy's mother admits that he has been using his right hand for almost all activities since the age of 1. Further questioning reveals that the child exhibited delayed milestones, such as rolling over at 6 months, sitting up at 1 year, and walking at 2 years. He does not speak, and he does not respond to his mother's verbal commands. His physical exam is noteworthy for spasticity, hyperreflexia, ataxia, involuntary movements, and weakness of his left arm and leg. His left arm and left leg are measured to be shorter and are atrophied in comparison to the right. Additionally, the child does not respond to verbal stimuli. What is the most likely diagnosis?

Cerebral palsy

A 3-year-old child presents with a 2-day history of high fever with headache and vomiting. The child is irritable and is not accepting feeds. Temperature is 103oF. Neck stiffness, Kernig's, and Brudzinsky's sign are positive. Cardiac, pulmonary, and abdominal examinations are normal. CSF examination shows cells 2000/mm3 predominantly polymorphs, protein 200mg/dL, and glucose 30mg/dL. Gram stain of CSF smear shows Gram-negative coccobacilli suggestive of Hemophilus influenza type b meningitis. The patient is successfully treated with intravenous ampicillin for 10 days. If required, what chemoprophylaxis would you advise before sending the child home? Chemoprophylaxis with ciprofloxacin Chemoprophylaxis with trimethoprim-sulfamethoxozole Chemoprophylaxis with clindamycin Chemoprophylaxis with rifampin No chemoprophylaxis required

Chemoprophylaxis with rifampin The child should be given chemoprophylaxis with rifampin, as it is effective against nasopharyngeal carriage of H. influenzae b (Hib). For prophylaxis, children should be given rifampin orally (0 - 1 month of age 10mg/Kg/dose, >1month of age 20mg/Kg/dose, not to exceed 600 mg/dose) once a day for consecutive 4 days. Rifampin prophylaxis is not recommended for pregnant women.

Treatment of Malaria

Chloroquine/hydroxychloroquine

Which medication to treat dyslipidemia is associated with reduced incidences of coronary events in middle aged men, but have no effect of total mortality?

Cholestyramine, cholesevelam, and colestipol

How do you treat myastenia gravis?

Cholinesterase inhibitors-pyriodostigmine. Thymectomy may be necessary.

The parents of an 8-year-old boy are worried about his height because he is the shortest child in his class. The boy was born at full term and had a normal birth weight. The neonatal period was uneventful, and he achieved his developmental milestones on time; his elder brother, who also achieved his developmental milestones normally, is of normal height. His anthropometric measurements are as follows: Height: 104 cm (41 in) Weight: 20 kg Head circumference: 53 cm Span 100 cm: Upper segment: Lower segment ratio 1.5: 1 His bone age is appropriate for his chronological age. What is the most likely diagnosis? Familial short stature Constitutional delay in growth Chondrodystrophy Growth hormone deficiency Hypothyroidism

Chondrodystrophy The most likely diagnosis is chondrodystrophy, which is also known as 'achondrosplasia' because of the characteristic features of short height, span less than height, upper segment: lower segment (US:LS) ratio of more than 1, and bone age appropriate for chronological age Normally US:LS ratio is 1.7:1 at birth and decreases by .07 to 0.1 every year; it becomes 1:1 at about 10 years. Normally, the span is equal to height from birth to puberty. Achondroplasia is a common cause of short stature with disproportionately short limbs. It is a genetic disorder of bone growth, and there is an autosomal dominant mode of inheritance.

What is coal workers pneumoconiosis?

Chronic fibrotic lung disease caused by inhalation of coal dust that is ingested by alveolar macrophages

A middle-aged hairdresser presents with a complaint of soreness of the proximal nail folds of several fingers on either hand, which has slowly worsened over the last 6 months. The nails appear thickened and distorted. Otherwise she is healthy and has no evidence of systemic disease. Which one of the following would be the most effective initial treatment? (check one) A. Soaking in a dilute iodine solution twice daily to cleanse and sterilize the nail beds B. Oral amoxicillin/clavulanate (Augmentin) for up to 4-6 weeks C. Topical betamethasone dipropionate (Diprolene) applied twice daily to the nail folds for 3-4 weeks D. Evaluation for HIV, hepatitis C, psoriasis, and rheumatoid arthritis

Chronic paronychia is a common condition in workers whose hands are exposed to chemical irritants or are wet for long periods of time. This patient is an otherwise healthy hairdresser, with frequent exposure to irritants. The patient should be advised to avoid exposure to harsh chemicals and water. In addition, the use of strong topical corticosteroids over several weeks can greatly reduce the inflammation, allowing the nail folds to return to normal and helping the cuticles recover their natural barrier to infection. Soaking in iodine solution would kill bacteria, but would also perpetuate the chronic irritation. Because the condition is related to chemical and water irritation, a prolonged course of antibiotics should not be the first treatment step, and could have serious side effects. There is no need to explore less likely autoimmune causes for nail changes at this time.

A 40-year-old female with chronic plaque psoriasis requests topical treatment. Which one of the following topical therapies would be most effective and have the fewest adverse effects? (check one) A. High-potency corticosteroids B. Tazarotene (Tazorac) C. Coal tar polytherapy D. Anthralin

Chronic plaque psoriasis is the most common type of psoriasis and is characterized by redness, thickness, and scaling. A variety of treatments were found to be more effective than placebo, but the best results were produced by topical vitamin D analogues and topical corticosteroids. Vitamin D and high-potency corticosteroids were equally effective when compared head to head, but the corticosteroids produced fewer local reactions (SOR A).

While rounding in the newborn nursery, you are told about a young woman on the labor and delivery floor who developed chickenpox during her first trimester. She had many skin lesions but otherwise recovered well. She apparently had never been vaccinated and has had poor and sporadic prenatal care. You are asked to attend the delivery. Which manifestation of varicella is most likely to occur in her infant? Skin superinfection Encephalitis Pneumonia Mild varicella Cicatricial skin scars

Cicatricial skin scars Varicella infection that occurs in the first or second trimester of pregnancy will cause approximately 2% of infants to develop congenital varicella syndrome. With current childhood vaccination recommendations this is now rare, however the most prominent manifestation seen is zigzag, cicatricial skin scars reported in >60% of cases. Other manifestations include small infant size, hypoplastic limbs, chorioretinitis, microphthalmos, Horner syndrome, cataracts, nystagmus, cortical atrophy or mental retardation, zoster, and early death. If maternal varicella occurs 5 days before or 2 days after delivery, infants are at increased risk of a severe or fatal varicella infection. Other clinical features may include pneumonia and hepatitis.

What helps with claudication

Cilastozol

What drug helps relieve claudication symptoms?

Cilostazol (Pletal)

Treatment of Antrax

Ciprofloxacin or another fluoroquinolone. Doxycycline is an alternative. Attenuated vaccination is available for a person with a high likelihood of exposure

A 70-year-old male presents to your office for a follow-up visit for hypertension. He was started on lisinopril (Prinivil, Zestril), 20 mg daily, 1 month ago. Laboratory tests from his last visit, including a CBC and a complete metabolic panel, were normal except for a serum creatinine level of 1.5 mg/dL (N 0.6-1.5). A follow-up renal panel obtained yesterday shows a creatinine level of 3.2 mg/dL and a BUN of 34 mg/dL (N 8-25). Which one of the following is the most likely cause of this patient's increased creatinine level? (check one) A. Bilateral renal artery stenosis B. Coarctation of the aorta C. Essential hypertension D. Hyperaldosteronism E. Pheochromocytoma

Classic clinical clues that suggest a diagnosis of renal-artery stenosis include the onset of stage 2 hypertension (blood pressure >160/100 mm Hg) after 50 years of age or in the absence of a family history of hypertension; hypertension associated with renal insufficiency, especially if renal function worsens after the administration of an agent that blocks the renin-angiotensin-aldosterone system; hypertension with repeated hospital admissions for heart failure; and drug-resistant hypertension (defined as blood pressure above the goal despite treatment with three drugs of different classes at optimal doses). The other conditions mentioned do not cause a significant rise in serum creatinine after treatment with an ACE inhibitor.

A healthy 48-year-old bookkeeper who works in a medical office has a positive PPD on routine yearly screening. Which one of the following would be most appropriate at this point? (check one) A. A chest radiograph B. A repeat PPD C. Treatment with isoniazid and one other antituberculous drug for 12 months D. Anergy testing

Clinical evaluation and a chest radiograph are recommended in asymptomatic patients with a positive PPD (SOR C). A two-step PPD is performed on those at high risk whose initial test is negative. Asymptomatic patients with a positive PPD and an abnormal chest film should have a sputum culture for TB, but a culture is not required if the chest film is negative. Persons with a PPD conversion should be encouraged to take INH for 9 months with proper medical supervision. Patients with a negative PPD who are still at high risk for TB, especially HIV-positive patients, could be evaluated for anergy, but it is not recommended at this time.

Which one of the following is most predictive of increased perioperative cardiovascular events associated with noncardiac surgery in the elderly? (check one) A. An age of 80 years B. Left bundle-branch block C. Atrial fibrillation with a rate of 80 beats/min D. A history of previous stroke E. Renal insufficiency (creatinine 2.0 mg/dL)

Clinical predictors of increased perioperative cardiovascular risk for elderly patients include major risk factors such as unstable coronary syndrome (acute or recent myocardial infarction, unstable angina), decompensated congestive heart failure, significant arrhythmia (high-grade AV block, symptomatic ventricular arrhythmia, supraventricular arrhythmias with uncontrolled ventricular rate), and severe valvular disease. Intermediate predictors are mild angina, previous myocardial infarction, compensated congestive heart failure, diabetes mellitus, and renal insufficiency. Minor predictors are advanced age, an abnormal EKG, left ventricular hypertrophy, left bundle-branch block, ST and T-wave abnormalities, rhythm other than sinus, low functional capacity, history of stroke, and uncontrolled hypertension. Ref: Schroeder BM: Updated guidelines for perioperative cardiovascular evaluation for noncardiac surgery

Treatment of nocturnal myoclonus?

Clonazepam

A 75-year-old male develops a mild Clostridium difficile infection and is treated with 10 days of metronidazole (Flagyl), 500 mg orally 3 times daily. The diarrhea recurs 10 days after he completes the course of treatment. Which one of the following would be most appropriate? (check one) A. Repeat the course of metronidazole B. Repeat the course of metronidazole and add vancomycin C. Administer vancomycin intravenously D. Prescribe loperamide (Imodium), 4 mg twice daily as needed E. Prescribe a probiotic

Clostridium difficile infection is more common with aging and can be treated with either metronidazole or vancomycin daily. For mild recurrent disease, repeating the course of the original agent is appropriate (SOR B). Multiple recurrences or severe disease warrants the use of both agents. The effectiveness of probiotics such as Lactobacillus remains uncertain. Intravenous vancomycin has not been effective. Antiperistaltic drugs should be avoided.

Which one of the following antipsychotic medications is most likely to cause agranulocytosis? (check one) A. Clozapine (Clozaril) B. Aripiprazole (Abilify) C. Risperidone (Risperdal) D. Olanzapine (Zyprexa)

Clozapine was the first atypical antipsychotic drug, so designated because it has antipsychotic effects without the adverse effects on movement seen with first-generation agents, in addition to having enhanced therapeutic efficacy compared with first-generation drugs. Because of these advantages, it was introduced into clinical practice in the United States despite a serious known adverse effect: an increased incidence of agranulocytosis. Although only clozapine causes agranulocytosis in a substantial proportion of patients, many second-generation drugs produce clinically significant weight gain.

What is the shape of H.Flu

Coccobacillis, gram negative short fat rods

A 2-month-old female infant was born after an uncomplicated pregnancy, labor, and delivery. The baby was fed artificial formula rather than nursed by her mother. She had an uncomplicated neonatal course and was well until about 1 month of age. Since that time, she has had some vomiting every day and has very loose, wet, runny yellow to green stools. In the last several days, her parents have noted red flecks in her stools. She has had intermittent periods of fussiness. Your work-up would include which of the following?

Colonic endoscopy with biopsy, CBC, stool for leukocytes, peripheral eosinophil count

Treatment of chronic venous insufficiency

Compression stockings, sclerosis of varicose veins, vascular bypass or angioplasty

A 69-year-old female sees you for an annual examination. She asks you to look at her toes, and you note a fungal infection in five toenails. She says the condition is painful and limits her ability to complete her morning walks. She asks for treatment that will allow her to resume her daily walks as soon as possible. Her only other medical problem is allergic rhinitis which is well controlled. (check one) A. Oral griseofulvin ultramicrosize (Gris-PEG) daily for 12 weeks B. Oral terbinafine (Lamisil) daily for 12 weeks C. Topical terbinafine (Lamisil AT) daily for 12 weeks D. Topical ciclopirox (Penlac Nail Lacquer) daily for 12 weeks E. Toenail removal

Continuous therapy with oral terbinafine for 12 weeks has the highest cure rate and best long-term resolution rate of the therapies listed. Other agents and pulsed dosing regimens have lower cure rates. Topical creams are not appropriate for onychomycosis because the infection resides in the cell of the toenail. Antifungal nail lacquers have a lower cure rate than systemic therapy and should be used only when oral agents would not be safe. Toenail removal is reserved for patients with an isolated infected nail or in cases involving a dermatophytoma.

Treatment of vascular dementia

Control hypertension and metabolic disorders

You see a 32-year-old white female for her first visit. She presents with numerous complaints which do not conform to patterns seen in organic disease. She states that she has seen several physicians and describes a changing set of symptoms. Although she appears to be well, she claims to have been "sickly" for years. From her affect, you suspect that she is depressed. The most likely diagnosis is: (check one) A. Conversion reaction B. Chronic somatization disorder C. Schizophrenia with multiple somatic delusions D. Histrionic personality E. Primary hypochondriasis

Conversion disorder usually involves a single symptom which is neurologic or pain-related. Symptoms of chronic somatization differ from psychoses in that the symptoms of the psychotic patient are bizarre and more vivid, persist over time, are unaltered by reasoned argument, and are not congruent with the patient's social or cultural background. The delusional nature of psychotic somatic symptoms usually unfolds as the patient talks. The essential feature of the histrionic (hysterical) personality is a pervasive pattern of excessive emotionality and attention seeking. People with this disorder constantly seek to be the center of attention. Emotions are often expressed with inappropriate exaggeration. People with this disorder tend to be very self-centered and have little tolerance for delayed gratification. These people are typically attractive and seductive, often to the point of looking flamboyant and acting inappropriately. Features of primary hypochondriasis include the patient's fixed conviction that he or she is ill, the interpretation of all somatic changes as confirmation of this, and a relentless pursuit of medical assistance despite persistent dissatisfaction with the results. The patient's symptoms remain consistent for years. Physicians frequently feel overwhelmed when initially presented with a patient with somatization disorder. This disorder begins before age 30 and is rarely seen in males. The patient complains of multiple symptoms which involve many organ systems and do not readily conform to patterns seen in organic diseases. The patient skips back and forth from symptom to symptom during the interview. Anxiety and depressed mood are frequent in this disorder, and suicide attempts are common

Glipizide vs. Glyburide: which is safe in chronic kidney disease?

Glipizide. Glyburide has a renally excreted toxic metabolite.

Treatment of tetanus

IMmune globin given IM. A full course of tetanus toxoid should be given once the patient fully reconvers. Penicillin is given to eradicate toxin-producing organisms.

Hemorrhoid Tx: Internal vs. External

INTERNAL - Rubber band ligation EXTERNAL, NON-THROMBOSED - Laxatives, Sitz Bath EXTERNAL, THROMBOSED - Incision and removal

Hypertensive Options in Pregnancy

IV Hydralazine, Labetalol, Oral Nifedipine

How do you treat ophthalmopathy associated with hyperthyroidism?

IV methylprednisolone or high-dose tapered prednisone treatment in non-smokers.

Treatment of a brain abscess?

IV penicillin plus either chloramphenico, metronidazole, or both. Nafcillin if staph is suspected.

Treatment of hypokalemia in the emergent setting or serum potassium <2.5 mEq/L or arrhythmias

IV potassium chloride

What are the classifications of acquired pneumothorax?

Iatrogenic Traumatic Barotrauma

Asbestosis is associated with what BUZZ word?

Plueral plaque. Asbestosis is nodular interstitial fibrosis that leads to progressive dyspnea. Associated with mesothelioma.

Treatment of pyelonephritis

Quinolones or Bactrim as outpatient IV fluoroquinolone or ampicillin and gentamicin for hospital admission patients.

A 3-year-old boy presents with difficulty in breathing and a cough that sounds like a seal. On examination, the child has fever, a harsh barking cough, a respiratory rate of 38/minute, and minimal stridor on agitation. On lung auscultation, there are no rales or wheezing. On cardiac auscultation, there is tachycardia. Radiological examination reveals the so-called 'steeple sign'. What is the most likely diagnosis?

Croup

Permanent control of hyperthyroidism (thyrotoxicosis)

Radioactive iodine unless PREGNANT.

How do you diagnose Influenza?

Raid test NP swab

Treatment of acute MS exacerbation's

Corticosteroids

How do you treat a patient with refractory myastenia gravis?

Corticosteroids, immunosuppressive agents, IVimmunoglobulin, and plasmapheresis

How does Mycoplasma Pneumniae present

Cough, BULLOUS MYRINGITIS, walking pneumonia-kids and young adults

What are common symptoms associated with bronchitis?

Cough, sputum production, concurrent URI. NEVER really FEVER!

Treatment of cholera

Fluid/electrolyte replacement, ABX therapy including tetracycline, ampicillin, bactrim, and fluoroquinolones

A 68-year-old white male with diabetes mellitus is hospitalized after suffering a right middle cerebral artery stroke. A nurse in the intensive-care unit calls to advise you that his blood pressure is 200/110 mm Hg. You should: (check one) A. continue monitoring the patient B. administer labetalol (Trandate) C. administer nicardipine (Cardene) D. administer nitroprusside (Nitropress) E. administer nitroglycerin

Current American Heart Association guidelines for blood pressure control in stroke patients advise monitoring with no additional treatment for patients with a systolic blood pressure <220 mm Hg or a diastolic blood pressure <120 mm Hg. The elevated blood pressure is thought to be a protective mechanism that increases cerebral perfusion, and lowering the blood pressure may increase morbidity.

A 2-year-old boy presents with failure to gain weight. He is the same weight that he was at his 18-month well check. His mother says that she offers him 3 meals per day and 3 snacks, but he usually just picks at them. He does not seem hungry. He does enjoy chicken nuggets, goldfish crackers, and cookies. She gives him a cup of juice each day and around 40 ounces of whole milk. He was a full-term infant and has no significant past medical history. He was fed breast milk exclusively until age 1. He was then switched to whole milk and solids were introduced. Other than his failure to gain weight, his physical exam is normal. He is gaining adequately in height and head circumference. He is reaching age-appropriate milestones and appears well, without any sign of systemic disease. What should his mother initially try in order to help him gain weight?

Cut down on milk consumption

Actinic keratosis is a precursoro for...?

Cutaneous squamous cell carcinoma

Xerostomia, dry skin, blurred vision, mydriasis, urinary retention, and delirium. Agitation and myoclonic jerks are also common findings are associated with what? In severe poisonings, hypotension, seizures, respiratory depression, and cardiac dysrhythmias are classic findings. In advanced poisonings, findings include adult respiratory distress syndrome (ARDS), rhabdomyolysis, and disseminated intravascular coagulation (DIC)

Cyclic antidepressant overdose

Which prescription medication is the leading cause of death by intentional overdose?

Cyclic antidepressants

While doing newborn nursery rounds, you are called about a male infant that was born 48 hours ago. His nurse is concerned that he has not passed any meconium yet. The baby was born at 38 weeks gestation, had a normal spontaneous vaginal delivery with Apgars of 7 and 9 at 1 and 5 minutes. There was a history of polyhydramnios. He had fed well, but is now refusing feedings and has had bilious emesis. On exam, he is somewhat irritable and has abdominal distention that has a doughy character that indents on palpation. Rectal exam is normal. Plain abdominal films reveal multiple loops of dilated small bowel. What is the most likely diagnosis in this infant? Hirschsprung's disease Imperforate anus Necrotizing enterocolitis Meconium plug syndrome Cystic fibrosis

Cystic fibrosis Newborns with cystic fibrosis (CF) frequently present with meconium ileus. This occurs when meconium becomes inspissated or thickened and tenacious and obstructs the distal ileum. Approximately 20% of infants with cystic fibrosis present with meconium ileus at birth. Infants may present with bilious vomiting, abdominal distention, and failure to pass meconium. Abdominal palpation reveals loops of bowel with a doughy character that may indent on palpation. Pulmonary manifestations usually develop soon after. Plain films show nonspecific low small bowel obstruction with numerous air-filled loops of bowel. There may be a history of polyhydramnios. A sweat test for cystic fibrosis should be performed and if meconium ileus is suspected, a contrast barium enema performed for clarification of the intestinal obstruction, and a surgical evaluation obtained. Later associated rectal prolapse can occur in 20% of patients with CF that may need surgical correction if manual reduction fails.

A 3-day-old male neonate is seen in the nursery due to failure to pass meconium and 2 episodes of vomiting. Prenatal and perinatal histories are unremarkable. Family history reveals a brother and sister with severe asthma. Examination revealed a moderately distended abdomen without signs of tenderness. A barium enema revealed meconium ileus with distal narrowing and proximal dilatation of the colon. Meconium passed during the procedure, relieving the distention. Rectal manometry and rectal biopsy were normal. Sweat chloride analysis on 2 occasions revealed 70 mmol/L and 74 mmol/L of chloride in adequate amounts of sweat. How do you advise the parents regarding the infants condition and prognosis? Cystic fibrosis has been confirmed, and the older siblings should also have sweat testing performed. Cystic fibrosis is an autosomal dominant condition; the parents should be sweat tested, as they may have a mild form. Cystic fibrosis is usually fatal by age 10 years. Hirschsprung disease is still likely. Mutation analysis must be done to confirm the diagnosis of cystic fibrosis.

Cystic fibrosis has been confirmed, and the older siblings should also have sweat testing performed. Elevated chloride levels on 2 occasions with adequate amounts of sweat confirms the diagnosis of cystic fibrosis. The "severe asthma" in the 2 siblings may well represent cystic fibrosis as well; therefore, they should also undergo sweat testing. Cystic fibrosis may be diagnosed if any of the following from the first group are present: 1) one or more typical phenotypic features, including chronic sinopulmonary disease, characteristic gastrointestinal and nutritional abnormalities, salt loss syndromes, and obstructive azoospermia; 2) a sibling affected with cystic fibrosis; or 3) positive neonatal screening test PLUS the presence of any of the following from the second group: 1) an elevated sweat chloride concentration of greater than 60 meq/L on at least 2 occasions; 2) identification of mutations of the cystic fibrosis transmembrane conductance regulator (CFTR) gene, located on the long arm of chromosome 7, that are known to cause cystic fibrosis; or 3) the demonstration in vivo of characteristic abnormalities of ion transport across the nasal epithelium.

First-line initial treatment for RA?

DMARDs like MTX

What vaccination is available to prevent diphtheria? What ages is it given?

DTap and tetanue and diphtheria toxoid (Td)

How do DVT present differently than PE's?

DVT: Ipsilateral edema/pin/warmth, palpable cord + Holmans sign PE:Unexplained, sudden dyspnea, pleuritic chest pain, cough, tachypnea, tachycardia

Which MS medication decreases the frequency of relapses in mild disease?

Daily subcutaneous injections of glatiramer acetate

How do you treat refractory pemphigus vulgaris

Dapsone, gold, or cyclophosphamide

Treatment of insomnia?

De-emphasis and reassurance, sleep hygiene rules, but avoid medications

Which antibiotic class is associated with tendon rupture?

Fluoroquinolones

Treatment of cystitis

Fluoroquinolones for 3-5 days. Bastrim in resistant strains

Your patient is 4-year-old girl with a 1-day history of eye redness and pain. Slit examination shows corneal ulceration. You also notice many freckles, spidery blood vessels, and dry skin with irregular dark spots, but what worries you the most is shiny, pearly nodule on her left cheek. Blood vessels are crossing its surface and there is a small central ulcer so that the edges of the lesion appear rolled. Her parents say that the nodule often bleeds spontaneously then it seems to heal over. They also report that since birth, her skin is extremely sensitive to sunlight; she gets severe sunburns on any exposure. The child was born on time to consanguineous parents and her prenatal and postnatal history was normal during the first 6 months. After that time, her parents started to notice skin changes and signs of developmental delay. They thought that it was reaction to the influenza vaccine and stopped all immunizations after that. What is the most probable mechanism of the development of skin changes on the cheek of this patient in this age group?

Defective DNA repair

Which one of the following medications should be discontinued in a patient with diabetic gastroparesis? (check one) A. Exenatide (Byetta) B. Benazepril (Lotensin) C. Metformin (Glucophage) D. Hydrochlorothiazide E. Prochlorperazine maleate

Delayed gastric emptying may be caused or exacerbated by medications for diabetes, including amylin analogues (e.g., pramlintide) and glucagon-like peptide 1 (e.g., exenatide). Delayed gastric emptying has a direct effect on glucose metabolism, in addition to being a means of reducing the severity of postprandial hyperglycemia. In a clinical trial of exenatide, nausea occurred in 57% of patients and vomiting occurred in 19%, which led to the cessation of treatment in about one-third of patients. The other medications listed do not cause delayed gastric emptying.

Treatment of granulomatous meningitis?

Dependent on cause

Which one of the following tinea infections in children always requires systemic antifungal therapy? (check one) A. Tinea cruris B. Tinea corporis C. Tinea capitis D. Tinea pedis E. Tinea versicolor

Dermatophyte infections caused by aerobic fungi produce infections in many areas. Tinea capitis requires systemic therapy to penetrate the affected hair shafts. Tinea cruris and tinea pedis rarely require systemic therapy. Extensive outbreaks of tinea corporis and tinea versicolor benefit from both oral and topical treatment (SOR A), but more localized infections require only topical treatment

How do you treat central diabetes insipidus and DI associated with pregnancy

Desmopressin Acetate. Hydrochlorothiazide with potassium may help

Treatment of narcolepsy?

Dextroamphetamine and modafinil

A patient who takes fluoxetine (Prozac), 40 mg twice daily, develops shivering, tremors, and diarrhea after taking an over-the-counter cough and cold medication. On examination he has dilated pupils and a heart rate of 110 beats/min. His temperature is normal. Which one of the following medications in combination with fluoxetine could contribute to this patient's symptoms? (check one) A. Dextromethorphan B. Pseudoephedrine C. Phenylephrine D. Guaifenesin E. Diphenhydramine (Benadryl)

Dextromethorphan is commonly found in cough and cold remedies, and is associated with serotonin syndrome. SSRIs such as fluoxetine are also associated with serotonin syndrome, and there are many other medications that increase the risk for serotonin syndrome when combined with SSRIs. The other medications listed here are not associated with serotonin syndrome, however.

What is the biggest risk factor for frozen shoulder?

Diabetes

What can be used for severe atopic dermatits?

Topical calcineurin inhibitors (immunomodulators) as there is less atrophy with prolonged use but they carry a potential to cause malignancy.

For 2 weeks, a 62-year-old male with biopsy-documented cirrhosis and ascites has had diffuse abdominal discomfort, fever, and night sweats. His current medications are furosemide (Lasix) and spironolactone (Aldactone). On examination, his temperature is 38.0° C (100.4° F), blood pressure 100/60 mm Hg, heart rate 92 beats/min and regular. The heart and lung examination is normal. The abdomen is soft with vague tenderness in all quadrants. There is no rebound or guarding. The presence of ascites is easily verified. Bowel sounds are quiet. The rectal examination is normal, and the stool is negative for occult blood. You perform diagnostic paracentesis and send a sample of fluid for analysis. Which one of the following findings would best establish the suspected diagnosis of spontaneous bacterial peritonitis? (check one) A. pH <7.2 B. Bloody appearance C. Neutrophil count >300/mL D. Positive cytology E. Total protein >1 g/dL

Diagnostic paracentesis is recommended for patients with ascites of recent onset, as well as for those with chronic ascites who present with new clinical findings such as fever or abdominal pain. A neutrophil count >250/mL is diagnostic for peritonitis. Once peritonitis is diagnosed, antibiotic therapy should be started immediately without waiting for culture results. Bloody ascites with abnormal cytology may be seen with hepatoma, but is not typical of peritonitis. The ascitic fluid pH does not become abnormal until well after the neutrophil count has risen, so it is a less reliable finding for treatment purposes. A protein level >1 g/dL is actually evidence against spontaneous bacterial peritonitis.

How do you evaluate a TB test?

Diameter of induration NOT erythema measured at 48 and 72 hours. Over 5mm in immunosuppressed, over 10 in the healthcare worked and homeless, over 15 for everyone else.

What medication is expected to induce the most rapid clinical response in a patient with generalized anxiety disorder

Diazepam

What is the shape of M.Cat

Diplococcal gram negative

What is the shape of s. pneumo

Diplococcal gram positive

What is an empyema?

Direct infection of an exudate-Abscess

An 85-year-old white male with terminal pancreatic cancer is expected to survive for another 2 weeks. His pain has been satisfactorily controlled with sustained-release morphine. He has now developed a disturbed self-image, hopelessness, and anhedonia, and has told family members that he has thought about suicide. Psychomotor retardation is also noted. His family is supportive. His daughter feels he is depressed, while his son feels this is more of a grieving process. Which one of the following would be most appropriate for managing this problem? (check one) A. Reassurance B. Alprazolam (Xanax) C. Trazodone (Desyrel) D. Olanzapine (Zyprexa) E. Methylphenidate (Ritalin)

Distinguishing between preparatory grief and depression in a dying patient is not always simple. Initially one should evaluate for unresolved physical symptoms and treat any that are present. If the patient remains in distress, mood should be evaluated. If it waxes and wanes with time and if self-esteem is normal, this is likely preparatory grief. The patient may have fleeting thoughts of suicide and likely will express worry about separation from loved ones. This usually responds to counseling. In patients with anhedonia, persistent dysphoria, disturbed self-image, hopelessness, poor sense of self-worth, rumination about death and suicide, or an active desire for early death, depression is the problem. For patients who are expected to live only a few days, psychostimulants such as methylphenidate should be used. For those who are expected to survive longer, SSRIs are a good choice.

Isolated Systolic HTN Tx

Diuretic

African American HTN Treatment

Diuretic or CCB (not Labetalol or ACE Inhibitor)

What are the recommendations for Grade 2 (visible) ascites?

Diuretics (particularly aldosterone antagonists) plus salt restriction

Asymptomatic Aortic Stenosis

Do nothing

Attentiton deficit hyperactivity disorder (ADHD), a disorder most associated with what abnormality in the brain?

Dopamine abnormality. Medications such as methylphenidate, a central nervous system stimulant, are considered a first-line therapy for ADHD, and work by regulating dopamine and norepinephrine levels.

What medication should be used in a patient that is refractory to levadopa?

Dopamine agonists (bromocriptine)

Treatment of Restless leg syndrome?

Dopamine agonists or ropinirole are the drugs of choice. Opiate agonists and benzodiazepines may be effective

Treatment of Rocky Mountain Spotted fever

Doxycycline

What single-dose antibiotic can be used within 72 hours of tick exposure for Lyme disease?

Doxycycline

Treatment of Lyme Disease

Doxycyline

Treatment of erectile dysfunction

Sex therapy. Phosphodiesterase-5 inhibitor therapy (sidenafil, vardenafil, tadalafil) Vacuum constriction devices, injected or inserted vasoactive substances and penile prostheses

Treatment of folliculitis

Topical clindamycin or erythromycin. Bactroban works as well.

Which one of the following is the most effective drug for the treatment of alcohol dependence? (check one) A. Disulfiram (Antabuse) B. Diazepam (Valium) C. Amitriptyline (Elavil) D. Fluoxetine (Prozac) E. Naltrexone (ReVia)

Drug therapy should be considered for all patients with alcohol dependence who do not have medical contraindications to the use of the drug and who are willing to take it. Of the several drugs studied for the treatment of dependence, the evidence of efficacy is strongest for naltrexone and acamprosate. Naltrexone is currently available in the U.S.; acamprosate and tiapride are currently available in Europe but not in the U.S. Ref: Swift RM: Drug therapy for alcohol dependence.

A 64-year-old African-American male presents with persistent pleuritic pain. The patient does not feel well in general and has had a low-grade fever of around 100°F (38°C). His medications include simvastatin (Zocor), lisinopril (Prinivil, Zestril), low-dose aspirin, spironolactone (Aldactone), furosemide (Lasix), isosorbide mononitrate (Imdur), hydralazine, carvedilol (Coreg), and nitroglycerin as needed. A chest radiograph is normal and does not demonstrate a pneumothorax. Further evaluation rules out pulmonary embolus, pneumonia, and myocardial infarction. A diagnosis of pleurisy is made. Which one of the patient's medications could be related to this condition? (check one) A. Hydralazine B. Simvastatin C. Lisinopril D. Spironolactone E. Carvedilol

Drug-induced pleuritis is one cause of pleurisy. Several drugs are associated with drug-induced pleural disease or drug-induced lupus pleuritis. Drugs that may cause lupus pleuritis include hydralazine, procainamide, and quinidine. Other drugs known to cause pleural disease include amiodarone, bleomycin, bromocriptine, cyclophosphamide, methotrexate, minoxidil, and mitomycin.

What medications help painful diabetic neuropathy?

Duloxetine (a serotonin and norepinephrine reuptake inhibitor)

A 2.9 kg male infant is born to a 28-year-old woman. He is the couple's 1st child. The pregnancy and the delivery were uneventful. The infant is doing fine. However, at the routine neonatal baby check, you notice that the infant has small ears, down-sloping oval eyelid openings, and a flat occiput. He also has broad, short-fingered hands. You send blood for karyotyping on suspicion of a genetic disorder, and you inform the parents accordingly. Refer to the image. What feature could have been associated with this disorder?

Duodenal atresia

Symptoms related to pulmonary hypertension

Dyspnes, fatigue, angina like chest pain, edema, syncope, palpitations

A 7-year-old boy presents with an "itchy scalp". His mother reports the child has been scratching his head frequently for the last week. She cannot see a rash; he denies fever, chills or other symptoms of viral illness. The child has not had any dermatologic problems in the past, and he has been healthy overall. The child just says his head "itches a lot", so he scratches it. The teacher has noticed his behavior, which has become disruptive in class. The child's past medical history is unremarkable; he has no medical conditions. He has not had any surgeries. He takes no medications, and he does not have any allergies. Family history is negative for atopy and psoriasis. He lives at home with his 2 younger siblings, mother and father. He is in 1st grade and has met normal developmental milestones. On physical exam, the child appears restless and fidgety. On exam of his scalp, excoriations and small buds are noted on the hair shaft, close to the scalp. The rest of his exam is normal. What is the best intervention for this patient's condition?

Education about the condition and topical permethrin

Hydrocele treatment

Elective repair

What is the most effective antidepressant therapy?

Electroconvulsive therapy (ECT)

A 3-year-old girl presents with terrible head pain, increasing fever, and irritability. On examination, you find the child febrile, 103o F, and she cries when you lift her head from the examining table. The neck appears to be quite stiff when manipulated. You perform a lumbar puncture. What findings would you expect if this child had bacterial meningitis?

Elevated protein

First medication used in a hypertensive diabetic patient

Enalapril

Outpatient DVT Tx

Enoxaparin (Lovenox)

What are the types of insulins and how do they differ?

Rapid acting (lispro,aspart, glulisine) which peak serum valuesin 1 hour and have a four hour duration. Can be taken 20 minutes before a meal. Regular insulin is short active and can be used 1 hour before a meal, effects in 30 minutes peaks at one hour and lasts 5-7 hours. NPH insulin is long acting with onset within 2-4 hours peaks after 8-10 hours and duration 18-24 hours

Acute Intersitial Nephritis

Eosinophiliuria, pyuria, hypersensitivity to nsaids

How do you treat osteomalacia and rickets

Ergocalciferol 50,000U PO twice per week for 6-12 months, follwed by 1,000-2,000u daily to treat vitamin D deficiency. Addition of phosphate supplementation

What is the most common cause of erythema multiforme, accounting for more than 50% of cases? (check one) A. Candida albicans B. Herpes simplex virus C. Mycoplasma pneumoniae D. Penicillin therapy E. Sulfonamide therapy

Erythema multiforme usually occurs in adults 20-40 years of age, although it can occur in patients of all ages. Herpes simplex virus (HSV) is the most commonly identified cause of this hypersensitivity reaction, accounting for more than 50% of cases.

Treatment of Pertussis

Erythromycin. Alternatives are azithromycin, clarithromycin, and bactrim.

Painful ingrown toenails that display granulation tissue and lateral nail fold hypertrophy are best treated by: (check one) A. Antibiotic therapy B. Cotton-wick elevation of the affected nail corner C. Removal of the entire nail D. Excision of the lateral nail plate combined with lateral matricectomy

Excision of the lateral nail plate with lateral matricectomy yields the best results in the treatment of painful ingrown toenails that display granulation tissue and lateral nail fold hypertrophy. Antibiotic therapy and cotton-wick elevation are acceptable for very mildly inflamed ingrown toenails. Partial nail avulsion often leaves a spicule of nail that will grow and become an ingrown nail. Phenol produces irregular tissue destruction and significant inflammation and discharge after the matricectomy procedure.

A 70-year-old male complains of lower-extremity pain. Increased pain with which one of the following would be most consistent with lumbar spinal stenosis? (check one) A. Lumbar spine extension B. Lumbar spine flexion C. Internal hip rotation D. Pressure against the lateral hip and trochanter E. Walking uphill

Extension that increases lumbar lordosis decreases the cross-sectional area of the spinal canal, thereby compressing the spinal cord further. Walking downhill can cause this. Spinal flexion that decreases lordosis has the opposite effect, and will usually improve the pain, as will sitting. Pain with internal hip rotation is characteristic of hip arthritis and is often felt in the groin. Pain in the lateral hip is more typical of trochanteric bursitis. Increased pain walking uphill is more typical of vascular claudication.

Treatment of metastatic bone disease

External radiation and brain metastasis with gamma knife radiation therapy.

You are asked to perform a preoperative evaluation on a 55-year-old white female with type 2 diabetes mellitus prior to elective femoral-anterior tibial artery bypass surgery. She is unable to climb a flight of stairs or do heavy work around the house. She denies exertional chest pain, and is otherwise healthy. Based on current guidelines, which one of the following diagnostic studies would be appropriate prior to surgery because the results could alter the management of this patient? (check one) A. Pulmonary function studies B. Coronary angiography C. Carotid angiography D. A dipyridamole-thallium scan E. A hemoglobin A1c level

Family physicians are often asked to perform a preoperative evaluation prior to noncardiac surgery. This requires an assessment of the perioperative cardiovascular risk of the procedure involved, the functional status of the patient, and clinical factors that can increase the risk, such as diabetes mellitus, stroke, renal insufficiency, compensated or prior heart failure, mild angina, or previous myocardial infarction. This patient is not undergoing emergency surgery, nor does she have an active cardiac condition; however, she is undergoing a high-risk procedure (>5% risk of perioperative myocardial infarction) with vascular surgery. As she cannot climb a flight of stairs or do heavy housework, her functional status is <4 METs, and she should be considered for further evaluation. The patient's diabetes is an additional clinical risk factor. With vascular surgery being planned, appropriate recommendations include proceeding with the surgery with heart rate control, or performing noninvasive testing if it will change the management of the patient. Coronary angiography is indicated if the noninvasive testing is abnormal. Pulmonary function studies are most useful in patients with underlying lung disease or those undergoing pulmonary resection. Hemoglobin A1c is a measure of long-term diabetic control and is not particularly useful perioperatively. Carotid angiography is not indicated in asymptomatic patients being considered for lower-extremity vascular procedures.

A 27-year-old white female has a 10-year history of significant premenstrual dysphoria. Her condition has significantly worsened in the past 3 years, to the point that it is endangering her marriage of 5 years. Her symptoms are worse for the 10 days prior to her menstrual period and are gone by day 2 of her period. She has tried several measures without success, including birth control pills, various herbal preparations, and counseling at a woman's health center. Which one of the following is most likely to improve her symptoms? (check one) A. Reduction of caffeine and refined sugar intake B. Alprazolam (Xanax) C. Bupropion (Wellbutrin) D. Progesterone for 2 weeks starting at about the time of ovulation E. Fluoxetine (Prozac, Serafem) for the last 2 weeks of the menstrual cycle

Fluoxetine

Which SSRI is associated with the least weight gain?

Fluoxetine

Which one of the following is the most reliable clinical symptom of uterine rupture? (check one) A. Sudden, tearing uterine pain B. Vaginal bleeding C. Loss of uterine tone D. Fetal distress

Fetal distress has proven to be the most reliable clinical symptom of uterine rupture. The "classic" signs of uterine rupture such as sudden, tearing uterine pain, vaginal hemorrhage, and loss of uterine tone or cessation of uterine contractions are not reliable and are often absent. Pain and bleeding occur in as few as 10% of cases. Even ruptures monitored with an intrauterine pressure catheter fail to show loss of uterine tone. Signs of fetal distress are often the only manifestation of uterine rupture.

How does CAP present?

Fever/hypothermia, rigors and sweats, new cough, dyspnea, fatigue, myalgia

Which medication to treat dyslipidemia is associated with reduced synthesis and increased breakdown of very low density lipoproteins?

Fibric acid derivatives (Gemfibrozil and fenofibrate)

A mother presents with a 5-week-old male infant for excessive vomiting He has been having consistent episodes of projectile vomiting 30-40 minutes after feeding and is not gaining weight. The infant has no significant past medical history, although the mother mentions that she began renovating their Victorian era house while pregnant. He has a 3-year-old brother, who is healthy and has no significant past medical history. Pyloric stenosis is suspected given the infant's projectile vomiting. What has congenital pyloric stenosis been most commonly associated with?

First born male child

Treatment of acne vulgaris

First line: Retinoids, azelaic acid, and salicylic acid Inflammatory lesion require topical benzoyl peroxide, tretinoin, erythromycin, clindamycin Accutane (oral isotretinoin)

A 28-year-old female sees you with a complaint of irregular menses. She has not had a menstrual period for 6 months. She is also concerned about weight gain, worsening acne, and dark hair on her upper lip, chin, and periareolar region. She is also interested in becoming pregnant soon. The patient tells you she has started an exercise program, which has helped with weight loss, but she continues to have amenorrhea. She has a negative urine β-hCG test, a mild elevation in free testosterone levels, and glucose intolerance. Which one of the following would you consider initially for inducing ovulation? (check one) A. Insulin B. Metformin (Glucophage) C. Ethinyl estradiol/norgestimate (Ortho Tri-Cyclen) D. Glipizide (Glucotrol) E. Spironolactone (Aldactone)

First-line agents for ovulation induction and treatment of infertility in patients with polycystic ovary syndrome (PCOS) include metformin and clomiphene, alone or in combination, as well as rosiglitazone (SOR A). In one study of nonobese women with PCOS, metformin was found to be more effective than clomiphene for improving the rate of conception (level of evidence 1b). However, the treatment of infertile women with PCOS remains controversial. One recent group of experts recommended that metformin use for ovulation induction in PCOS be restricted to women with glucose intolerance (SOR C). Oral contraceptives are commonly used to treat menstrual irregularities in women with PCOS; however, there are few studies supporting their use, and they would not be appropriate for ovulation induction. Spironolactone is a first-line agent for treatment of hirsutism (SOR A) and has shown promise in treating menstrual irregularities, but is not commonly recommended for ovulation induction. There is a high prevalence of insulin resistance in women with PCOS, as measured by glucose intolerance; insulin-sensitizing agents are therefore indicated, but not insulin or sulfonylurea medications.

Treatment of Shigellosis

Fluid replacement. Bactrim is the ABX of choice though ciprofloxacin or fluoroquinolones may be substituted.

Which one of the following should be used first for ventricular fibrillation when an initial defibrillation attempt fails? (check one) A. Amiodarone (Cordarone) B. Lidocaine (Xylocaine) C. Adenosine (Adenocard) D. Vasopressin (Pitressin) E. Magnesium

For persistent ventricular fibrillation (VF), in addition to electrical defibrillation and CPR, patients should be given a vasopressor, which can be either epinephrine or vasopressin. Vasopressin may be substituted for the first or second dose of epinephrine. Amiodarone should be considered for treatment of VF unresponsive to shock delivery, CPR, and a vasopressor. Lidocaine is an alternative antiarrhythmic agent, but should be used only when amiodarone is not available. Magnesium may terminate or prevent torsades de pointes in patients who have a prolonged QT interval during normal sinus rhythm. Adenosine is used for the treatment of narrow complex, regular tachycardias and is not used in the treatment of ventricular fibrillation.

A 3-year-old, HIV-positive boy has contracted varicella infection. He has not shown a satisfactory response to treatment with acyclovir. He is suspected of having an acyclovir resistant varicella zoster infection. What is the alternate drug appropriate for treatment of varicella in this child? Zidovudine Ribavirin Oseltamivir Foscarnet Nevirapine

Foscarnet Foscarnet is the appropriate drug for treatment of varicella in the above child. Acyclovir resistant varicella-zoster virus (VZV) infection has been identified in children infected with HIV. These children may be treated with intravenous foscarnet 120 mg/kg/day, divided every 8 hours for up to 3 weeks. The dose needs to be modified in the presence of renal insufficiency. When foscarnet is used to treat VZV infection in acquired immunodeficiency syndrome (AIDS) patients, it also decreases HIV viral load, but it is not used primarily for HIV infection.

A 46-year-old female presents to your office with a 2-week history of pain in her left shoulder. She does not recall any injury, and the pain is present when she is resting and at night. Her only chronic medical problem is type 2 diabetes mellitus. On examination, she has limited movement of the shoulder and almost complete loss of external rotation. Radiographs of the shoulder are normal, as is her erythrocyte sedimentation rate. Which one of the following is the most likely diagnosis? (check one) A. Frozen shoulder B. Torn rotator cuff C. Impingement syndrome D. Chronic posterior shoulder dislocation E. Osteoarthritis

Frozen shoulder is an idiopathic condition that most commonly affects patients between the ages of 40 and 60. Diabetes mellitus is the most common risk factor for frozen shoulder. Symptoms include shoulder stiffness, loss of active and passive shoulder rotation, and severe pain, including night pain. Laboratory tests and plain films are normal; the diagnosis is clinical (SOR C). Frozen shoulder is differentiated from chronic posterior shoulder dislocation and osteoarthritis on the basis of radiologic findings. Both shoulder dislocation and osteoarthritis have characteristic plain film findings. A patient with a rotator cuff tear will have normal passive range of motion. Impingement syndrome does not affect passive range of motion, but there will be pain with elevation of the shoulder.

What is tarsal coalition?

Fusion of two or more tarsal bones; presents with pain around the ankle, decreased motion of hindfoot, and pain with inversion; bilateral 50% of the time and occurs during adolescence

A previously healthy 82-year-old male is brought to your office by his daughter after a recent fall while getting up to go to the bathroom in the middle of the night. The patient denies any history of dizziness, chest pain, palpitations, or current injury. He has a history of bilateral dense cataracts. On examination, he is found to have an increased stance width and walks carefully and cautiously with his arms and legs abducted. A timed up-and-go test is performed, wherein the patient is asked to rise from a chair without using his arms, walk 3 meters, turn, return to his chair, and sit down. It takes the patient 25 seconds and he is noted to have an "en bloc" turn. Which one of the following is the most likely cause of this patient's gait and balance disorder? (check one) A. Visual impairment B. Cerebellar degeneration C. Frontal lobe degeneration D. Parkinson's disease E. Motor neuropathy

Gait and balance disorders are one of the most common causes of falls in older adults. Correctly identifying gait and balance disorders helps guide management and may prevent consequences such as injury, disability, loss of independence, or decreased quality of life. The "Timed Up and Go" test is a reliable diagnostic tool for gait and balance disorders and is quick to administer. A time of <10 seconds is considered normal, a time of >14 seconds is associated with an increased risk of falls, and a time of >20 seconds usually suggests severe gait impairment. This patient has the cautious gait associated with visual impairment. It is characterized by abducted arms and legs; slow, careful, "walking on ice" movements; a wide-based stance; and "en bloc" turns. Patients with cerebellar degeneration have an ataxic gait that is wide-based and staggering. Frontal lobe degeneration is associated with gait apraxia that is described as "magnetic," with start and turn hesitation and freezing. Parkinson's disease patients have a typical gait that is short-stepped and shuffling, with hips, knees, and spine flexed, and may also exhibit festination and "en bloc" turns. Motor neuropathy causes a "steppage" gait resulting from foot drop with excessive flexion of the hips and knees when walking, short strides, a slapping quality, and frequent tripping.

A male newborn is brought to the NICU with an uncovered anterior abdominal mass. The mass extends off the right side of the abdomen and measures 3.5 cm. It appears to contain the small bowel and is next to the umbilical cord. No other physical abnormalities are seen and a genetic profile is pending. What is the most likely differential diagnosis for this child?

Gastroschisis

Which one of the following is recommended for the treatment of intravaginal genital warts in pregnant women? (check one) A. Imiquimod 5% cream (Aldara) B. Podofilox 0.5% solution (Condylox) C. Podophyllin 10%-25% in tincture of benzoin (Podofin) D. Cryotherapy with liquid nitrogen E. Interferon-alpha

Genital warts can proliferate and fragment during pregnancy, and many specialists recommend that they be eliminated. Imiquimod, podophyllin, and podofilox are not recommended for use during pregnancy. For the treatment of vaginal warts, the Centers for Disease Control and Prevention (CDC) recommends the use of cryotherapy. Liquid nitrogen, rather than a cryoprobe, should be used to avoid possible vaginal perforation and subsequent fistula formation. An alternative is the use of trichloroacetic acid or bichloroacetic acid carefully applied to the lesions to avoid damage to adjacent tissue. Interferon is no longer recommended for routine use in treating genital warts, due to a high frequency of systemic adverse effects.

Treatment of blepharitis

Gentle scrubs using baby sampoo, follow with a suspension of sulfa and/orsteroid preparation or ketoconazole cream

Jaw claudication, intractable headache, and visual field changes

Giant Cell Arteritis

An African American female newborn born 12 hours ago presents with yellowish coloration of the whites of her eyes. Her skin also appears darker and yellowish compared with her twin brother' skin. Pregnancy was normal, and the 23-year-old mother had no infections or complications and took no drugs during the pregnancy. Delivery was uneventful; the babies were born on term with APGAR score 9 and 10, respectfully. Family history of anemia, splenectomy, bile stones, and liver disease is negative, but the father has a "beans allergy," which presents with abdominal pain and jaundice. Peripheral smear does not reveal spherocytosis, echinocytosis, or eliptocytosis, but some keratocytes are present. Bilirubin levels in your patient are high (13 mg/dL) with direct bilirubin 1 mg/dL. Coombs test is negative and hemoglobin is low.

Glucose-6-phosphate dehydrogenase deficiency

What exacerbates pain in patellofemoral pain syndrome?

Going up or down stairs

Which medication to treat dyslipidemia is associated with inhibiting the formation of cholesterol, and reducing in CAD and total mortality

HMG-CoA inhibitors (Statins) Lovastatin, pravastatin, simvastatin, fluvastatin, rosuvastatin, atorvastatin

What infection is most associated with erythema multiforme?

HSV

Neuroblastoma markers

HVA, VMA (same as Pheo)

A 1-year-old girl presents for a well child visit. She has been well, except for 1 upper respiratory infection in the past 6 months. She is not in daycare. She has begun to eat table foods and is growing well. Her mother has no specific concerns. Past medical history and family history are unremarkable. Physical examination is normal. What developmental milestone should she have now achieved that is met by greater than 95% of infants her age? Walks well Has a very good pincer grasp Hugs her parents Indicates wants by pointing Has several intelligible words

Has a very good pincer grasp

Thiazolidinediones are primarily contraindicated in...?

Heart failure

Best initial managment of A-Fib

Rate control with beta blockers or calcium channel blockers

While doing newborn nursery rounds, you are called about a male infant that was born 48 hours ago. His nurse is concerned that he has not passed any meconium yet. The baby was born at 38 weeks gestation and had a normal spontaneous vaginal delivery with Apgars of 9 and 9 at 1 and 5 minutes. He had fed well, but is now refusing feedings and had 1 episode of slightly green-tinged emesis. On exam, he is somewhat irritable and has abdominal distention with a tightly contracted anal sphincter with empty rectal vault. There is some passage of stool after the exam. Plain abdominal films reveal multiple loops of dilated small bowel with decreased gas in the rectum. His mother also tells you that her other toddler son has been having problems with constipation. What is the most likely diagnosis in this infant? Hirschsprung's disease Imperforate anus Necrotizing enterocolitis Meconium plug syndrome Cystic fibrosis

Hirschsprung's disease Hirschsprung's disease (HSD) is characterized by the congenital absence of ganglion cells in the myenteric and submucosal plexuses of the intestine resulting in abnormal intestinal motility and obstruction. This usually begins at the internal anal sphincter and extends a variable distance proximally. HSD has a male predominance, and it can be sporadic or familial, and is usually an isolated defect. There is an inherited predisposition for HSD with the incidence of affected siblings being about 3.5%. Most infants will fail to pass meconium by 48 hours of age, may have bilious vomiting or infrequent stools, abdominal distention, and refusal to feed. A contracted anal sphincter, empty rectal vault, and release of explosive foul-smelling stool may be present on digital exam.

A 45-year-old female presents with a 3-month history of hoarseness that is not improving. She works as a high-school teacher. The most appropriate management at this time would be: (check one) A. voice therapy B. azithromycin (Zithromax) C. a trial of inhaled corticosteroids D. a trial of a proton pump inhibitor E. laryngoscopy

Hoarseness most commonly affects teachers and older adults. The cause is usually benign, but extended symptoms or certain risk factors should prompt evaluation; specifically, laryngoscopy is recommended when hoarseness does not resolve within 3 months or when a serious underlying cause is suspected (SOR C). The American Academy of Otolaryngology/Head and Neck Surgery Foundation guidelines state that antireflux medications should not be prescribed for patients with hoarseness without reflux symptoms (SOR C). Antibiotics should not be used, as the condition is usually caused by acute laryngitis or an upper respiratory infection, and these are most likely to be viral. Inhaled corticosteroids are a common cause of hoarseness. Voice therapy should be reserved for patients who have undergone laryngoscopy first (SOR A).

What is a homeopathic treatment for varicose veins?

Horse chestnut seed extract

Treatment of Diphtheria

Horse serum antitoxin obtained from the CDC. Penicillin or erythromycin are effective.

A male neonate is brought to the NICU for care post-premature birth. He weights 4.43 lbs. A neonatal brain sonogram is ordered for day 1 and day 7 to rule out any intracranial hemorrhage. A respirator is attached to ensure his consistent breathing. Blood gasses and skin color of the neonate are good, as is his urine output. His chest X-ray is normal. The neonate is most susceptible to what condition at this time?

Hyaline membrane disease

Which one of the following drugs used to treat rheumatoid arthritis can delay the progression of the disease? (check one) A. Aspirin B. Ibuprofen C. Indomethacin (Indocin) D. Capsaicin (Zostrix) E. Hydroxychloroquine (Plaquenil)

Hydroxychloroquine, originally developed as an antimalarial drug, is a well-known disease-modifying agent that can slow the progression of rheumatoid arthritis. Aspirin, indomethacin, and ibuprofen are anti-inflammatory agents. They relieve pain and improve mobility, but do not alter the progression of the disease. Capsaicin, a topical substance-P depleter, can relieve pain symptoms.

A 2-year-old child presents with a tonic convulsion. His mother states that he is usually sick; he has had recurrent infections since he was born. His temp is 99° F. He also has special facial features; his eyes are separated by a wide space, and he is known to have a congenital heart disease. What is the most likely cause of the convulsion?

Hypoparathyroidism

What is the most common electrolyte abnormality in Refeeding Syndrome

Hypophosphatemia

Whats is the preexposure immunization schedule for rabies?

IM doses on days 0, 7, and either day 21 or 28.

Which one of the following medications has the best evidence for preventing hip fracture? (check one) A. Ibandronate (Boniva) B. Raloxifene (Evista) C. Denosumab (Prolia) D. Etidronate (Didronel) E. Alendronate (Fosamax)

Ibandronate, raloxifene, denosumab, and etidronate have been shown to reduce new vertebral fractures, but are not proven to prevent hip fracture. Only zoledronic acid, risedronate, and alendronate have been confirmed in sufficiently powered studies to prevent hip fracture, and these are the anti-osteoporosis drugs of choice.

When to biopsy lymph node

Immediate biopsy is warranted if the patient does not have inflammatory symptoms and the lymph node is >3 cm, if the node is in the supraclavicular area, or if the patient has coexistent constitutional symptoms such as night sweats or weight loss. Immediate evaluation is also indicated if the patient has risk factors for malignancy. Treatment with antibiotics is warranted in patients who have inflammatory symptoms such as pain, erythema, fever, or a recent infection.

A 4-year-old HIV-infected (but asymptomatic) child presents for a polio vaccination. Her parents received a postcard reminding them of the routinely recommended immunization. What is the most appropriate next step? Immunize and record it in the patient's chart Immunize but do not record in the chart Postpone immunization till the age of 5 years Cancel immunization with this vaccine Administer human serum immunoglobulin

Immunize and record it in the patient's chart The inactivated polio vaccine (IPV) is the routinely recommended polio vaccine is the US now. The oral polio vaccine (OPV) is no longer recommended. HIV positive children can receive the IPV; it is inactivated and does not pose a risk of neurovirulence.

Which MS medication may arrest the course of the secondary progressive form

Immunosuppresive agents such as cyclophosphamie or azathioprine

You would recommend pneumococcal vaccine for which one of the following? (check one) A. A 20-year-old male who smokes 1 pack of cigarettes daily B. A 52-year-old male with type 2 diabetes mellitus who received pneumococcal vaccine 6 years ago C. A 60-year-old male who is a long-term resident of a nursing home because of a previous stroke, and who received pneumococcal vaccine at age 54 D. A 62-year-old male with chronic renal failure who received pneumococcal vaccine at age 50 and age 55 E. A 71-year-old male with no medical problems who received pneumococcal vaccine at age 65

In October 2008 the Advisory Committee on Immunization Practices (ACIP) of the Centers for Disease Control and Prevention recommended adding cigarette smoking to the list of high-risk conditions that are indications for the 23-valent pneumococcal polysaccharide vaccine. All persons between the ages of 19 and 64 who smoke should receive this vaccine. One-time revaccination after 5 years is recommended for persons with chronic renal failure, asplenia (functional or anatomic), or other immunocompromising conditions. The patient with chronic renal failure in this question has already received two immunizations. The diabetic patient and the nursing-home resident have both received one immunization and should not receive a second dose until age 65. The 71-year-old has already been immunized after age 65, and a repeat immunization is not recommended.

A 26-year-old gravida 3 para 2 was diagnosed with gestational diabetes mellitus at 24 weeks gestation. She was prescribed appropriate nutritional therapy and an exercise program. After 4 weeks, her fasting plasma glucose levels remain in the range of 105-110 mg/dL. Which one of the following would be the most appropriate treatment for this patient at this time? (check one) A. Continuation of the current regimen B. Long-acting insulin glargine (Lantus) once daily C. Pioglitazone (Actos) once daily D. A combination of intermediate-acting insulin (e.g., NPH) and a short-acting insulin (e.g., lispro) twice daily E. Sliding-scale insulin 4 times daily using ultra-short-acting insulin

In addition to an appropriate diet and exercise regimen, pharmacologic therapy should be initiated in pregnant women with gestational diabetes mellitus whose fasting plasma glucose levels remain above 100 mg/dL despite diet and exercise. There is strong evidence that such treatment to maintain fasting plasma glucose levels below 95 mg/dL and 1-hour postprandial levels below 140 mg/dL results in improved fetal well-being and neonatal outcomes. While oral therapy with metformin or glyburide is considered safe and possibly effective, insulin therapy is the best option for the pharmacologic treatment of gestational diabetes. Thiazolidinediones such as pioglitazone have not been shown to be effective or safe in pregnancy. The use of long-acting basal insulin analogues, such as glargine and detemir, has not been sufficiently evaluated in pregnancy. Sliding-scale coverage with ultra-short-acting insulin or insulin analogues, such as lispro and aspart, is generally not required in most women with gestational diabetes. While it may be effective, it involves four daily glucose checks and injections. Most patients are successfully treated with a twice-daily combination of an intermediate-acting insulin and a short-acting insulin while continuing a diet and exercise program.

A 59-year-old male with known cirrhosis is beginning to show some lower abdominal distention. Ultrasonography confirms your suspicion that he has developed moderate ascites for the first time. Which one of the following is recommended as the initial treatment of choice for this condition? (check one) A. Chlorthalidone B. Spironolactone (Aldactone) C. Furosemide (Lasix) D. Ramipril (Altace) E. Large-volume paracentesis

In patients with grade 2 ascites (visible clinically by abdominal distention, not just with ultrasonography), the initial treatment of choice is diuretics along with salt restriction. Aldosterone antagonists such as spironolactone are more effective than loop diuretics such as furosemide (SOR A). Chlorthalidone, a thiazide diuretic, is not recommended. Large-volume paracentesis is the recommended treatment of grade 3 ascites (gross ascites with marked abdominal distention), and is followed by salt restriction and diuretics.

In a patient who presents with symptoms of acute myocardial infarction, which one of the following would be an indication for thrombolytic therapy? (check one) A. New-onset ST-segment depression B. New-onset left bundle branch block C. New-onset first degree atrioventricular block D. New-onset Wenckebach second degree heart block E. Frequent unifocal ventricular ectopic beats

In patients with ischemic chest pain, the EKG is important for determining the need for fibrinolytic therapy. Myocardial infarction is diagnosed by ST elevation ≥1 mm in two or more limb leads and ≥2 mm in two or more contiguous precordial leads. In a patient with an MI, new left bundle branch block suggests occlusion of the left anterior descending artery, placing a significant portion of the left ventricle in jeopardy. Thrombolytic therapy could be harmful in patients with ischemia but not infarction - they will show ST-segment depression only. Frequent unifocal ventricular ectopy may warrant antiarrhythmic therapy, but not thrombolytic therapy.

What occurs during latent TB

Individual is infect but not ill. The immune system has the disease process under control. Not infectious, will have positive test results.

Which NSAID causes more CNS effects in elderly patients?

Indomethacin

How do you treat nephrogenic diabetes insipidus

Indomethacin alone or in combination with hydrochlorothiazide, desmopressin, or ameloride

Treatment of psoriasis

Topical corticosteroids and topical vitamin D preparations (calcipotriene) . Systemic steroids, coal tar or salicyclic acid preparations, tazarotene gel. More severe cases UVB light, PUVA, and methotrexate

A 42-year-old male with well-controlled type 2 diabetes mellitus presents with a 24-hour history of influenza-like symptoms, including the sudden onset of headache, fever, myalgias, sore throat, and cough. It is December, and there have been a few documented cases of influenza recently in the community. The CDC recommends initiating treatment in this situation: (check one) A. on the basis of clinical symptoms alone B. only if rapid influenza testing is positive C. only if the diagnosis is confirmed by immunoassay testing D. only if the diagnosis is confirmed by reverse transcriptase polymerase chain reaction (PCR) assay

Influenza is a highly contagious viral illness spread by airborne droplets. This patient's symptoms are highly suggestive of typical influenza: a sudden onset of malaise, myalgia, headache, fever, rhinitis, sore throat, and cough. While influenza is typically uncomplicated and self-limited, it can result in severe complications, including encephalitis, pneumonia, respiratory failure, and death. The effectiveness of treatment for influenza is dependent on how early in the course of the illness it is given. Because of the recent global H1N1 influenza outbreak that resulted in demand potentially outstripping the supply of antiviral medication, the Centers for Disease Control and Prevention has modified its recommendation as follows: Antiviral treatment is recommended as soon as possible for patients with confirmed or suspected influenza who have severe, complicated, or progressive illness or who require hospitalization. Antiviral treatment is recommended as soon as possible for outpatients with confirmed or suspected influenza who are at higher risk for influenza complications based on their age or underlying medical conditions. Clinical judgment should be an important component of outpatient treatment decisions. Antiviral treatment also may be considered on the basis of clinical judgment for any outpatient with confirmed or suspected influenza who does not have known risk factors for severe illness, if treatment can be initiated within 48 hours of illness onset. Many rapid influenza tests produce false-negative results, and more accurate assays can take more than 24 hours. Thus, treatment of patients with a clinical picture suggesting influenza is recommended, even if a rapid test is negative. Delaying treatment until further test results are available is not recommended.

A 7-year-old boy presents for evaluation of behavior problems in school; the problems have been occurring for over 6 months. The boy's teacher recommended he be evaluated. She reports that he tests at grade level, but he seems to make careless mistakes on schoolwork and has trouble maintaining attention to instruction; he does not finish his homework, and he often loses his homework, pencils, and books. The boy is seen often fidgeting at his desk; he blurts out answers and has difficulty waiting his turn. The parents tell you that they have seen similar traits at home, such as forgetting to do daily activities; for the past several years, he has been easily distracted. They view him as a happy, bright boy, and they report that he is very active. He has had normal vision and hearing screenings. The father reports that he was very similar as a child and still struggles with focus and concentration as an adult, but he has never received any help. The parents deny any major changes in the family situation. They would like medical help to improve their son's performance in school. The boy has been seen regularly for his well-child exams, and he has always met milestones and had normal exams. Today, he is quickly moving about the exam room; he looks at a book for a few moments, and then he looks out the window for a short time before interrupting his parents. What is the most appropriate intervention in this case?

Initiate medication

You evaluated a 5-year-old girl at a well-child visit. There were no complaints. Past medical and family histories were unremarkable. Growth and development were normal and immunizations were current. Vital signs, growth parameters, and physical examination were normal. In reviewing safety guidelines as part of anticipatory guidance, the parents noted that they had recently installed an in-ground pool and that their daughter was quite interested in swimming. What is the most effective water safety measure regarding the pool? Instructing the child to stay away from the pool unless accompanied by an adult Insuring the child learns to swim from a certified instructor Providing approved flotation devices Installing appropriate pool fencing with self-latching gates Hiring a full time lifeguard during the summer

Installing appropriate pool fencing with self-latching gates Drowning is still a major cause of death and disability in children, with significant neurologic sequelae in survivors. Prevention of drowning is more effective than treatment. While constant parental supervision is the ideal, given the inevitable lapses of such supervision, other measures are necessary. As to in-ground pools, the most effective preventive measure is to equip the pool with appropriate fencing with self-latching gates to protect children who live at the residence, as well as guests and children who may wander onto the property. This will help to insure that no child is in the water without appropriate supervision. It would not be prudent to expect a 5-year-old child to always follow her parents' instructions and, in any event, this would not prevent accidents to guests and those who wander onto the property.

Treatment of type I DM

Insulin. Diet control. every 10-15 g of carbs = 1 U of insulin

Which MS medication decreases the frequency of relapses, especially moderate to severe attacks?

Interferon-Beta

A 65-year-old female who is morbidly obese presents to your office with intertrigo in the axilla. On examination you detect small, reddish-brown macules that are coalescing into larger patches with sharp borders. You suspect cutaneous erythrasma complicating the intertrigo. What would be the most appropriate topical treatment for this condition? (check one) A. Cornstarch B. A mild corticosteroid lotion C. A high-potency corticosteroid lotion D. Erythromycin

Intertrigo is inflammation of skinfolds caused by skin-on-skin friction and is common on opposing cutaneous or mucocutaneous surfaces. Secondary cutaneous bacterial and fungal infections are common complications. Cutaneous erythrasma may complicate intertrigo of interweb areas, intergluteal and crural folds, axillae, or inframammary regions. Erythrasma is caused by Corynebacterium minutissimum and presents as small reddish-brown macules that may coalesce into larger patches with sharp borders. Intertrigo complicated by erythrasma is treated with topical or oral erythromycin.

Treatment of severe hypocalcemia

Intravenous calcium gluconate or calcium chloride

What is a temporary treatment option in a highly symptomatic patient?

Iodinated contrast agents

Latent TB Infection Tx

Isoniazid for 9 months

Treatment of hypercalcemia

Isotonic saline to help with volume repletion. Loop diuretics may be necessary if patient is hypervolemic after treatment.

A 68-year-old female is being monitored in the hospital after elective surgery. On her third postoperative day she suddenly develops hypoxia, fever, tachycardia, and hypotension. You institute high-rate intravenous fluids and empiric antibiotics. However, approximately 2 hours into this therapy, her blood pressure remains at 80 mm Hg systolic with sluggish urine output. Which one of the following hormones should be assessed at this time? (check one) A. Aldosterone B. Catecholamines C. Cortisol D. Renin E. TSH

It has been recognized that patients suffering from a critical illness with an exaggerated inflammatory response often have a relative cortisol deficiency. Clinically, this can cause hypotension that is resistant to intravenous fluid resuscitation, and evidence is mounting that survival is increased if these patients are treated with intravenous corticosteroids during acute management. Cortisol levels can be assessed with a single serum reading, or by the change in the cortisol level after stimulation with cosyntropin (referred to as Δcortisol). The other hormones listed are not important for the acute management of a critically ill patient.

For a healthy 1-month-old, daily vitamin D intake should be: (check one) A. 50 IU B. 100 IU C. 200 IU D. 400 IU E. 800 IU

It is now recommended that all infants and children, including adolescents, have a minimum daily intake of 400 IU of vitamin D, beginning soon after birth. The current recommendation replaces the previous recommendation of a minimum daily intake of 200 IU/day of vitamin D supplementation beginning in the first 2 months after birth and continuing through adolescence. These revised guidelines for vitamin D intake for healthy infants, children, and adolescents are based on evidence from new clinical trials and the historical precedent of safely giving 400 IU of vitamin D per day in the pediatric and adolescent population. New evidence supports a potential role for vitamin D in maintaining innate immunity and preventing diseases such as diabetes mellitus and cancer.

A 67-year-old white male with hypertension and chronic kidney disease presents with the recent onset of excessive thirst, frequent urination, and blurred vision. Laboratory testing reveals a fasting blood glucose level of 270 mg/dL, a hemoglobin A 1c of 8.5%, a BUN level of 32 mg/dL, and a serum creatinine level of 2.3 mg/dL. His calculated glomerular filtration rate is 28 mL/min. Which one of the following medications should you start at this time? (check one) A. Glipizide (Glucotrol) B. Metformin (Glucophage) C. Glyburide (DiaBeta) D. Acarbose (Precose)

It is recommended that metformin be avoided in patients with a creatinine level >1.5 mg/dL for men or >1.4 mg/dL for women. Glyburide has an active metabolite that is eliminated renally. This metabolite can accumulate in patients with chronic kidney disease, resulting in prolonged hypoglycemia. Acarbose should be avoided in patients with chronic kidney disease, as it has not been evaluated in these patients. Glipizide does not have an active metabolite, and is safe in patients with chronic renal disease.

What is the triad of symptoms associated with Cor pulmonale?

JVD, accentuated 2nd heart sound, peripheral edema, hepatomegaly

A 3-year-old boy presents with an 8-day fever as high as 104° F; it is accompanied by a rash and oral sores. His parents have had little success in reducing the fever with antipyretics. He has been alert, but irritable, and has managed to take fluids well. There is no history of recent exposure to illness. Past history is unremarkable except for upper respiratory symptoms a few days before the fever began. Family history is unremarkable. Growth and development have been normal and immunizations are current. Examination reveals a mildly ill-appearing child. A pale, red maculopapular rash is present on the trunk and extremities. The dorsae of the hands and feet are red and swollen. The bulbar conjunctivae are red, bilaterally, without exudates. The tongue appears red with white spots and the oral mucosa is generally reddened. Enlarged, nonfluctuant, right anterior cervical lymph nodes are present. What is the most likely diagnosis? Kawasaki disease Lyme disease Mononucleosis Rocky Mountain spotted fever Scarlet fever

Kawasaki disease The presentation in this child is typical of Kawasaki disease (mucocutaneous lymph node syndrome), a generalized vasculitis involving small to medium-sized arteries affecting mainly children under 10 years of age (most of whom are younger than 5 years of age). While bacterial, viral, rickettsial, environmental, and other etiologies have been proposed, the cause remains unknown; it is presumed to be infectious. Fever typically lasts 7 - 10 days, often reaching 104° F. Fever for 5 days or longer is a necessary diagnostic criterion, in addition to 4 out of 5 of the following symptoms: 1) erythema and edema of the hands and feet, with desquamation approximately 10 days later; 2) nonexudative bulbar conjunctivitis; 3) oral mucosal manifestations, including strawberry tongue, mucosal erythema, and fissuring of the lips; 4) nonpurulent cervical lymphadenopathy, usually unilateral; and 5) a polymorphous skin eruption that may be macular, papular, maculopapular, morbilliform, or target-like (but not vesicular) and may be generalized or confined to the extremities. Cutaneous findings are the most variable and most commonly absent. Incomplete forms occur.

Hep C Risk Factors

Long term hemodialysis IV Drug Use Blood Transfusion or Organ Transplantation prior to 1992 Receipt of Clotting Factors before 1987

A 4-year-old girl presents with a 5-day history of fever, sore throat, skin rash, and right sided neck swelling. She has no significant past medical history. The neck swelling has increased in size over the last 24 hours to about 2.5 cm. There is no history of any injury, recent travel, or sick contacts. On examination, the neck mass is located near the angle of the jaw; it is rounded, firm, tender, and mobile, but the overlying skin shows no erythema or breakdown. Her conjunctiva appear red, but no eye discharge is noted. The skin rash is diffuse and maculopapular, and her hands and feet also show mild swelling. Oral examination reveals red, fissured lips, a strawberry tongue, and erythematous pharyngeal walls; there is no tonsillar enlargement. A rapid Streptococcal test done in the clinic is negative. Given this patient's history, symptoms, and exam findings, what is the most likely cause of her symptoms? Staphylococcus aureus lymphadenitis Streptococcal lymphadenitis Tubercular lymphadenitis Kawasaki disease Histoplasmosis

Kawasaki disease This child has the classic findings of Kawasaki Disease, and this diagnosis has to be made on the basis of this combination of clinical findings. Along with presence of fever of 5 days' duration, presence of any 4 of the following is considered diagnostic for Kawasaki Disease: Edema and redness of hands and feet Polymorphic rash Bilateral painless conjunctival injection without exudates Lips and oral cavity showing fissured and cracked lips, strawberry tongue or erythema of oral and pharyngeal mucosa Unilateral cervical lymphadenopathy (>1.5cm diameter)

A 9-year-old boy presents with sudden onset of anterior chest pain, vomiting, and general malaise. 12 months prior to presentation, the patient had a febrile illness associated with conjunctivitis and peeling of the skin on the hands and feet. Physical exam is as follows: Temperature 99°F; pulse 120 beat per minute: respirations 23 per minute; blood pressure 110/60. Skin: normal with no petechiae or splinter hemorrhages. Lungs: Clear. Heart: Normal. Lab: WBC 8,900/mm3, hematocrit 35%. EKG: ST elevation in the right precordial leads with an anterior myocardial infarction. Based on the history and physical exam, what is the most likely diagnosis? Bicuspid aortic valve Kawasaki's disease Rheumatic heart disease Coarctation of the aorta Bernard-Soulier syndrome

Kawasaki's disease The patient's presentation is most consistent with the diagnosis of Kawasaki's disease. Kawasaki's disease is a febrile condition of early childhood whose exact etiology is uncertain. The clinical syndrome includes erythema and desquamation of the skin on the palms of the hands and feet. Conjunctivitis and reddening of the tongue (so-called "strawberry tongue") as well as cervical lymphadenopathy may also be present. Long-term, the illness causes vasculitis that may eventually lead to coronary aneurysmal dilatation. These aneurysms can subsequently thrombose, producing acute coronary insufficiency and myocardial ischemia/infarction.

Treatment of incontinence

Kegel exercises, electrical muscle stimulation, bladder training. Anticholinergic medication such as oxybutynin or tolterodine. Estrogen for stress incontinence.

A 65-year-old white male comes to your office with a 0.5-cm nodule that has developed on his right forearm over the past 4 weeks. The lesion is dome shaped and has a central plug. You schedule a biopsy but he does not return to your office for 1 year. At that time the lesion appears to have healed spontaneously. The most likely diagnosis is (check one) A. benign lentigo B. lentigo maligna C. basal cell carcinoma D. squamous cell carcinoma E. keratoacanthoma

Keratoacanthoma grows rapidly and may heal within 6 months to a year. Squamous cell carcinoma may appear grossly and histologically similar to keratoacanthoma but does not heal spontaneously. The other lesions do not resemble keratoacanthoma.

Which asthma medication should not be monotherapy

LABA

What are the recommendations for Grade 3 (marked) ascites?

Large-volume paracentesis plus aldosterone antagonists plus salt restriction

Late decelerations on fetal monitoring are thought to indicate which one of the following? (check one) A. Fetal head compression B. Umbilical cord compression C. Fetal sleep D. Uterine hypotonus E. Uteroplacental insufficiency

Late decelerations are thought to be associated with uteroplacental insufficiency and fetal hypoxia due to decreased blood flow in the placenta. This pattern is a warning sign and is associated with increasing fetal compromise, worsening fetal acidosis, fetal central nervous system depression, and/or direct myocardial hypoxia. Early decelerations are thought to result from vagus nerve response to fetal head compression, and are not associated with increased fetal mortality or morbidity. Variable decelerations are thought to be due to acute, intermittent compression of the umbilical cord between fetal parts and the contracting uterus.

A 30-year-old white gravida 2 para 1 who has had no prenatal care presents for urgent care at 33 weeks gestation. Her symptoms include vaginal bleeding, uterine tenderness, uterine pain between contractions, and fetal distress. Her first pregnancy was uncomplicated, with a vaginal delivery at term. Which one of the following is the most likely diagnosis? (check one) A. Uterine rupture B. Vasa previa C. Placenta previa D. Placental abruption E. Cervical cancer

Late pregnancy bleeding may cause fetal morbidity and/or mortality as a result of uteroplacental insufficiency and/or premature birth. The condition described here is placental abruption (separation of the placenta from the uterine wall before delivery). There are several causes of vaginal bleeding that can occur in late pregnancy that might have consequences for the mother, but not necessarily for the fetus, such as cervicitis, cervical polyps, or cervical cancer. Even advanced cervical cancer would be unlikely to cause the syndrome described here. The other conditions listed may bring harm to the fetus and/or the mother. Uterine rupture usually occurs during active labor in women with a history of a previous cesarean section or with other predisposing factors, such as trauma or obstructed labor. Vaginal bleeding is an unreliable sign of uterine rupture and is present in only about 10% of cases. Fetal distress or demise is the most reliable presenting clinical symptom. Vasa previa (the velamentous insertion of the umbilical cord into the membranes in the lower uterine segment) is typically manifested by the onset of hemorrhage at the time of amniotomy or by spontaneous rupture of the membranes. There are no prior maternal symptoms of distress. The hemorrhage is actually fetal blood, and exsanguination can occur rapidly. Placenta previa (placental implantation that overlies or is within 2 cm of the internal cervical os) is clinically manifested as vaginal bleeding in the late second or third trimester, often after sexual intercourse. The bleeding is typically painless, unless labor or placental abruption occurs.

How to treat TB?

Latent TB-Isoniazd for 9 months Active- four drugs (Isoniazid, rifampin, pyrazinamide, ethambutol) for 8 weeks then two drugs( isoniazid and rifampin) for 16 weeks

Treatment of Pityriasis rosea

Lotions or emollients. UVB phototherapy

Treatment of a benign essential (familial) tremor

Low dose of a Beta blocker or primidone

Osteoporosis Dx

Low impact fracture of T score < -2.5

Your patient is an 18-month-old boy whose parents are concerned that he is "mentally backward." He was born to a 27-year-old mother and delivered at term with no prenatal, labor, or postnatal complications. His birth weight was 3.6 kg, and he was breast fed for 3 months. Between 3 and 6 months, a babysitter took care of the boy while the mother began renovating an old farm house the family wanted to move into. The mother has had several episodes of stomach aches and severe nausea that she thinks have been caused by unhealthy foods since beginning renovations. Also, heavy physical activity (sanding and torching) left her feeling very tired and weak. As soon as the renovation was completed, the family moved into their new home and the child has since lost his appetite, has frequents attacks of abdominal pain and constipation, vomits often, and is not gaining weight as expected. He rolled over at 5 months, sat without support at 7 months, crawled at 10 months, and began to walk around 14 months, but most worrisome is the fact that he is still not talking; he started to babble at about 9 months of age and only recently learned a few words such as "Dada" and "Boo." At the examination, you find a pale irritable child without dysmorphic features. His height and weight are both less than the 5th percentile. Currently, his speech is at the 7- to 9-month level, motor skills at a 6-month level, and the rest of examination is not contributing. Routine laboratory test shows microcytic anemia with normal iron studies. What is the most probable cause of developmental delay in this child?

Lead

A 51-year-old immigrant from Vietnam presents with a 3-week history of nocturnal fever, sweats, cough, and weight loss. A chest radiograph reveals a right upper lobe cavitary infiltrate. A PPD produces 17 mm of induration, and acid-fast bacilli are present on a smear of induced sputum. While awaiting formal laboratory identification of the bacterium, which one of the following would be most appropriate? (check one) A. Observation only B. INH only C. INH and ethambutol (Myambutol) D. INH, ethambutol, and pyrazinamide E. INH, ethambutol, rifampin (Rifadin), and pyrazinamide

Leading authorities, including experts from the American Thoracic Society, CDC, and Infectious Diseases Society of America, mandate aggressive initial four-drug treatment when tuberculosis is suspected. Delays in diagnosis and treatment not only increase the possibility of disease transmission, but also lead to higher morbidity and mortality. Standard regimens including INH, ethambutol, rifampin, and pyrazinamide are recommended, although one regimen does not include pyrazinamide but extends coverage with the other antibiotics. Treatment regimens can be modified once culture results are available.

Most common cause of right heart failure in adults

Left heart failure

What are the atypical causes of CAP?

Leginella pneumophilia (WATER), Mycoplasma pneumonia, and chlamydophila pneumonia

A young woman presents with her daughter; the girl appears to be about 6 years old. You note that the child's eyes are downcast and she is scratching her scalp. On closer inspection, you note a tiny, white, wingless insect and tiny eggs attached to the hair shafts. The child's mother also notes that the same insects are present in the girl's eyelashes.

Lice

Treatment of Hashimoto's thyroiditis

Lifelong replacement with thyroid hormone

How do you differentiate transudative from exudative effusion?

Lights criteria: Plueral fluid: serum protein >0.5 Plueral fluid LDH :Serum LDH >0.6 Plueral Fluid LDH>2/3 upper normal serum LDH Transudative Protein <3 (Heart failure) Exudative Protein >3 (Pneumonia, Cancer, empyemas, hemothorax, chylothorax)

Of the following antidepressants, which one is LEAST likely to cause drug interactions? (check one) A. Citalopram (Celexa) B. Fluoxetine (Prozac) C. Paroxetine (Paxil) D. Mirtazapine (Remeron)

Like all drugs, SSRIs have significant side effects, including inhibition of the cytochrome P-450 system. However, citalopram is least likely to inhibit this system, making it a preferred SSRI for patients taking multiple medications for other illnesses

Treatment of Human Papillomavirus

Liquid nitrogen, salicylic acid, podophyllum, or topical interferon (imiguimod Aldaral). Surgical removal is possible.

HTN Tx w/ Diabetes

Lisinopril

Polyuria, tremors, seizures, and coma, other less common side effects include: rashes, drowsiness, blurred vision, diarrhea, polyuria, polydipsia, and a metallic taste are side effects of what drug?

Lithium

A 49-year-old female who takes multiple medications has a chemistry profile as part of her routine monitoring. She is found to have an elevated calcium level. All other values on the profile are normal, and the patient is not currently symptomatic. Follow-up testing reveals a serum calcium level of 11.2 mg/dL (N 8.4-10.2) and an intact parathyroid hormone level of 80 pg/mL (N 10-65). Which one of the following should be discontinued for 3 months before repeat laboratory evaluation and treatment? (check one) A. Lithium B. Furosemide (Lasix) C. Raloxifene (Evista) D. Calcium carbonate E. Vitamin D

Lithium therapy can elevate calcium levels by elevating parathyroid hormone secretion from the parathyroid gland. This duplicates the laboratory findings seen with mild primary hyperparathyroidism. If possible, lithium should be discontinued for 3 months before reevaluation (SOR C). This is most important for avoiding unnecessary parathyroid surgery. Vitamin D and calcium supplementation could contribute to hypercalcemia in rare instances, but they would not cause elevation of parathyroid hormone. Raloxifene has actually been shown to mildly reduce elevated calcium levels, and furosemide is used with saline infusions to lower significantly elevated calcium levels.

What class of antibiotics should be used for cat-scratch?

Macrolides

Head Lice Tx

Malathion, Permetherin, Pyrethrins

Treatment of testicular torsion

Manual detorsion (Twisting hte testes outwards and laterally. Surgery is not treated within 6 hours

How do you manage Bell's Palsy?

May resolve spontaneously. Administration of prednisone with or without acyclovir may have benifits

Treatment of hookworms

Mebendazole

Treatment of pinworms

Mebendazole

A previously healthy 3-year-old male is brought to your office with a 4-hour history of abdominal pain followed by vomiting. Just after arriving at your office he passes bloody stool. A physical examination reveals normal vital signs, and guarding and tenderness in the right lower quadrant. A rectal examination shows blood on the examining finger. Which one of the following is the most likely diagnosis? (check one) A. Appendicitis B. Viral gastroenteritis C. Midgut volvulus D. Meckels diverticulum E. Necrotizing enterocolitis

Meckels diverticulum is the most common congenital abnormality of the small intestine. It is prone to bleeding because it may contain heterotopic gastric mucosa. Abdominal pain, distention, and vomiting may develop if obstruction has occurred, and the presentation may mimic appendicitis. Children with appendicitis have right lower quadrant pain, abdominal tenderness, guarding, and vomiting, but not rectal bleeding. With acute viral gastroenteritis, vomiting usually precedes diarrhea (usually without blood) by several hours, and abdominal pain is typically mild and nonfocal with no localized tenderness. The incidence of midgut volvulus peaks during the first month of life, but it can present anytime in childhood. Volvulus may present in one of three ways: as a sudden onset of bilious vomiting and abdominal pain in the neonate; as a history of feeding problems with bilious vomiting that now appears to be due to bowel obstruction; or, less commonly, as a failure to thrive with severe feeding intolerance. Necrotizing enterocolitis is typically seen in the neonatal intensive-care unit, occurring in premature infants in their first few weeks of life. The infants are ill, and signs and symptoms include lethargy, irritability, decreased oral intake, abdominal distention, and bloody stools. A plain abdominal film showing pneumatosis intestinalis, caused by gas in the intestinal wall, is diagnostic of this disease.

After 22 hours of difficult labor, a 25-year-old primigravida delivers a full-term female neonate. Both baby and the placenta demonstrated green staining. The USG at 20 weeks showed normal development for age; major organs were identified and normal in size for the age. Amniotic fluid volume was normal. An alpha-fetoprotein level was in the normal range. What condition is most likely to explain the respiratory distress that develops in this neonate?

Meconium aspiration

A 23-year-old Hispanic female at 18 weeks' gestation presents with a 4-week history of a new facial rash. She has noticed worsening with sun exposure. Her past medical history and review of systems is normal. On examination, you note symmetric, hyperpigmented patches on her cheeks and upper lip. The remainder of her examination is normal. The most likely diagnosis is: (check one) A. Lupus erythematosus B. Pemphigoid gestationis (herpes gestationis) C. Melasma (chloasma) D. Prurigo gestationis

Melasma or chloasma is common in pregnancy, with approximately 70% of pregnant women affected. It is an acquired hypermelanosis of the face, with symmetric distribution usually on the cheeks, nose, eyebrows, chin, and/or upper lip. The pathogenesis is not known. UV sunscreen is important, as sun exposure worsens the condition. Melasma often resolves or improves post partum. Persistent melasma can be treated with hydroquinone cream, retinoic acid, and/or chemical peels performed post partum by a dermatologist. The facial rash of lupus is usually more erythematous, and lupus is relatively rare. Pemphigoid gestationis is a rare autoimmune disease with extremely pruritic, bullous skin lesions that usually spare the face. Prurigo gestationis involves pruritic papules on the extensor surfaces and is usually associated with significant excoriation by the uncomfortable patient.

What are the screening recommendations for osteoporosis?

Men > 70 and women > 65; Both > 50 with significant risk factors

Treatment of orchitis/epididymitis

Men younger than 35- Ceftriaxone+ doxycycline Men older than 35-Ciprofloxacin

Treatment of prostatitis

Men younger than 35- Ofloxacin or ceftriaxone Men older than 35-Fluoroquinolone or bactrim

Diabetes drug that increases insulin sensitivity

Metformin

Most effective drug for treatment of Polycystic Ovarian Syndrome?

Metformin

Which DM II medication reduces hepatic glucose production

Metformin

Which classes of diabetes drugs lead to weight loss?

Metformin and incretin mimetics (like exenatide)

Weight Loss causing Diabetes Drugs

Metformin, Exenatide

A 7-year-old boy presents with his parents for evaluation of behavior problems in school; the problems have been ongoing for over 6 months. The boy's teacher recommended he be evaluated. She reports that he tests at grade level, but he seems to make careless mistakes on schoolwork and has trouble maintaining attention to instruction; he does not finish his homework, and he often loses his homework, pencils, and books. The boy is seen often fidgeting at his desk; he blurts out answers and has difficulty waiting his turn. The parents agree that they have seen similar traits at home, such as forgetting to do daily activities and being easily distracted, for several years. They view him as a happy, bright boy who is very active. He has had normal vision and hearing screenings. The father reports that he was very similar as a child and still struggles with focus and concentration as an adult, but he never received any help. The parents deny any major changes in the family situation. They would like medical help to improve their son's performance in school. The boy has been seen regularly for his well-child exams and has always met milestones and had normal exams. Today, he is quickly moving about the exam room; he looks at a book for a few moments, and he then looks to the window for a short while before interrupting his parents. Assuming the parents agree to medication for their son, what medication is the most appropriate initial treatment?

Methylphenidate (ADHD)

What antihypertensive medication is contraindicated sinus bradycardia, 2nd or 3rd degree AV-block, overt heart failure, or cardiogenic shock.

Metoprolol

Treatment of trichomonas

Metronidazole 2 g dose

Treatment for Rosacea

Metronidazole and other Abx

A 6-week-old infant was the product of an uncomplicated pregnancy, labor, and delivery. She has been breast-feeding well with only the occasional spit-up. Stools have been seedy, yellow, and at the rate of 5-6 times per 24 hours. Over the last 24 hours, she has vomited 6-7 times. The last 2 have produced yellow-green material. She has not stooled in the last 24 hours. She is markedly lethargic and has a tense distended abdomen.

Midgut volvulus

A 28-year-old male recreational runner has a midshaft posteromedial tibial stress fracture. Although he can walk without pain, he cannot run without pain. The most appropriate treatment at this point includes which one of the following? (check one) A. A short leg walking cast B. A non-weight-bearing short leg cast C. A non-weight-bearing long leg cast D. An air stirrup leg brace (Aircast) E. Low-intensity ultrasonic pulse therapy

Midshaft posteromedial tibial stress fractures are common and are considered low risk. Management consists of relative rest from running and avoiding other activities that cause pain. Once usual daily activities are pain free, low-impact exercise can be initiated and followed by a gradual return to previous levels of running. A pneumatic stirrup leg brace has been found to be helpful during treatment (SOR C). Non-weight bearing is not necessary, as this patient can walk without pain. Casting is not recommended. Ultrasonic pulse therapy has helped fracture healing in some instances, but has not been shown to be beneficial in stress fractures.

Treatment of nummular dermatitis?

Moisturizers or topical steroids. Tar baths or UVB phototherapy

A 53-year-old male presents with a 3-month history of despondency, insomnia, and irritability with family and co-workers. During your interview you also discover that he is drinking heavily at times and has several firearms at home. He thinks his life is "useless," noting that he "would be better off dead." The most appropriate action at this time would be to: (check one) A. Prescribe an SSRI B. Arrange immediate hospitalization C. Have the patient agree to a suicide prevention contract D. Avoid direct questions regarding suicidal thoughts

More than 50% of suicides are associated with a major depressive episode and 25% are associated with a substance abuse disorder. Suicide rates increase with age and are higher among men. Increased suicide rates also occur in patients with significant medical illnesses. Because discussing suicidal ideation may relieve the patient's anxiety, the physician should directly ask depressed patients about any suicidal thoughts. There are no known reliable tools for assessing suicide risk, so the assessment is subjective. The initial management of suicidal ideation should establish safety, often by hospitalization. The suicide prevention contract is of unproven clinical and legal usefulness. Antidepressant medication has not been shown to reduce suicide rates, especially on a short-term basis

Centor Criteri and determining when to swab and stuff

Most episodes of pharyngitis are caused by viral rather than bacterial infections. The use of clinical decision rules for diagnosing group A β-hemolytic streptococcal pharyngitis improves quality of care while reducing unwarranted treatment and overall cost (SOR A). The original Centor score used four signs and symptoms to estimate the probability of acute streptococcal pharyngitis in adults with a sore throat, and was later modified by adding age as a fifth criterion. One point each is assigned for (1) absence of cough, (2) swollen, tender anterior cervical nodes, (3) temperature >38.0°C (100.4°F), and (4) a tonsillar exudate and swelling. One point is added for patients between the ages of 3 and 14 years, and a point is subtracted for patients over the age of 45. The cumulative score determines the likelihood of streptococcal pharyngitis and the need for antibiotics, and guides testing strategies. Patients with a score of zero or 1 are at very low risk for streptococcal pharyngitis and do not require testing or antibiotic therapy. Patients with a score of 2-3 should be tested using a rapid antigen test or throat culture, and a positive result warrants antibiotic therapy. Patients with a score of 4 or higher are at high risk for streptococcal pharyngitis, and empiric treatment may be considered. This patient's score is zero, and no testing or treatment is warranted.

Treatment of Autosomal Dominant Polycystic Kidney disease

No cure. Supportive treatment. Control HTN <130/80 mmHg

Treatment of lichen planus

Topical steroids and occulsive dressives. Severe localized lesions treated with topical tretinoin. Oral lesions treated with cyclosporine

Which arrhythmia is associated with high-dose methadone?

Torsades de pointes

A 36-year-old male presents with pain over the lumbar paraspinal muscles. He says the pain began suddenly while he was shoveling snow. Which one of the following is true regarding this patients injury? (check one) A. Systemic corticosteroids speed recovery B. Exercises specific to low back injuries speed recovery C. Opioids have significant advantages for symptom relief when compared with NSAIDs or acetaminophen D. Continued activity rather than bed rest helps speed recovery E. Trigger-point injections are superior to placebo in relieving acute back pain

Multiple studies have demonstrated that bed rest is detrimental to recovery from low back pain. Patients should be encouraged to remain as active as possible. Exercises designed specifically for the treatment of low back pain have not been shown to be helpful. Neither opioids nor trigger-point injections have shown superiority over placebo, NSAIDs, or acetaminophen in relieving acute back pain. There is no good evidence to suggest that systemic corticosteroids are effective for low back pain with or without sciatica

What is sarcoidosis and who gets it?

Multisystem disease of unknown etiology characterized by granulomatous inflammation seen in North American Blacks and northern European whites

What is a common side effect of HMG-CoA inhibitors (Statins)

Myositis, leg cramps

What is the antidote for acetaminophen toxicity?

N-acetylcysteine

How to treat A.Fib in a patient who is hyperthyroid

NOT ELECTRICALLY. Digoxin in large doses and beta blockers with caution. Warfarin may be used to prevent thrombi.

What pain medication drug class should be avoided in the elderly?

NSAIDs- increased risk for GI bleed and renal toxicity

Acute opioid overdose is treated with what medication?

Naloxone is an opioid receptor antagonist with a high affinity for opioid receptors. Naloxone displaces receptor-bound opioids, which leads to a reversal of opioid-induced respiratory depression and coma

Symptoms of miotic pupils and decreased pulse and blood pressure are associated with which overdose

Narcotic/opioid overdose

Which medication to treat dyslipidemia is associated with reduced long-term mortality and has optimal effect on lipids, but is poorly tolerated at full doses because of flushing?

Niacin

Which one of the following sleep problems in children is most likely to occur during the second half of the night? (check one) A. Confusional arousals B. Sleepwalking C. Sleep terrors D. Nightmares

Nightmares occur in the second half of the night, when rapid eye movement (REM) sleep is most prominent. Parasomnias, including sleepwalking, confusional arousal, and sleep terrors, are disorders of arousal from non-REM (NREM) sleep. These are more common in children than adults because children spend more time in deep NREM sleep. Such disorders usually occur within 1-2 hours after sleep onset, and coincide with the transition from the first period of slow-wave sleep.

A 5-year-old girl presents with a 5-day history of fever, coughing, sneezing, and watery nasal discharge. According to her mother, she passed out and began shaking yesterday. 1 hour before that, her temperature peaked at 40.1 °C (104.2 F). The episode lasted for 1.5 minutes, and she remained drowsy for about an hour afterwards. 1 year ago, she suffered a similar shaking episode during a bout of otitis media. The child reached all developmental milestones at appropriate ages, received all immunizations, and has no other relevant past medical or family history. Her only medication is acetaminophen. Vital signs are as follows: BP 110 / 70 mm Hg, HR 86 bpm, RR 20 rpm, and temperature 37.3 °C (99.1 F). On physical examination, there is some wheezing in both lung fields. The remainder of the examination, including a full neurologic exam, was normal. What is the most appropriate next step??

No further workup is needed.

Treatment of rabies

No specific treatment is available. Rabies vaccine immunoglobulin is given along with monoclonal antibodies, ribaviron, interferon-alpha, and ketamine.

Spermatocele treatment

No treatment necessary. Large spermatoceles can be surgically removed

When to do Thyroid Scan

Nodule + Decreased TSH

Contraindications to electroconvulsive therapy

None

Hypercalcemia Tx

Normal Saline + Lasix if in danger of fluid overload Can do Pamidronate after fluid is replenished

In response to a complaint by her kindergarten teacher, a mother brings her 4-year-old girl to her pediatrician. The child has been noticed spending time alone while at kindergarten, talking to her self and to imaginary friends. The conversations also occur at home. Her reading and writing are comparable to her classmates. At her bedside, she keeps a pillow and blanket for her companion. Her mother also reports that her child has frequent nightmares, sometimes waking her in the night in tears. The mother wants her child evaluated further.

Normal development

Treatment of hypokalemia without cardiac manifestations

Not an emergency. Oral potassium therapy is preferred with potassium chloride

Treatment of molluscum contagiosum

Not necessary- usually self-limitng. If treatment is necessary curettage is first line followed by cryotherapy, electrodessication, or an acid or exfoliative peel

You have just finished a health maintenance visit on a feisty 2-year-old brought in by his mother. He seems to be growing well, his weight and height are following the 50th percentile, he is meeting major motor and language milestones on time, and his mother has no concerns. On first entering the room, he is observed sitting quietly in a corner of the room on the floor looking at some picture books while you take a full history from his mother. When you try to pick him up to put on the examining table, however, he begins to throw himself backwards on the floor repeatedly, cries and screams loudly, and hits you and his mother. With some difficulty, you complete a cursory exam and ask about his behavior at home and any family stressors while he moves away from his mother and goes back to his corner to look at books. What observed behavior would most likely indicate a disorder of affective development?

Not seeking out his mother

An 82-year-old resident of a local nursing home is brought to your clinic with fever, difficulty breathing, and a cough productive of purulent sputum. The patient is found to have an oxygen saturation of 86% on room air and a chest radiograph shows a new infiltrate. A decision is made to hospitalize the patient. Which one of the following intravenous antibiotic regimens would be most appropriate for this patient? (check one) A. Levofloxacin (Levaquin) B. Ceftriaxone (Rocephin) and azithromycin (Zithromax) C. Ceftazidime (Fortaz, Tazicef) and levofloxacin D. Ceftazidime and vancomycin E. Ceftazidime, levofloxacin, and vancomycin

Nursing home-acquired pneumonia should be suspected in patients with a new infiltrate on a chest radiograph if it is associated with a fever, leukocytosis, purulent sputum, or hypoxia. Nursing-home patients who are hospitalized for pneumonia should be started on intravenous antimicrobial therapy, with empiric coverage for methicillin-resistant Staphylococcus aureus (MRSA) and Pseudomonas aeruginosa. The 2005 American Thoracic Society/Infectious Diseases Society of America guideline recommends combination therapy consisting of an antipseudomonal cephalosporin such as cefepime or ceftazidime, an antipseudomonal carbapenem such as imipenem or meropenem, or an extended-spectrum β-lactam/β-lactamase inhibitor such as piperacillin/tazobactam, PLUS an antipseudomonal fluoroquinolone such as levofloxacin or ciprofloxacin, or an aminoglycoside such as gentamicin, tobramycin, or amikacin, PLUS an anti-MRSA agent (vancomycin or linezolid). Ceftriaxone and azithromycin or levofloxacin alone would be reasonable treatment options for a patient with nursing home-acquired pneumonia who does not require hospitalization.

Hip Osteoarthritis Risk Factors

Obesity, Old Age, Hypothyroidism, Weight Bearing Sports, High Bone Mass

Femoral Anteversion Tx

Observation

A 9-month-old female infant presents with an acute onset of a rash on her trunk. She has a 4-day history of fever up to 104oF, but the mother states her daughter has no fever today. She has had some diarrheal stools, but no vomiting. No coughing or nasal congestion has been noted. The child has previously been well. Her past medical history is unremarkable, and she is up-to-date on her immunizations. She attends day care, and her mother notes that some children have been ill with non-specific febrile illnesses over the past 2 weeks. Her only medication has been ibuprofen for the fever. Physical exam shows a temperature of 98.8 F, pulse of 124 BPM, and respiratory rate of 28/min. She is alert and shows no other abnormalities. Her exam shows normal tympanic membranes in the ear and normal conjunctivae in the eye. The neck shows some shotty anterior cervical adenopathy; the throat shows slight erythema of the posterior pharynx, and the skin appears with an erythematous maculopapular rash most pronounced on the trunk. Her chest is clear to auscultation, heart rhythm is regular without murmurs, abdomen is soft and non-tender, and her neurological exam is normal. What is the most appropriate intervention for this patient? Draw measles titers of acute and convalescent sera Oral acyclovir Observe the child and reassure the parent Draw a complete blood count and blood cultures Treat presumptively with oral antibiotics

Observe the child and reassure the parent The clinical syndrome in this vignette is typical for roseola infantum, also known as exanthem subitum. Typically, the patient will have a moderate-to-high fever for 3 to 4 days, followed by defervescence and the outbreak of an erythematous macular or maculopapular rash that usually begins on the trunk, spreads to the arms and neck, and occasionally affects the face and legs. The rash is usually gone within 3 days. Occasional cervical lymphadenopathy is seen, as well as a slight pharyngeal inflammation and coryza. The patient can present with a bulging fontanelle and suffer from febrile convulsions during the high fever stage of the illness. Human Herpes virus 6 is the etiologic agent of the vast majority of cases. Once the rash has appeared and the child appears well, the only treatment is reassurance for the parents in regard to the benign nature of the condition.

A 24-year-old female at 36 weeks' gestation plans to breastfeed her infant. She has a history of bipolar disorder, but currently is doing well without medication, and also has a history of frequent urinary tract infections. She asks you about medications that she may need to take after delivery, and how they may affect her newborn. Which one of the following would be contraindicated if she breastfeeds her infant? (check one) A. Amoxicillin B. Macrodantin (Macrobid) C. Valproic acid (Depakote) D. Lithium

Of the drugs listed, the only maternal medication that affects the infant is lithium. Breastfed infants of women taking lithium can have blood lithium concentrations that are 30%-50% of therapeutic levels. Ref: Ressel G: AAP updates statement for transfer of drugs and other chemicals into breast milk.

A 24-year-old woman who is Gravida 4, Para 2, Abort 1 presents to the sonography department for a mid pregnancy examination. She is sure her LMP was 24 weeks ago but is considered LGA and has an elevated MSAFP. The sonographic examination reveals a female fetus with a femur length appropriate for 24 weeks, however the fetal head measures below 2 standard deviations. Facial examination reveals the presence of a cleft maxilla and the feet are bilaterally clubbed. The abdominal circumference is also small and echogenic dilated loops of bowel are seen in the amniotic fluid. Surface appearance of the bowel is smooth indicating a membrane. What condition is a differential diagnosis?

Omphalocele

The mother of an 8-year-old female is concerned about purple "warts" on her daughter's hands. The mother explains that the lesions started a few months ago on the right hand along the top of most of the knuckles and interphalangeal joints, and she has recently noticed them on the left hand. The child has no other complaints and the mother denies any unusual behaviors. A physical examination is unremarkable except for the slightly violaceous, flat-topped lesions the mother described. What is the most likely cause for this patient's finger lesions? (check one) A. Dermatomyositis B. Aggressive warts C. Rubbing/wringing of the hands D. Bulimia nervosa E. Child abuse

One of the most characteristic findings in dermatomyositis is Gottron's papules, which are flat-topped, sometimes violaceous papules that often occur on most, if not all, of the knuckles and interphalangeal joints.

Which one of the following seafood poisonings requires more than just supportive treatment? (check one) A. Ciguatera B. Neurotoxic shellfish C. Paralytic shellfish D. Scombroid fish

Only symptomatic treatment is indicated for ciguatera poisoning, as there is no specific treatment. The same is true for shellfish poisoning, although potential respiratory distress or failure must be kept in mind. Scombroid poisoning is a pseudoallergic condition resulting from consumption of improperly stored scombroid fish such as tuna, mackerel, wahoo, and bonito. Nonscombroid varieties such as mahi-mahi, amberjack, sardines, and herring can also cause this problem. The poisoning is due to high levels of histamine and saurine resulting from bacterial catabolism of histidine. Symptoms occur within minutes to hours, and include flushing of the skin, oral paresthesias, pruritus, urticaria, nausea, vomiting, diarrhea, vertigo, headache, bronchospasm, dysphagia, tachycardia, and hypotension. Therapy should be the same as for allergic reactions and anaphylaxis, and will usually lead to resolution of symptoms within several hours.

You are evaluating a 3-year-old African-American boy at a well-child visit. Prenatal and past medical histories are unremarkable. Growth and developmental are normal. Immunizations are up to date. Examination is normal. When inquiring about living arrangements, the child's mother tells you that they live in a 3rd story apartment of an older building that has exterior fire escapes accessible by the windows. What should you advise regarding the windows? She should have window bars spaced 8 inches part installed Operable window guards should be installed on all windows All windows should be screened. She should have all windows bricked over Fixed window guards should be installed on all windows

Operable window guards should be installed on all windows The leading cause of childhood injuries is a fall; they are rarely fatal. Childhood falls that are fatal usually occur from a height greater than 2 stories (approximately 22 feet). Falls from windows from the 2nd story or higher are still a serious problem in older buildings. Operable window guards should be placed on all windows so that exit by a child is prevented, but exit under adult supervision in case of emergency, such as fire, can be readily accomplished.

When can activated charcoal be used to treat acetaminophen overdose?

Oral activated charcoal adsorbs acetaminophen if administered within 4 hours of ingestion.

Treatment of serious or cystic acne

Oral antibiotics and topical preparations Erythromycin, tetracyclines, bactrim, clindamycin

Treatment for sarcoidosis?

Oral corticosteroids if: Constitutional symptoms, hypercalcemia, iritis, arthritis, CNS of cardiac involvement

Treatment of Addison's disease (chronic adrenocortical insufficiency)

Oral hydrocortisone or prednisone. DHEA can also be given in primary disease.

What routes of corticosteroids are to be used for COPD exacerbations?

Oral or systemic; NOT inhaled

What is the clinical presentation for ARDS?

Pink, frothy sputum, bilateral diffuse pulmonary infiltrates

What arrhythmias is magnesium IV used for?

Torsades de pointes and it can be used for rapid AFib as well

How do you treat CAP?

Outpatient-Doxycycline, macrolide, fluoroquinolone Inpatient: Ceftriaxone and azithromycin

First-line treatment for plantar fasciitis

Over-the-counter heal inserts

Basic therapy for COPD exacerbations?

Oxygen, bronchodilators, systemic steroids and antibiotics

Pros vs. Con of HRT

PROS - Decreased Osteoporosis, Colon CA risk, decreased hot flashes CONS - Increased embolus, breast CA

A 55-year-old overweight male presents with a complaint of pain in the left big toe. He recently started jogging 2 miles a day to try to lose weight, but has not changed his diet and says he drinks 4 cans of beer every night. The pain has developed gradually over the last 2 weeks and is worse after running. An examination shows a normal foot with tenderness and swelling of the medial plantar aspect of the left first metatarsophalangeal joint. Passive dorsiflexion of the toe causes pain in that area. Plantar flexion produces no discomfort, and no numbness can be appreciated. Which one of the following is the most likely diagnosis? (check one) A. Sesamoid fracture B. Gout C. Morton's neuroma D. Cellulitis

Pain involving the big toe is a common problem. The first metatarsophalangeal (MTP) joint has two sesamoid bones, and injuries to these bones account for 12% of big-toe injuries. Overuse, a sharp blow, and sudden dorsiflexion are the most common mechanisms of injury. Gout commonly involves the first MTP joint, but the onset is sudden, with warmth, redness, and swelling, and pain on movement of the joint is common. Morton's neuroma commonly occurs between the third and fourth toes, causes numbness involving the digital nerve in the area, and usually is caused by the nerve being pinched between metatarsal heads in the center of the foot. Cellulitis of the foot is common, and can result from inoculation through a subtle crack in the skin. However, there would be redness and swelling, and the process is usually more generalized. Sesamoiditis is often hard to differentiate from a true sesamoid fracture. Radiographs should be obtained, but at times they are nondiagnostic. Treatment, fortunately, is similar, unless the fracture is open or widely displaced. Limiting weight bearing and flexion to control discomfort is the first step. More complex treatments may be needed if the problem does not resolve in 4-6 weeks.

A 50-year-old female presents with right eye pain. On examination, you find no redness, but when you test her extraocular muscles she reports marked pain with eye movement. This finding suggests that her eye pain is caused by: (check one) A. an intracranial process B. an ocular condition C. a retinal problem D. an orbital problem E. an optic nerve problem

Pain with eye movement suggests an orbital condition. Orbital inflammation, infection, or tumor invasion can lead to such eye pain. Other findings suggestive of an orbital cause of eye pain include diplopia or proptosis. If an orbital lesion is suspected, imaging studies should be performed.

Primary Dysmenorrhea

Pain with menses without an identifiable cause. Tx with NSAIDs

A 3-year-old boy presents with a loud cough. 2 days ago, the boy developed a runny nose and irritability; the next day, he began to cough loudly and felt warm. His mother tells you that the cough sounded like a wounded animal or a dog barking. The child is diagnosed with croup, and humidification is prescribed. What organism is most likely responsible for the patient's croup? Group A streptococci Haemophilus influenzae type b Influenza virus Parainfluenza virus Respiratory syncytial virus (RSV)

Parainfluenza virus The correct answer is parainfluenza virus,the most common cause of croup. Croup is more formally known as acute laryngotracheobronchitis. It occurs most often in children 2 to 3 years of age, and it has a distinctive 'seal barking' sound. This is caused by the marked edema in the respiratory tract. The same edema may show up on an A-P neck film as subglottic edema, or the "hourglass sign." This sign is also referred to as the "steeple sign."

How do you treat Lambert-Eaton Syndrome?

Plamsapheresis and immunosuppressive therapy

How do you manage a patient with Guillain-Bare Syndrome?

Plamsapheresis or IV immunoglobulin in adults with cardiovascular instability and in children.

Which one of the following is most characteristic of patellofemoral pain syndrome in adolescent females? (check one) A. Posterior knee pain B. Pain exacerbated by walking on a flat surface C. Inadequate hip abductor strength D. A high rate of surgical intervention

Patellofemoral pain syndrome is a common overuse injury observed in adolescent girls. The condition is characterized by anterior knee pain associated with activity. The pain is exacerbated by going up or down stairs or running in hilly terrain. It is associated with inadequate hip abductor and core strength; therefore, a prescription for a rehabilitation program is recommended. Surgical intervention is rarely required.

Rough, machinery murmur that is best heard at the 2nd left intercostal space.

Patent ductus arteriosus (PDA)

A 30-year-old African-American female presents with a vaginal discharge. On examination the discharge is homogeneous with a pH of 5.5, a positive whiff test, and many clue cells. Which one of the following findings in this patient is most sensitive for the diagnosis of bacterial vaginosis? (check one) A. The pH of the discharge B. The presence of clue cells C. The character of the discharge D. The whiff test

Patients must have 3 of 4 Amsel criteria to be diagnosed with bacterial vaginosis. These include a pH >4.5 (most sensitive), clue cells >20% (most specific), a homogeneous discharge, and a positive whiff test (amine odor with addition of KOH)

When is surgery a preferred treatment option for hyperthyroidism?

Patients that are pregnant, have larger goiters, and when malignancy is likely.

A 25-year-old male presents to your office with a 1-week history of neck pain with radiation to the left hand, along with intermittent numbness and tingling in the left arm. His history is negative for injury, fever, or lower extremity symptoms. Extension and rotation of the neck to the left while pressing down on the head (Spurling's maneuver) exacerbates the symptoms. His examination is otherwise normal. Cervical radiographs are negative. Which one of the following would be most appropriate at this point? (check one) A. NSAIDs for pain relief B. A trial of tricyclic antidepressants C. Cervical corticosteroid injection D. Cervical MRI E. Referral to a spine subspecialist

Patients who present with acute cervical radiculopathy and normal radiographs can be treated conservatively. The vast majority of patients with cervical radiculopathy improve without surgery. Of the interventions listed, NSAIDs are the initial treatment of choice. Tricyclic antidepressants, as well as tramadol and venlafaxine, have been shown to help with chronic neuropathic pain. Cervical MRI is not indicated unless there are progressive neurologic defects or red flags such as fever or myelopathy. Likewise, referral to a subspecialist should be reserved for patients who have persistent pain after 6-8 weeks of conservative management and for those with signs of instability. Cervical corticosteroid injections have been found to be helpful in the management of cervical radiculopathy, but should not be administered before MRI is performed (SOR C)

Which one of the following is a risk factor for endometrial cancer? (check one) A. Polycystic ovary syndrome B. Multiparity C. Late menarche D. Use of an IUD E. Use of oral contraceptives

Patients with persistent hyperestrogenic states are at heightened risk for the development of endometrial cancer. The chronic anovulation and consequent hyperstimulation of the endometrium seen with polycystic ovary syndrome predispose women to endometrial hyperplasia and carcinoma. Conversely, multiparity and late menarche are protective of the endometrium. Combination oral contraceptive use seems to decrease the risk for endometrial cancer. There is no evidence that IUD use leads to endometrial cancer, and it is thought that copper-containing IUDs may in fact provide some protection against endometrial cancer.

Treatment of treponema pallidum

Penicillin G

Treatment for group A beta hemolytic streptococci?

Penicillina or cephalosporins. Macrolides if allergic

What drug is best for venous ulcers?

Pentoxifylline (PDE inhibitor acts as NSAID and vasodilator)

When added to compression therapy, which one of the following has been shown to be an effective adjunctive treatment for venous ulcers? A. Warfarin (Coumadin) B. Enoxaparin (Lovenox) C. Clopidogrel (Plavix) D. Pentoxifylline (Trental) E. Atorvastatin (Lipitor)

Pentoxifylline is effective when used with compression therapy for venous ulcers, and may be useful as monotherapy in patients unable to tolerate compression therapy. Aspirin has also been shown to be effective. Other treatments that have been studied but have not been found to be effective include oral zinc and antibiotics (SOR A).

What exam findings are seen with a pleural effusion?

Percussion dullness, decreased tactile fremitus, decreased of absent breath sounds over effusion.

A 74-year-old white male complains of pain in the right calf that recurs on a regular basis. He smokes 1 pack of cigarettes per day and is hypertensive. He has a history of a previous heart attack but is otherwise in fair health. Which one of the following findings would support a diagnostic impression of peripheral vascular disease? (check one) A. Pain during rest and exercise and the presence of swelling and soreness behind the knee and in the calf B. Pain that begins immediately upon walking and is unrelieved by rest C. Doppler waveform analysis showing accentuated waveforms at a point of decreased blood flow D. Treadmill arterial flow studies showing a 20-mm Hg decrease in ankle systolic blood pressure immediately following exercise E. An ankle-brachial index of 1.15

Peripheral vascular disease (PVD) is a clinical manifestation of atherosclerotic disease and is caused by occlusion of the arteries to the legs. Patients with significant arterial occlusive disease will have a prominent decrease in the ankle-brachial index from baseline following exercise, and usually a 20-mm Hg or greater decrease in systolic blood pressure. Pain during rest and exercise and the presence of swelling and soreness behind the knee and in the calf is found in those with Baker's cysts. Peripheral nerve pain commonly begins immediately upon walking and is unrelieved by rest. Doppler waveform analysis is useful in the diagnosis of PVD and will reveal attenuated waveforms at a point of decreased blood flow. Employment of the ankle-brachial index is encouraged in daily practice as a simple means to diagnose the presence of PVD. Generally, ankle-brachial indices in the range of 0.91-1.30 are thought to be normal. Ref: Hiatt WR: Medical treatment of peripheral arterial disease and claudication

A 25-year-old male who came to your office for a pre-employment physical examination is found to have 2+ protein on a dipstick urine test. You repeat the examination three times within the next month and results are still positive. Results of a 24-hour urine collection show protein excretion of <2 g/day and normal creatinine clearance. As part of his further evaluation you obtain split urine collections with a 16-hour daytime specimen containing an increased concentration of protein, and an 8-hour overnight specimen that is normal. Additional appropriate evaluation for this man's problem at this time includes which one of the following? (check one) A. Serum and urine protein electrophoresis B. Antinuclear antibody C. Serum albumin and lipid levels D. Renal ultrasonography E. No specific additional testing

Persons younger than 30 years of age who excrete less than 2 g of protein per day and who have a normal creatinine clearance should be tested for orthostatic proteinuria. This benign condition occurs in about 3%-5% of adolescents and young adults. It is characterized by increased protein excretion in the upright position, but normal protein excretion when the patient is supine. It is diagnosed using split urine collections as described in the question. The daytime specimen has an increased concentration of protein, while the nighttime specimen contains a normal concentration. Since this is a benign condition with normal renal function, no further evaluation is necessary.

How do you treat cerebral palsy?

Pharmacological treatment for spasticity and seizures is often required.

When prescribing an inhaled corticosteroid for control of asthma, the risk of oral candidiasis can be decreased by: (check one) A. using a valved holding chamber B. limiting use of the inhaled corticosteroid to once daily C. adding nasal fluticasone propionate (Flonase) D. adding montelukast (Singulair) E. adding salmeterol (Serevent)

Pharyngeal and laryngeal side effects of inhaled corticosteroids include sore throat, coughing on inhalation of the medication, a weak or hoarse voice, and oral candidiasis. Rinsing the mouth after each administration of the medication and using a valved holding chamber when it is delivered with a metered-dose inhaler can minimize the risk of oral candidiasis.

What medication can be given to provide urinary analgesia

Phenazopyridine. Will discolor the urine dark orange or brown.

What medications will help with shooting or stabbing neuropathic pain?

Phenytoin, mexiletine, or carbamazepine

How do you diagnose a PE?

Physical Exam, Well's prediction rules, thrombophilia work up, D-dimer, ABG's, ECG, CXR, Spiral CT, CT angiography, V/Q scans. Gold standard Pulmonary angiography

How do you diagnose a DVT?

Physical Exam, Well's prediction rules, thrombophilia work up, D-dimer, compression venous US, venography

You see a 17-year-old white female who has recently become sexually active. She requests oral contraceptives and you perform a brief evaluation, including blood pressure measurement. A pregnancy test is negative. She is resistant to further evaluation unless it is necessary. In addition to appropriate counseling, which one of the following should be done before prescribing oral contraceptives? (check one) A. No further evaluation at this visit unless indicated by history B. A pelvic examination and Papanicolaou test C. Screening for sexually transmitted diseases D. A breast examination

Policy statements from major organizations based on reviews of relevant medical literature support the practice of prescribing initial hormonal contraception after performing only a careful review of the medical history plus measurement of blood pressure. Requiring that patients undergo pelvic and breast examinations leads many young women to avoid this most reliable method of contraception, resulting in a much higher rate of unwanted pregnancy. Follow-up blood pressure measurements are important. Often, younger women will be willing to undergo Papanicolaou (Pap) tests and STD screening later, and periodic follow-up must be scheduled. Sexually active adolescents should have annual screening for cervical cancer and sexually transmitted diseases, but these are not necessary before prescribing oral contraceptives. The longest period of time a prescription should be given without a Pap test is 1 year, but this restriction is under study. Obviously, any history indicative of high risk would modify this approach.

A 62-year-old white male complains of fatigue and proximal extremity discomfort without any localized joint pain. Which one of the following conditions is associated with a consistently normal creatine kinase enzyme level at all phases of disease? (check one) A. Polymyalgia rheumatica B. Polymyositis C. Dermatomyositis D. Drug-induced myopathy E. Hypothyroid endocrinopathy

Polymyalgia rheumatica is a disease of the middle-aged and elderly. Discomfort is common in the neck, shoulders, and hip girdle areas. There is an absence of objective joint swelling, and findings tend to be symmetric. Characteristically, the erythrocyte sedimentation rate and C-reactive protein levels are significantly elevated; however, these tests are nonspecific. Occasionally there are mild elevations of liver enzymes, but muscle enzymes, including creatine kinase, are not elevated in this disorder. Elevation of muscle enzymes strongly suggests another diagnosis. Polymyositis and dermatomyositis are associated with variable levels of muscle enzyme elevations during the active phases of the disease. Drug-induced myopathies such as those seen with the cholesterol-lowering statin medications tend to produce some elevation of muscle enzymes during the course of the disorder. Hypothyroidism is associated with creatine kinase elevation. It should be strongly considered in the patient with unexplained, otherwise asymptomatic creatine kinase elevation found on a routine chemistry profile. Hyperthyroidism may cause muscle disease and loss of muscle, but it is not associated with creatine kinase elevation.

You evaluate a 13-year-old boy for sexual infantilism. He has a history of developmental delay, and he has been classified as moderately intellectually disabled. The parents note that he has always appeared quite "floppy"; walking was delayed. Although he had feeding difficulties as an infant, his appetite now seems insatiable. Family history is unremarkable. Immunizations are current. Vital signs are normal. Height is 4 feet 9 ½ inches (10th percentile for age); weight is 176 pounds (> 98th percentile), and BMI is 37.4 (> 99th percentile). The bifrontal diameter is narrow, with almond shaped, up-slanting palpebral fissures. Genitalia are Tanner stage I, with a small penis, hypoplastic scrotum, and testes measuring 1.5 cc. bilaterally. Hands and feet are small. In light of the above findings, to what do you attribute this boy's obesity? Klinefelter syndrome Exogenous obesity Cushing syndrome Prader-Willi syndrome Down syndrome

Prader-Willi syndrome It occurs equally in male and female patients; it features hypotonia and feeding problems in infancy, hypogonadism, onset of satiety disorder with hyperphagia and obesity in childhood, cognitive impairment, and a variety of dysmorphic features. It is caused by a variety of abnormalities in the chromosome 15q11-q13 region, including microdeletion of the paternally derived chromosome number fifteen (70%), uniparental disomy for chromosome number 15 (with both chromosomes number 15 being maternally derived) (25%), imprinting errors for the 15q11-q13 region (5%) and balanced translocations involving the 15q11-q13 region (rare). The 15q11-q13 region contains genes that are normally imprinted, meaning that they are differentially expressed, depending upon whether they are paternally or maternally derived. Absence of the paternally expressed genes in this region leads to the findings of Prader-Willi syndrome.

A 14-month-old boy is brought to the well-baby clinic by his mother. He was born at 30 weeks of gestation and was hospitalized for the first 2 weeks of life for pulmonary hypoplasia. His mother is worried that he seems to be attaining his developmental milestones 'very slowly' compared to his older brother. He has just started to walk unassisted, says 'mama and dada' and gets upset if she leaves the room. He is unable to turn the pages of a book, and is unable to use any other words. She wonders if this could have anything to do with his pre-term delivery. What is the most appropriate next step in counseling the mother??

Pre-term birth results in milestones that are expected to be delayed

Treatment of lichen simplex chronicus

Preventing hte itch-scratch cycle. Occlusive dressings with or without topical steroids. Antihistamines

First line testing for late-onset male hypogonadism

Total testosterone (Free is expensive and should only be used if total is borderline and there is a suspected issue in sex hormone-binding globulin)

Which one of the following community health programs best fits the definition of secondary prevention? (check one) A. An antismoking education program at a local middle school B. Blood pressure screening at a local church C. A condom distribution program D. Screening diabetic patients for microalbuminuria

Prevention traditionally has been divided into three categories: primary, secondary, and tertiary. Primary prevention targets individuals who may be at risk to develop a medical condition and intervenes to prevent the onset of that condition (e.g., childhood vaccination programs, water fluoridation, antismoking programs, and education about safe sex). Secondary prevention targets individuals who have developed an asymptomatic disease and institutes treatment to prevent complications (e.g., routine Papanicolaou tests; screening for hypertension, diabetes, or hyperlipidemia). Tertiary prevention targets individuals with a known disease, with the goal of limiting or preventing future complications (e.g., screening diabetics for microalbuminuria, rigorous treatment of diabetes mellitus, and post-myocardial infarction prophylaxis with β-blockers and aspirin).

How do you prevent CAP?

Prevnar 13 and pneumovax

What are the classifications for a spontaneous pneumothroax?

Primary-Absence of underlying disease-Tall, skinny young males that smoke (Michael Phelps) Secondary-Complication of underlying disease-Asthma, COPD, CF, ILD, bleb

Silicosis is what?

Prolonged inhalation of silica caused by formation of small rounded opacities throughout the lungs. 30% higher incidence of TB

How do you treat Paget's Disease

Prompt cyclic administration of bisphosphonates (alendronate, tiludronate, risedronate, zoledronic acid or pamidronate) or Nasal calcitonin-salmon

Treatment of a TIA?

Prophylactic antiplatelet therapy when the TIA is not cardiogenic. Cardiogenic TIA requires anticoagulation

Treatment of hyperthyroidism (thyrotoxicosis) during pregnancy

Propylthiouracil

What is palmetto used for?

Prostatism

Where does a scaphoid fracture occur that it must be referred to ortho?

Proximal 1/3

An 8-year-old boy sustained a puncture wound to his right foot 4 days ago. He was playing and stepped on a nail that went through his sneaker. His mother said the wound bled profusely but the nail did not go completely through his foot. They washed the wound at home with soap and water, wrapped it in a bandage, and did not seek further care. This morning, he complained that it was very painful and his mother noted that his foot looked red and swollen. On exam, his temperature is 99°F; pulse is 114 BPM, and his BP is 104/68 mm Hg. The plantar surface of his right foot has a small 2 mm scabbed entry wound that is surrounded by a 5 - 6 cm area that is erythematous, swollen, and quite tender. There is a scant amount of thin, seropurulent material from the entry wound on examination. You are worried about possible osteochondritis. What bacteria would most likely cause this complication? Streptococci Eikenella Pasturella Clostridium Pseudomonas

Pseudomonas Pseudomonas aeruginosa causes more than 90% of foot osteochondritis in plantar puncture wounds. It should be considered as a likely pathogen when there is a penetrating injury through footwear, especially sneakers or from soil and manure contamination. It is believed that the penetrating object may push particles of foam contaminated with bacteria into the wound. In addition, anaerobes, Klebsiella, Bacteroides, Serratia, and Salmonella sp. are less common organisms in this setting that may also cause secondary infections.

Most common cause of infective endocarditis (IE) in IV drug user

Pseudomonas aeruginosa, Staphylococcus aureus, Candida albicans, viridans streptococci, and enterococcus

How does active TB present?

Pulmonary Presentation 95% of the time-fever, night sweat, anorexia, weight loss, cough, pleuritic chest pain, SOB, hemoptysis Non-pulmonary presentation 5% of the time-Lymphadenitis (Scrofula) -Anterior cervical lymph nodes, or skeletal TB (Potts disease)

Kerley B lines are usually seen with what diseases?

Pulmonary edema and congestive heart failure.

A 50-year-old male has a pre-employment chest radiograph showing a pulmonary nodule. There are no previous studies available. Which one of the following would raise the most suspicion that this is a malignant lesion if found on the radiograph? . (check one) A. The absence of calcification B. Location above the midline of the lung C. A diameter of 4 mm D. A solid appearance

Pulmonary nodules are a common finding on routine studies, including plain chest radiographs, and require evaluation. Radiographic features of benign nodules include a diameter <5 mm, a smooth border, a solid appearance, concentric calcification, and a doubling time of less than 1 month or more than 1 year. Features of malignant nodules include a size >10 mm, an irregular border, a "ground glass" appearance, either no calcification or an eccentric calcification, and a doubling time of 1 month to 1 year (SOR B).

What medication is given in response to Isoniazid (INH) seizure control?

Pyridoxine (Vitamin B6)

Second Generation Anti-Pscyhotic Side Fx

QT Prolongation

A 79-year-old male has psychosis secondary to dementia associated with Parkinson's disease. After exhausting all other options you decide to prescribe an antipsychotic agent. Which one of the following would be the best choice in this situation? (check one) A. Haloperidol B. Olanzapine (Zyprexa) C. Risperidone (Risperdal) D. Quetiapine (Seroquel) E. Thioridazine

Quetiapine is an atypical antipsychotic that has no clinically significant effect on the dopamine D2 receptor, which is responsible for the parkinsonian side effects of antipsychotic medications. Because of this, it is considered the antipsychotic of choice in patients with dementia associated with Parkinson's disease, although its use has not been studied extensively in this clinical situation. The other atypical antipsychotics listed, olanzapine and risperidone, have some D2 receptor effect. Thioridazine and haloperidol are typical antipsychotics and have more side effects, including parkinsonian side effects; they are not recommended in this clinical situation.

A 24-year-old Caucasian woman, gravida 2, para 2, presents in a distressed state. She has brought her 5-day-old daughter with her, and she reports that the child started to bleed spontaneously from her vagina. The pregnancy was without any complications; labor was at term and occurred spontaneously without any problems. Examination of the child reveals an afebrile child with blood at the introitus. The child does not appear to be in pain, and he sleeps through the examination. What is the next step in management?

Reassure the mother and send her home

A 6-year-old boy is performing poorly in school. According to his teacher, he can perform some schoolwork and answer test questions, but he has difficulty understanding the instructions. He has to ask for detailed help regarding instructions, and his teacher sometimes has to demonstrate it to him with action so that he can proceed with his lessons. He is the 2nd among 3 siblings, and he is in Kindergarten at the present time. Except for language skills, developmental milestones are appropriate for his age. At 15 months, he could not respond to simple instructions like "no" and "give me". At the time, his mother thought, "He's just a little different than the other siblings." He has had difficulty with school ever since he started. What is the most likely diagnosis?

Receptive language dysfunction

A 3-day-old male neonate is seen in the nursery due to failure to pass meconium and 2 episodes of vomiting. Prenatal and perinatal histories are unremarkable. Family history reveals a brother and sister with severe asthma. Examination reveals a moderately distended abdomen without signs of tenderness. A barium enema reveals meconium ileus with distal narrowing and proximal dilatation of the colon. Meconium is passed during the procedure, relieving the distention. What study/studies should be ordered? Chest radiograph Chromosome analysis of peripheral blood lymphocytes Qualitative stool fat Rectal manometry, rectal biopsy, sweat chloride Urine electrolytes

Rectal manometry, rectal biopsy, sweat chloride Meconium ileus is associated with both Hirschsprung's disease (colonic aganglionosis) and cystic fibrosis. Rectal manometry and biopsy will provide the most information regarding the former, and sweat chloride analysis is the standard for the initial diagnosis of cystic fibrosis.

Treatment of acne rosacea

Reduce triggers such as alcohol or hot beverages. Topical metronidazole if this fails then systemic antibiotics (tetracycline). Severe cases oral isotretinoin

Treatment of paraphimosis

Reduced EMERGENTLY. Manuel reduction-Firmly squeeze the glans for 5 minues to reduce the tissue edema and decrease the size of the glans then try to bring the foreskin back over it. If not successful-Surgery!

What does treating H. pylori actually do?

Reduces risk of bleeding, not cancer

How does leginella pneumophilia CAP present?

Relative bradycardia, high fever, hyponatremia diarrhea, nothing impressive on X-Ray, and the patient gets better faster and then worsens.

IBS Symptoms

Relief with defecation, diarrhea and constipation, passage of mucous, abdominal bloating

An 8-year-old male is brought to the emergency department with an acute asthma attack that began 48 hours earlier. His mother initiated his asthma action plan when the attack began, starting oral prednisolone plus albuterol (Proventil, Ventolin) by metered-dose inhaler with a spacer every 3-4 hours. In the emergency department the child is alert, with a respiratory rate of 30 beats/min and an oxygen saturation of 94% on room air. He is audibly wheezing. Peak flow is 40% of the predicted value. (check one) A. Continue the current albuterol treatment but switch to a nebulizer B. Administer high-dose albuterol via nebulizer every 20 minutes for 1 hour C. Administer intravenous corticosteroids within the first hour D. Administer magnesium sulfate intravenously E. Prescribe high-dose mucolytics and chest physiotherapy

Repeated doses of a short-acting β2-agonist and correction of hypoxia are the main elements of initial emergency department treatment for acute asthma exacerbations in children. Nebulizer treatments are no better than a metered-dose inhaler with a spacer (SOR A). High-dose nebulized albuterol every 20 minutes for 1 hour has not been shown to be beneficial. In children already receiving standard treatment with albuterol and corticosteroids the addition of intravenous magnesium sulfate has been shown to improve lung function and reduce the need for hospitalization (SOR A). Oral administration of corticosteroids is as effective as the intravenous route for reducing the need for hospital admission (SOR A). Mucolytics and chest physiotherapy have not been shown to be effective in children with acute asthma attacks.

A 5-month-old infant was found cold and blue by his mother when she went to feed the baby. The infant was born prematurely at 34 weeks of gestation with a low birth weight of 4 pounds. He was the third child of a 20-year-old female. The child was developing normally with no major illnesses. What would be the likely cause?

SIDS

Bulimina Tx

SSRIs

OCD Tx

SSRIs

You see a 68-year-old mechanic for a routine evaluation. He has a 2-year history of hypertension. His weight is normal and he adheres to his medication regimen. His current medications are metoprolol (Lopressor), 100 mg twice daily; olmesartan (Benicar), 40 mg/day; and hydrochlorothiazide, 25 mg/day. His serum glucose levels have always been normal, but his lipid levels are elevated. A physical examination is unremarkable except for an enlarged prostate and a blood pressure of 150/94 mm Hg. Laboratory studies show a serum creatinine level of 1.6 mg/dL (N 0.6-1.5) and a serum potassium level of 4.9 mmol/L (N 3.5-5.0). The patient's record shows blood pressures ranging from 145/80 mm Hg to 148/96 mm Hg over the past year. Which one of the following would be most appropriate at this point? (check one) A. Continue his current management with no changes B. Substitute furosemide (Lasix) for hydrochlorothiazide C. Add clonidine (Catapres) D. Add spironolactone (Aldactone) E. Add hydralazine (Apresoline)

Resistant or refractory hypertension is defined as a blood pressure ≥140/90 mm Hg, or ≥130/80 mm Hg in patients with diabetes mellitus or renal disease (i.e., with a creatinine level >1.5 mg/dL or urinary protein excretion >300 mg over 24 hours), despite adherence to treatment with full doses of at least three antihypertensive medications, including a diuretic. JNC 7 guidelines suggest adding a loop diuretic if serum creatinine is >1.5 mg/dL in patients with resistant hypertension.

A newborn infant is delivered at 30 weeks since the last menstrual period. At birth, the infant weighs 1,500gm, but otherwise appears normal. Soon after birth, the infant becomes cyanotic and breathes with a grunting noise. Chest X-rays reveal dense lungs with significant atelectasis but no cardiovascular abnormalities.

Respiratory distress syndrome

Treatment for posteromedial tibial stress fracture

Rest and air stirrup leg brace

Treatment of influenza

Rest, analgesics, cough suppressant, Tamifu or Relenza

The mother of a 2-year-old boy came to the outpatient clinic concerned about deviation of the right eye of her child. She has also noticed that the pupil is white. She said she noticed these features a few weeks ago and that it is stationary. He was born at 40 weeks of gestation, had up-to-date vaccination, and never complained of anything regarding his eyes, which were normal. What condition is the primary suspect to exclude in diagnostic workup?

Retinoblastoma

What is Cor Pulmonale?

Right ventricle enlargement due to abnormalities of lung thorax, or pulmonary ventilation/circulation

Which one of the following side effects induced by traditional neuroleptic agents responds to treatment with beta-blockers? (check one) A. Akathisia B. Rigidity C. Dystonia D. Sialorrhea E. Stooped posture

Rigidity, sialorrhea, and stooped posture are parkinsonian side effects of neuroleptic drugs. These are treated with anticholinergic drugs such as benztropine or amantadine. Dystonia, often manifested as an acute spasm of the muscles of the head and neck, also responds to anticholinergics. Akathisia (motor restlessness and an inability to sit still) can be treated with either anticholinergic drugs or beta-blockers.

An 18-month-old infant is brought to your clinic office with a 5-day history of fever of 104°F. On physical examination, you note a mildly lethargic and irritable infant. There are no other clinically significant findings. You prescribe Children's Tylenol (acetaminophen) and tell the mother to monitor the infant's fever for the next few days; if the fever goes down, everything should be fine. The mother calls the next day and says that the fever has stopped, but a rash has developed, and she is concerned. The infant examination reveals a diffuse, fine, maculopapular rash. Presently, the child does not appear ill. What is the most likely diagnosis? Rubella Rubeola Erythema infectiosum Roseola Chicken pox

Roseola The clinical picture is suggestive of roseola. Roseola is a benign illness in humans caused by the human herpes virus 6 or 7. It is the major cause of acute febrile illness in young children. The most prominent feature is a fever lasting up to 8 days; the fever can be as high as 104° F. After the fever subsides, a rose-pink maculopapular rash appears.

A 13-month-old boy presents with a rash. The mother tells you that the child has had high fevers over the past 4 days, although he has not had a fever for the past 24 hours. The rash began 6 hours ago; it started on his chest and back, and it spread to his neck, face, and arms. The child doesn't appear to be itchy, and he has been acting normally since the fever subsided. The mother denies cough, runny nose, vomiting, and diarrhea. The only medication that the child has taken is acetaminophen. On examination, the child is happy and playful. The only physical finding is the rash that is shown in the image. What is the most likely diagnosis? Rubella Roseola Measles Drug hypersensitivity Varicella

Roseola This child exhibits the typical rash of roseola. Roseola infantum (also called exanthem subitum) is caused by Human Herpesvirus-6 and occurs almost exclusively during infancy. There is usually no prodromal period. The illness begins with high temperatures (averaging 103 F) for 3 to 5 days; the fever typically resolves rather abruptly. The rash appears within 12 to 24 hours of the fever resolution. The rash of roseola is rose-colored and begins as discrete, small, slightly raised pink lesions on the trunk and spreads to the neck, face and proximal extremities. The rash is not pruritic and no vesicles or pustules develop. The lesions may become confluent. Roseola is self-limited and the treatment is supportive only (e.g., antipyretics during the febrile phase).

In the winter, an 11-month-old infant presents with a 2-day history of vomiting, diarrhea, and fever. His temperature is 104° Fahrenheit. Clinically, he is significantly dehydrated; his white blood cell count is 5400 cells/mm3 with a normal differential. His stool and urine are negative for white blood cells. What is the most likely cause of gastroenteritis in this child?

Rotavirus

In the winter, an 11-month-old infant presents with a 2-day history of vomiting, diarrhea, and fever. His temperature is 104° Fahrenheit. Clinically, he is significantly dehydrated; his white blood cell count is 5400 cells/mm3 with a normal differential. His stool and urine are negative for white blood cells. What is the most likely cause of gastroenteritis in this child? Escherichia coli Clostridium difficile Norwalk virus Rotavirus Shigella

Rotavirus A viral etiology is suspected in a patient with a normal white blood cell count. In the 1st 2 years of life, especially in the winter months, rotavirus is a common cause of gastroenteritis. Epidemics are not uncommon; therefore, rotavirus vaccinations are now considered standard at 2, 4, and 6 months of age.

What is the safe therapeutic range of lithium??

Safe range of 0.5-1.2 mmoles/l.

You are evaluating a 7-year-old boy for a pruritic rash. It began on the hands 10 days ago; it has spread to the arms, trunk, and genitalia. Pruritus has been intense, especially at night. A younger sibling has a similar rash. Past medical history is unremarkable. Growth and development are normal, and immunizations are current. Vital signs are normal. There are papules, vesicles, and pustules; some in linear array (especially at the interdigital web spaces), with multiple excoriations of the interdigital spaces of the fingers, palms, soles, flexion creases of the elbows, trunk, and genitalia. What is the most likely causative agent of this rash? Borrelia burgdorferi Sarcoptes scabiei Pox virus Parvovirus B19 Malassezia furfur

Sarcoptes scabiei The presentation is that of scabies, which is caused by the human itch mite, Sarcoptes scabiei, an ectoparasiteacquired by humans from close physical contact.The pregnant female mite burrows under the skin, depositing eggs and excrement. 3 to 10 days later, the human host develops a hypersensitivity reaction to the mite, ova and excrement, at which time intense pruritus (especially nocturnal) begins. Burrows (linear tracks of the progression of the mite) are typical, but papules, vesicles and pustules are also present. Lesions are present at the scalp, face, extremities (including the palms and soles and characteristically featuring burrows at the web spaces of the fingers), chest, back, abdomen and genitalia; however, lesions are usually not present above the neck in patients older than 5 years.

Failure to enjoy or desire close relationships, a preference for solitary activities, a lack of close friends, very little emotion, and generally flat affect describes what personality disorder?

Schizoid

An 8-month-old male infant presents with rashes over the scalp and eyebrows. Physical examination shows a dry, scaly, and crusting lesion over the scalp, eyebrows, and nape area. He is comfortable, so his mother presumed that it was not itchy at all. He has been breastfed up to the present, and he started solid food at about 5 months old. There are no other signs and symptoms noted. Bowel movement and urination are normal. Developmental milestones are consistent with age. What is the most likely diagnosis?

Seborrheic dermatitis

An 8-month-old male infant presents with rashes over the scalp and eyebrows. Physical examination shows a dry, scaly, and crusting lesion over the scalp, eyebrows, and nape area. He is comfortable, so his mother presumed that it was not itchy at all. He has been breastfed up to the present, and he started solid food at about 5 months old. There are no other signs and symptoms noted. Bowel movement and urination are normal. Developmental milestones are consistent with age. What is the most likely diagnosis? Atopic dermatitis Seborrheic dermatitis Psoriasis Candidiosis Lichen simplex chronicus

Seborrheic dermatitis The clinical picture is suggestive of seborrheic dermatitis, which is a chronic inflammatory disease common in the pediatric age group especially during infancy and adolescent. The etiology is unknown, but it has been attributed to stressful situations, poor hygiene, and excessive perspiration.

Osteoporotic bone loss can be caused or accelerated by prolonged use of which one of the following medications? (check one) A. Hydrochlorothiazide B. Phenytoin C. Raloxifene (Evista) D. Diazepam (Valium) E. Fluoxetine (Prozac)

Secondary osteoporosis can result from a variety of endocrine, nutritional, or genetic disorders, as well as from prolonged use of certain medications. Anticonvulsants such as phenytoin increase the hepatic metabolism of vitamin D, thereby reducing intestinal calcium absorption. Other medications that adversely affect bone mineral density include glucocorticoids, cyclosporine, phenobarbital, and heparin. Thiazide diuretics reduce urinary calcium loss and are believed to preserve bone density with long-term use. Benzodiazepines and SSRIs have not been associated with increases in bone loss or in hip fractures. Raloxifene, a selective estrogen receptor modulator, is indicated for the prevention and treatment of osteoporosis in postmenopausal women.

In the secondary prevention of ischemic cardiac events, which one of the following is most likely to be beneficial in a 68-year-old female with known coronary artery disease and preserved left ventricular function? (check one) A. ACE inhibitors B. Hormone therapy C. Calcium channel blockers D. Vitamin E E. Oral glycoprotein IIb/IIIa receptor inhibitors

Secondary prevention of cardiac events consists of long-term treatment to prevent recurrent cardiac morbidity and mortality in patients who have either already had an acute myocardial infarction or are at high risk because of severe coronary artery stenosis, angina, or prior coronary surgical procedures. Effective treatments include aspirin, β-blockers after myocardial infarction, ACE inhibitors in patients at high risk after myocardial infarction, angiotensin II receptor blockers in those with coronary artery disease, and amiodarone in patients who have had a myocardial infarction and have a high risk of death from cardiac arrhythmias. Oral glycoprotein IIb/IIIa receptor inhibitors appear to increase the risk of mortality when compared with aspirin. Calcium channel blockers, class I anti-arrhythmic agents, and sotalol all appear to increase mortality compared with placebo in patients who have had a myocardial infarction. Contrary to decades of large observational studies, multiple randomized, controlled trials show no cardiac benefit from hormone therapy in postmenopausal women.

What is the most common cause of ARDS?

Sepsis-Acute hypoxemic respiratory failure after a systemic pr pulmonary insult without hear failure.

What are the first-line agents for the treatment of panic disorder

Serotonin selective reuptake inhibitors (SSRIs)

What is a Hypertensive emergency?

Severely elevated blood pressure (>180/120 mm Hg) accompanied by signs of progressive organ dysfunction.

Which one of the following is recommended in the treatment of all four stages of COPD, from mild through very severe? (check one) A. Scheduled oral mucolytics B. Scheduled inhaled corticosteroids such as fluticasone (Flovent HFA) C. Scheduled long-acting inhaled bronchodilators such as salmeterol (Serevent) D. Scheduled long-acting anticholinergics such as tiotropium (Spiriva) E. Short-acting inhaled ß2 -agonists such as albuterol (Ventolin HFA), as needed for dyspnea

Short-acting bronchodilators such as albuterol and ipratropium are recommended on an as-needed basis for treatment of breathlessness in stage I (mild) COPD. They are also recommended for as-needed use in stage II (moderate), stage III (severe), and stage IV (very severe) COPD. Long-acting bronchodilators such as salmeterol or tiotropium are recommended for stages II, III, and IV. Inhaled corticosteroids are recommended for stages III and IV. Mucolytics can be considered for stages III and IV.

Postexposure prevention of HIV

Should be started in the first 72 hours. Antiretrovirals for 4 weeks.

When alprazolam is stopped abruptly what are the side effects?

Signs and symptoms of withdrawal, such as sweating, tremors, and insomnia

Metformin (Glucophage) should be stopped prior to which one of the following, and withheld until 48 hours after completion of the test? (check one) A. An upper GI series B. Abdominal ultrasonography C. CT angiography D. MRI of the brain E. Colonoscopy

Since even a temporary reduction in renal function, such as occurs after pyelography or angiography, can cause lactic acidosis in patients taking metformin, the drug should be discontinued 48 hours before such procedures (SOR C) and restarted 48 hours after the procedure if renal function is normal. The other procedures listed are not indications for stopping metformin.

A 75-year-old Hispanic male presents with dyspnea on exertion which has worsened over the last several months. He denies chest pain and syncope, and was fairly active until the shortness of breath slowed him down recently. You hear a grade 3/6 systolic ejection murmur at the right upper sternal border which radiates into the neck. Echocardiography reveals aortic stenosis, with a mean transvalvular gradient of 55 mm Hg and a calculated valve area of 0.6 cm2. Left ventricular function is normal. Which one of the following is appropriate management for this patient? (check one) A. Aortic valve replacement B. Aortic balloon valvotomy C. Medical management with beta-blockers and nitrates D. Watchful waiting until the gradient is severe enough for treatment E. Deferring the decision pending results of an exercise stress test

Since this patient's mean aortic-valve gradient exceeds 50 mm Hg and the aortic-valve area is not larger than 1 cm2, it is likely that his symptoms are due to aortic stenosis. As patients with symptomatic aortic stenosis have a dismal prognosis without treatment, prompt correction of his mechanical obstruction with aortic valve replacement is indicated. Medical management is not effective, and balloon valvotomy only temporarily relieves the symptoms and does not prolong survival. Patients who present with dyspnea have only a 50% chance of being alive in 2 years unless the valve is promptly replaced. Exercise testing is unwarranted and dangerous in patients with symptomatic aortic stenosis.

Croup Tx

Single dose of steroids

Most common hip disorder in boys 8-15

Slipped capital femoral epiphysis

What is the drug of choice for the treatment of ventricular dysrhythmias and/or hypotension, secondary to tricarboxylic acid antidepressant (TCA) poisoning.

Sodium bicarbonate

Treagtment of hyperkalemia acutely.

Sodium bicarbonate, glucose D50, and insulin can be used to drive potassium back into the cell. These treatments are short acting but also have short duration.

A 2-year old infant was admitted for failure to thrive. The infant suffered from 4 episodes of respiratory tract infection since birth. During neonatal period, the child had intestinal obstruction due to inspissated meconium. The child is asthenic with little subcutaneous tissue and has a protuberant abdomen. Both the parents of the child are normal. Which of the following laboratory tests will be enable you to establish a diagnosis in this case? Serum immunoglobulin level CT scan of the abdomen Bronchoscopic examination Sodium chloride content of sweat Endoscopic biopsy of intestinal mucosa

Sodium chloride content of sweat Cystic fibrosis should be suspected in young adults suffering from a history of: Chronic lung disease (esp. bronchiectasis) Pancreatitis Or infertility Characteristically sodium chloride levels in the sweat of such individuals is unusually high. A pilocarpine iontophoeresis sweat test reveals elevated sodium and chloride levels In the sweat of patients with cystic fibrosis.

What is the treatment option for extremely high levels of potassium

Sodium polystyrene sulfonate is used to remove potassium from the body. Hemodialysis may be necessary.

A 50-year-old Hispanic male has a solitary 5-mm pulmonary nodule on a chest radiograph. His only medical problem is severe osteoarthritis. He quit smoking 10 years ago. Which one of the following would be the most appropriate follow-up for the pulmonary nodule? (check one) A. Positron emission tomography (PET) B. Chest CT C. A repeat chest radiograph in 6 weeks D. A repeat chest radiograph in 6 months

Solitary pulmonary nodules are common radiologic findings, and the differential diagnosis includes both benign and malignant causes. The American College of Chest Physicians guidelines for evaluation of pulmonary nodules are based on size and patient risk factors for cancer. Lesions ≥8 mm in diameter with a "ground-glass" appearance, an irregular border, and a doubling time of 1 month to 1 year suggest malignancy, but smaller lesions should also be evaluated, especially in a patient with a history of smoking. CT is the imaging modality of choice to reevaluate pulmonary nodules seen on a radiograph (SOR C). PET is an appropriate next step when the cancer pretest probability and imaging results are discordant (SOR C). Patients with notable nodule growth during follow-up should undergo a biopsy (SOR C).`

A 25-year-old white female comes to your office complaining of abdominal pain. She requests that you hospitalize her and do whatever is necessary to get rid of the pain that has been present for a number of years. She has difficulty describing the pain. She is a single parent, and becomes defensive when asked about her previous marriage, stating only that her former husband is an alcoholic, "just like my father." Her previous medical history includes an appendectomy, a cholecystectomy, and a hysterectomy. On physical examination she appears healthy and a CBC, erythrocyte sedimentation rate, serum amylase level, serum electrolyte levels, and multiple chemical profile are all normal. Management of this patient should include which one of the following? (check one) A. Long-term use of antidepressants B. Referral to a surgeon for exploratory laparotomy C. Informing her that her problems are psychogenic and that there is nothing to worry about D. Hospitalization as requested, then consultation with a psychiatrist E. Scheduling frequent, regular office visits

Somatoform disorder is often encountered in family practice. Studies have documented that 5% of patients meet the criteria for somatization disorder, while another 4% have borderline somatization disorder. Most of these patients are female and have a low socioeconomic status. They have a high utilization of medical services, usually reflected by a thick medical chart, and are often single parents. As a rule, physicians tend to be less satisfied with the care rendered to these patients as opposed to those without the disorder. Patients with multiple unexplained physical complaints have been described as functionally disabled, spending an average of one week per month in bed. Many of these patients seek and are ultimately granted surgical procedures, and it is not uncommon for them to have multiple procedures, especially involving the pelvic area. Often there are associated psychiatric symptoms such as anxiety, depression, suicidal threats, alcohol or drug abuse, interpersonal or occupational difficulties, and antisocial behavior. A background of a dysfunctional family unit in which one or both parents abused alcohol or drugs or were somatically preoccupied is also quite common. Unfortunately, these individuals tend to marry alcohol abusers, and thus continue the pattern of dysfunctional family life. Treatment of somatoform disorder should be by one primary physician where an established relationship and regular visits can curtail the dramatic symptoms that many times lead to hospitalization. The family physician is in a position to monitor family dynamics and provide direction on such issues as alcoholism and child abuse. Each office visit should be accompanied by a physical examination, and the temptation to tell the patient that the problem is not physical should be avoided. Knowing the patient well helps to avoid unnecessary hospitalization, diagnostic procedures, surgery, and laboratory tests. These should be done only if clearly indicated. Psychotropic medications should be avoided except when clearly indicated, as medications reinforce the sick role, may be abused, and may be used for suicidal gestures. Following these recommendations significantly decreases the cost of care for the patient.

Hirsutism Tx

Spironolactone and OCPs

A 5-year-old boy presents with 6-day history of fever, fatigue, and rash. He has no significant past medical history. He is current with his vaccinations, except for varicella, which his parents have refused in the past. On exam, his temperature is 101.3º F; heart rate is 110, and blood pressure is 94. He has bilateral conjunctival injection, an erythematous pharynx without exudate, cracked red lips, and an erythematous right tympanic membrane. He has shotty enlarged anterior cervical lymph nodes bilaterally, the largest nodes measuring 1.6 cm on the right side and 1.5 cm on the left side. His lungs are clear, and his heart has a regular rhythm. His abdomen is soft. He is in no acute distress, and he has a generalized maculopapular rash. What are some other possible findings associated with his probable diagnosis? Sterile pyuria, meningismus Peripheral neuropathy, thrombocytopenia Epistaxes, hematuria Colonic polyps, gastritis Splenomegaly, pleuritis

Sterile pyuria, meningismus This patient meets the diagnostic criteria for Kawasaki disease, which includes fever for at least 5 days, and at least 4 of the 5 following signs: bilateral conjunctival injection, generally nonpurulent changes in the mucosa of the oropharynx changes of the peripheral extremities rash cervical adenopathy >1.5 cm, usually unilateral The most serious complication of Kawasaki disease is a coronary aneurysm with the potential for thrombosis, myocardial infarction, and rupture. Other possible findings include aseptic meningitis, diarrhea, hepatitis, hydrops of the gallbladder, urethritis and meatitis with sterile pyuria, otitis media, arthritis, myocarditis, and pericarditis.

Glomerulonephritis treatment

Steroids and immunosuppressives to control the inflammatory response. Salt/fluid management. Dialysis if azotemia is present. ACE to reduce protein loss

What are the symptoms of frozen shoulder symptom?

Stiffness, loss of active and passive rotation, severe pain including at night/rest

Most common innocent murmur of early childhood. It is described as musical, vibratory, short, and high-pitched.

Still Murmur

What medication should be given when an opioid pain medication is started?

Stimulant laxative. Methylnaltrexone bromide (Relistor)

Treatment for nephrolithiasis

Stones less than 5 mm should pass spontaneously. Alpha or calcium channel blocker may help to pass them. 5-10 mm should have elective lithotripsy or ureteroscopy with stone basket extraction Greater than 10 mm requires urethral stent of percutaneous nephrostomy is the gold standard. Extracorporeal shock wave lithotripsy can be used for renal stones less than 2 cm and ureteral stones less than 1 cm

What are the typical causes of CAP?

Strep pneumo (single rigor, pleurisy) >H. Flu (common with COPDers)>M. Cat

Most common cause of infective endocarditis (IE)

Streptococcus viridans followed by Staphylococcus aureus

Which one of the following would suggest that the sudden and unexpected death of a healthy infant resulted from deliberate suffocation rather than sudden infant death syndrome? (check one) A. No previous history of apneic episodes B. An age of 9 months C. Mottled skin D. Clenched fists E. Blood-tinged froth in the mouth

Sudden infant death syndrome (SIDS) is the most common cause of death during the first 6 months of life in the United States, with a peak incidence at 2-4 months of age and a quick dropoff by the age of 6 months. The cause of death is a retrospective diagnosis of exclusion, and is supported by a history of quiet death during sleep in a previously healthy infant younger than 6 months of age. Evidence of terminal activity may be present, such as clenched fists or a serosanguineous, blood-tinged, or mucoid discharge from the mouth or nose. Lividity and mottling are frequently present in dependent areas.

How does uncomplicated Influenza present?

Sudden onset of HA, Fever, chills, sore throat and myalgias particularly in the thighs and back following an incubation period of one to four day

How does complicated influenza present?

Sudden onset of HA, Fever, chills, sore throat and myalgias particularly in the thighs and back with PROGRESSIVE DYSPNEA

Most commonly used agents for treatment for type II diabetes

Sulfonylureas (glyburide, glipizide, and glimepiride

Treatment of acute exacerbation of COPD

Supplemental O2, albuterol and ipatropium may be necessary, systemic steroid taper, antibiotics (Doxy or amoxicillin)

A 6-week-old male infant presents with a 4-day history of cough and nasal congestion. According to his mother, he occasionally has a bluish tint around his lips while sleeping. There is no history of fever. His older siblings have an upper respiratory infection. The patient's appetite has been decreased somewhat, mostly due to the copious nasal secretions; however, he has been maintaining a normal urine output. He was delivered pre-term at approximately 34 weeks gestation. He had mild respiratory distress syndrome, spending 2 days on a ventilator in the neonatal intensive care unit (NICU). He went home in 10 days and has done well since. He has had no immunizations. A physical exam reveals an infant in mild respiratory distress, respirations of 52/min, with slight intercostal retractions. Temperature is 100.2 F with a HR of 130/min. Perioral duskiness is seen. Oxygen saturation at room air is 83%, and HEENT exam is otherwise normal. His chest exam shows coarse rhonchi and expiratory wheezes. Heart rate and rhythm are regular. No murmurs appreciated. Abdomen is soft and non-tender. Neurological is intact. Chest X-ray shows mild hyper expansion, but no consolidation. Nasal swab for respiratory syncytial virus (RSV) is positive. What is the most appropriate treatment? Aerosolized albuterol Intravenous steroids Empiric ampicillin and cefotaxime Aerosolized ribavirin Supplemental oxygen and supportive therapy

Supplemental oxygen and supportive therapy The severity of bronchiolitis is diagnosed and assessed based on history and physical examination. Infants with mild bronchiolitis can be treated symptomatically at home. The following are the criteria for hospitalization in RSV bronchiolitis: Age less than 12 weeks Gestational age at birth of less than 34 weeks Cardiopulmonary disease or immunodeficiencies Wheezing and respiratory distress associated with oxygen saturation below 92 percent on room air (hypoxemia) History of significant apnea before assessment. The management in moderate-to-severe bronchiolitis involves supplemental oxygen therapy and supportive measures such as prevention of dehydration and respiratory support. Supplemental oxygen is the single most useful therapy, usually delivered via nasal prongs when oxygen saturation (SpO2) falls persistently below 90% in previously healthy infants. Mechanical ventilation may be needed for respiratory failure or severe apnea.

Treatment of Varicella-Zoster

Supportive

Treatment of a cerebral aneurysm/subarachnoid hemorrhage?

Supportive Medical treatment (strict bed rest, mild sedation, or administration of stool softeners. Management of hypertension. Surgical management includes wrapping or clipping the aneursym

Varicocele Treatment

Surgical repair (left spermatic vein ligation) if it is painful or appears to cause infertility.

Treatment of thyroid cancer

Surgical resection and near-total thyroidectomy follow by radiation for anaplastic disease. Monitor for hypocalcemia. T4 therapy to prevent hypothyroidism. Radioactive iodine ablation for residual disease.

A 7-year-old boy presents with a 6-day history of a rash on his right flank. It is painful and appears blistered in some areas. His mother states that his right flank became painful a few days prior to the onset of the rash. He is up to date on his immunizations. He had chicken pox at the age of 18 months. He has been well recently, his past medical history is unremarkable, and he has no known drug allergies. On exam, the boy has a vesiculobullous rash on his right flank in a continuous band from the middle of his back spreading around to his mid-axillary line. No other lesions are present. He does have some right axillary lymphadenopathy as well. What is the most appropriate treatment option? Symptomatic treatment and prevention of secondary infection Oral acyclovir Intravenous varicella-zoster immune globulin Oral cephalexin Aspirin

Symptomatic treatment and prevention of secondary infection In a normal child, symptomatic treatment and prevention of secondary infection are the mainstays of treatment.

Treatment for Acute Bronchitis

Symptomatic treatment. Cough suppressants and expectorants not recommended.

Treatment of Epstein Barr virus (EBV)

Symptomatic- NSAIDs. Steroids are indicated for thrombocytopenia

How do you treat Huntington's Disease?

Sypmtopatically.

Treatment of pemphigus vulgaris

Systemic therapy starting with prednisone then an immunosuppressant, azathioprine and/or methotrexate.

How do you treat bullous pemphigoid/

Systemic therapy starting with prednisone then azathioprine

A 2-year-old boy is evaluated by a specialist at his primary physician's referral. His medical records indicate multiple admissions; there were 2 for CMV pneumonia, pneumocystis pneumonia, esophageal candidiasis, and severe herpes simplex labialis. His mother says her pregnancy and delivery were uneventful. Soon after birth, the child had multiple seizures due to an electrolyte disturbance; the mother is not sure exactly what the disturbance was. Immunoglobulin levels are normal. There is no significant family history of disease. Examination reveals a malnourished baby with flat facies and a fish-shaped mouth. The rest of the examination appears normal. A chest X-ray shows the absence of a 'sail sign'. What is the likely pathogenic mechanism?

T cell dysfunction

What pharm treatment should be done for post-surgical urinary retention?

Tamsulosin (alpha-blockers)

A 62-year-old male with a history of prostate cancer and well-controlled hypertension presents with severe osteoporosis. At 55 years of age he received prostate brachytherapy and androgen deprivation for his prostate cancer and has been disease-free since. He presently takes lisinopril (Prinivil, Zestril), 5 mg daily; alendronate (Fosamax), 70 mg weekly; calcium, 1000 mg daily; and vitamin D, 1200 units daily. He has never smoked, exercises five times a week, and maintains a healthy lifestyle. In spite of his lifestyle and the medications he takes, he continues to have severe osteoporosis on his yearly bone density tests. In addition to recommending fall precautions, which one of the following would you consider next to treat his osteoporosis? (check one) A. Testosterone B. Calcitonin C. Teriparatide (Forteo) D. Raloxifene (Evista) E. Zoledronic acid (Reclast)

Teriparatide is indicated for the treatment of severe osteoporosis, for patients with multiple osteoporosis risk factors, or for patients with failure of bisphosphonate therapy (SOR B). Therapy with teriparatide is currently limited to 2 years and is contraindicated in patients with a history of bone malignancy, Paget disease, hypercalcemia, or previous treatment with skeletal radiation. Its route of administration (subcutaneous) and high cost should be considered when prescribing teriparatide therapy. Testosterone therapy is contraindicated in patients with a history of prostate cancer. Zoledronic acid is a parenterally administered bisphosphonate and would not be appropriate in a patient who has already failed bisphosphonate therapy. Likewise, raloxifene and calcitonin are not indicated in patients with severe osteoporosis who have failed bisphosphonate therapy.

A critically ill adult male is admitted to the intensive-care unit because of sepsis. He has no history of diabetes mellitus, but his glucose level on admission is 215 mg/dL and insulin therapy is ordered. Which one of the following is the most appropriate target glucose range for this patient? (check one) A. 80-120 mg/dL B. 100-140 mg/dL C. 120-160 mg/dL D. 140-180 mg/dL E. 160-200 mg/dL

The 2009 consensus guidelines on inpatient glycemic control issued by the American Association of Clinical Endocrinologists and the American Diabetes Association recommend insulin infusion with a target glucose level of 140-180 mg/dL in critically ill patients. This recommendation is based on clinical trials in critically ill patients. In the groups studied, there was no reduction in mortality from intensive treatment targeting near-euglycemic glucose levels compared to conventional management with a target glucose level of <180 mg/dL. There also were reports of harm resulting from intensive glycemic control, including higher rates of severe hypoglycemia and even increased mortality.

The mother of a 3-year-old male is concerned that he doesn't like being held, doesn't interact much with other children, and rarely smiles. Of the following, which feature would be most helpful in distinguishing Asperger's syndrome from autism in this patient? (check one) A. Normal language development B. Delayed gross motor development C. Repetitive fine motor mannerisms D. Preoccupation with parts of objects E. Focused patterns of intense interest

The DSM-IV categorizes Asperger's syndrome and autism as pervasive developmental disorders. In both conditions, children have significant difficulties with social interactions, although the impairment is more severe and sustained in autism. Both Asperger's and autism may be associated with symptoms of repetitive motor mannerisms, restricted patterns of interest (which are abnormal in focus or intensity), or preoccupation with parts of objects. However, unlike children with Asperger's syndrome, autistic children have serious problems with communication skills, either in the development of speech itself or in the ability to carry on a conversation. Normal, age-appropriate language skills in a 3-year-old would rule out a diagnosis of autism. It is an important distinction to make, as the prognosis for independent functioning in children with Asperger's syndrome is significantly better than in children with autism.

Which one of the following patients should be advised to take aspirin, 81 mg daily, for the primary prevention of stroke? (check one) A. A 42-year-old male with a history of hypertension B. A 72-year-old female with no chronic medical conditions C. An 80-year-old male with a history of depression D. An 87-year-old female with a history of peptic ulcer disease `

The U.S. Preventive Services Task Force (USPSTF) has summarized the evidence for the use of aspirin in the primary prevention of cardiovascular disease as follows: The USPSTF recommends the use of aspirin for men 45-79 years of age when the potential benefit from a reduction in myocardial infarctions outweighs the potential harm from an increase in gastrointestinal hemorrhage (Grade A recommendation) The USPSTF recommends the use of aspirin for women 55-79 years of age when the potential benefit of a reduction in ischemic strokes outweighs the potential harm of an increase in gastrointestinal hemorrhage (Grade A recommendation) The USPSTF concludes that the current evidence is insufficient to assess the balance of benefits and harms of aspirin for cardiovascular disease prevention in men and women 80 years of age or older (Grade I statement) The USPSTF recommends against the use of aspirin for stroke prevention in women younger than 55 and for myocardial infarction prevention in men younger than 45 (Grade D recommendation) In summary, consistent evidence from randomized clinical trials indicates that aspirin use reduces the risk for cardiovascular disease events in adults without a history of cardiovascular disease. It reduces the risk for myocardial infarction in men, and ischemic stroke in women. Consistent evidence shows that aspirin use increases the risk for gastrointestinal bleeding, and limited evidence shows that aspirin use increases the risk for hemorrhagic strokes. The overall benefit in the reduction of cardiovascular disease events with aspirin use depends on baseline risk and the risk for gastrointestinal bleeding.

A 12-year-old white male is brought to your office after accidentally cutting his left hand with a pocketknife. On examination you find a deep 2-cm laceration at the base of the thenar eminence. To test for motor injury to the median nerve you would have the patient: (check one) A. Extend the thumb and fingers B. Oppose the thumb and little finger C. Flex the wrist D. Abduct the thumb and index finger

The ability to touch the tip of the thumb to the tip of the little finger indicates normal motor function of the median nerve. The radial nerve controls extension of the thumb and fingers. The median nerve partially controls flexion of the wrist, but the site of innervation is proximal to the wound site at the base of the thumb. Abduction of the thumb is a function of the radial nerve. Finger abduction is a function of the ulnar nerve.

You are examining a 6-month-old male infant who is receiving sulfamethoxazole/trimethoprim IV 20 mg/kg for suspected Pneumocystis jiroveci pneumonia. The patient has suffered respiratory tract infections and chronic diarrhea since birth. His parents are not related. 3 of his older brothers died of pneumonia at the age of 6, 8, and 12 months; however, both his 10-year-old sister and 12-year-old sister are healthy. On examination, his vitals are normal; his weight, height, and head circumference are lower than expected for the age. Eczematous skin rash and thrush are noted. You are not able to palpate the lymph nodes; there are no signs of hepatosplenomegaly, and the rest of the examination is normal. What do you expect that the chest X-ray of this patient will reveal?

The absence of a thymus shadow (FYI SCID not HIV/AIDS)

You are consulted for medical management of a 45-year-old male, previously unknown to you, who is hospitalized in the psychiatric unit with paranoid schizophrenia. His fasting blood glucose level is 180 mg/dL. Which one of the following medications is the most likely cause of the hyperglycemia? (check one) A. Alprazolam (Xanax) B. Haloperidol (Haldol) C. Chlorpromazine (Thorazine) D. Olanzapine (Zyprexa) E. Thiothixene (Navane)

The atypical antipsychotics include clozapine, olanzapine, risperidone, ziprasidone, quetiapine, and aripiprazole. As a class, they have fewer extrapyramidal side effects than the classical antipsychotics haloperidol, thiothixene, chlorpromazine, and others. Some of the atypical agents, notably olanzapine and clozapine, have been associated with hyperglycemia and the development of type 2 diabetes mellitus. Neither benzodiazepines like alprazolam nor the classical antipsychotics have been associated with hyperglycemia.

The FDA has imposed a black box warning on all thiazolidinediones, such as pioglitazone (Actos). This warning addresses a contraindication to the prescription of these drugs in patients with: (check one) A. renal insufficiency B. dementia C. exposure to radiocontrast media D. heart failure E. respiratory failure

The black box warning for thiazolidinediones specifically addresses heart failure. These agents are also contraindicated in patients with type 1 diabetes mellitus or hepatic disease, and in premenopausal anovulatory women

Which cardiac arrhythmia has been reported with high-dose methadone use? (check one) A. Atrial fibrillation B. Paroxysmal supraventricular tachycardia C. Third degree heart block D. Torsades de pointes E. Multifocal atrial tachycardia

The cardiac toxicity of methadone is primarily related to QT prolongation and torsades de pointes

You have provided care for a 27-year-old married, monogamous female for several years. One year ago, she had abnormal cervical cytology that was interpreted as "atypical squamous cells of undetermined significance" (ASC-US). She had repeat cervical cytologic examinations 6 months ago and again last week, both reported as negative. Which one of the following would be the most appropriate next step? (check one) A. Repeat cervical cytology again in 4-6 months B. Screening for human papillomavirus C. Colposcopic examination with a biopsy and endocervical curettage D. Resuming a routine screening protocol E. Cervical culture for herpesvirus

The cervical cytology category of atypical squamous cells of undetermined significance (ASC-US) is one that is poorly reproducible, having been shown to be frequently downgraded to negative or upgraded to a low- or high-grade squamous intraepithelial lesion on review. Recommended management strategies for women with ASC-US include repeat cytology at 4-6 months, immediate colposcopy, and reflex DNA testing for oncogenic HPV types. Should two repeat cytologic examinations at 4- to 6-month intervals prove negative, the patient can safely return to routine cytologic screenings. Should any repeat examination detect ASC-US or more significant cytology, colposcopy is indicated

An otherwise healthy 37-year-old male presents to your office with a 2-week history of redness and slight irritation in his groin. On examination a tender erythematous plaque with mild scaling is seen in his right crural fold. The area fluoresces coral-red under a Wood's light. Which one of the following would be the most appropriate treatment at this time? (check one) A. Amoxicillin B. Erythromycin C. Ketoconazole D. Nystatin (Mycostatin) E. Triamcinolone (Kenalog)

The characteristics of this lesion, including coral-red fluorescence under a Wood's light, suggests Corynebacterium infection, which is associated with erythrasma. Tinea cruris caused by Microsporum infection fluoresces green, while intertrigo and tinea cruris caused by Epidermophyton or Trichophyton infections do not fluoresce. Erythromycin, either systemic or topical, is the treatment of choice.

A 40-year-old female comes to your office for a routine examination. She has been in good health and has no complaints other than obesity. Her mother is diabetic and the patient has had a child that weighed 9 lb at birth. Her examination is negative except for her obesity. A fasting glucose level is 128 mg/dL, and when repeated 2 days later it is 135 mg/dL. Which one of the following would be most appropriate at this point? (check one) A. Diagnose type 2 diabetes mellitus and begin diet and exercise therapy B. Begin an oral hypoglycemic agent C. Order a glucose tolerance test D. Tell the patient that she has impaired glucose homeostasis but is not diabetic

The criteria for diagnosing diabetes mellitus include any one of the following: symptoms of diabetes (polyuria, polydipsia, weight loss) plus a casual glucose level ≥200 mg/dL; a fasting plasma glucose level ≥126 mg/dL; or a 2-hour postprandial glucose level ≥200 mg/dL after a 75 gram glucose load. In the absence of unequivocal hyperglycemia the test must be repeated on a different day. The criteria for impaired glucose homeostasis include either a fasting glucose level of 100-125 mg/dL (impaired fasting glucose) or a 2-hour glucose level of 140-199 mg/dL on an oral glucose tolerance test. Normal values are now considered <100 mg/dL for fasting glucose and <140 mg/dL for the 2-hour glucose level on an oral glucose tolerance test.

A 15-month-old male is brought to your office 3 hours after the onset of an increased respiratory rate and wheezing. He has an occasional cough and no rhinorrhea. His immunizations are up to date and he attends day care regularly. His temperature is 38.2°C (100.8°F), respiratory rate 42/min, and pulse rate 118 beats/min. The child is sitting quietly on his mother's lap. His oxygen saturation is 94% on room air. On examination you note inspiratory crackles in the left lower lung field. The child appears to be well hydrated and the remainder of the examination, including an HEENT examination, is normal. Nebulized albuterol (AccuNeb) is administered and no improvement is noted. Which one of the following would be most appropriate in the management of this patient? (check one) A. Laboratory evaluation B. Inpatient monitoring, with no antibiotics at this time C. Hospitalization and intravenous ceftriaxone (Rocephin) D. Close outpatient follow-up, with no antibiotics at this time E. Oral high-dose amoxicillin (90 mg/kg/day), with close outpatient follow-up

The diagnosis of community-acquired pneumonia is mostly based on the history and physical examination. Pneumonia should be suspected in any child with fever, cyanosis, and any abnormal respiratory finding in the history or physical examination. Children under 2 years of age who are in day care are at higher risk for developing community-acquired pneumonia. Laboratory tests are rarely helpful in differentiating viral versus bacterial etiologies and should not be routinely performed. Outpatient antibiotics are appropriate if the child does not have a toxic appearance, hypoxemia, signs of respiratory distress, or dehydration. Streptococcus pneumoniae is one of the most common etiologies in this age group, and high-dose amoxicillin is the drug of choice.

A 50-year-old male is brought to the emergency department with shortness of breath, chest tightness, tremulousness, and diaphoresis. Aside from tachypnea, the physical examination is normal. Arterial blood gases on room air show a pO2 of 98 mm Hg (N 80-100), a pCO2 of 24 mm Hg (N 35-45), and a pH of 7.57 (N 7.38-7.44). The most likely cause of the patient's blood gas abnormalities is: (check one) A. carbon monoxide poisoning B. anxiety disorder with hyperventilation C. an acute exacerbation of asthma D. pulmonary embolus E. pneumothorax

The elevated pH, normal oxygen saturation, and low pCO2 are characteristic of acute respiratory alkalosis, as seen with acute hyperventilation states. In patients with a pulmonary embolism, pO2 and pCO2 are decreased, while the pH is elevated, indicating the acute nature of the disorder. With the other diagnoses, findings on the physical examination would be different than those seen in this patient. Vital signs would be normal with carbon monoxide poisoning, and patients with an asthma exacerbation have a prominent cough and wheezing, and possibly other abnormalities. Tension pneumothorax causes severe cardiac and respiratory distress, with significant physical findings including tachycardia, hypotension, and decreased mental activity.

A 3-year-old toilet-trained female is brought to your office by her mother, who has noted a red rash on the child's perineum for the last 5 days. The rash is pruritic and has been spreading. The mother has treated the area for 3 days with nystatin cream with no obvious improvement. The child has not used any other recent medications and has no significant past medical history. Your examination reveals a homogeneous, beefy red rash surrounding the vulva and anus. The most likely etiologic agent is: (check one) A. Malassezia furfur B. Escherichia coli C. Haemophilus influenzae D. Staphylococcus aureus E. group A Streptococcus pyogenes

The epidemiology of group A streptococcal disease of the perineum is similar to that of group A streptococcal pharyngitis, and the two often coexist. It is theorized that either auto-inoculation from mouth to hand to perineum occurs, or that the bacteria is transmitted through the gastrointestinal tract. In one study, the average age of patients with this disease varied from 1 to 11 years, with a mean of 5 years. Girls and boys were almost equally affected. The incidence is estimated to be about 1 in 200 pediatric visits and peaks in March, April, and May in North America. The condition usually presents with itching and a beefy redness around the anus and/or vulva and will not clear with medications used to treat candidal infections.

A 28-year-old female presents for evaluation of a persistent thin discharge, with a "fishy" odor particularly noticeable after intercourse. She has no dyspareunia or dysuria, is in a monogamous relationship, and has used oral contraceptives for many years. Physical examination reveals no vulvar, vaginal, or cervical erythema. There is a homogenous white discharge that coats the vaginal walls. The vaginal pH is 7.0 and on microscopy you note stippled epithelial cells but no hyphae or trichomonads. Which one of the following is true regarding this patient? (check one) A. The treatment of choice may interact with alcohol B. The patient's partner needs to be treated simultaneously C. The diagnosis should be confirmed with a culture D. Oral contraceptives contribute to the risk for this condition

The patient has the typical symptoms and signs of bacterial vaginosis. There is no need for confirmatory testing. The treatment of choice is oral metronidazole, which may cause a disulfiram-like interaction with alcohol. Treatment of the partner has not been shown to improve the outcome.

An 8-year-old African-American girl is admitted to the hospital for reassessment of her seizure disorder. She has suffered generalized tonic-clonic seizures since birth in spite of many different medications and consultations. She is in the neurologic ward of the hospital, and you are the intern covering the pediatric ward for the night. At the evening report, you are relieved that the patient has not had any prolonged seizure activity since admission two days ago. She has been undergoing video-EEG testing, and the results will not be available until tomorrow. The nature of her seizure disorder is unknown. Her prenatal course was uncomplicated. She was a full term, natural spontaneous vaginal delivery. Her Apgar scores were 8 and 9. She had no perinatal fever, sepsis, or meningitis. She was first observed to have seizure form activity at 10 days of life, when she was rushed back to the hospital. Since infancy, she has seen many specialists in the area without any definitive explanation for the seizures. The medications have tempered the frequency and duration of the seizures, but she did not meet her developmental milestones and has been diagnosed with mild mental retardation. You review her lab reports and find that her electrolytes are normal, phenytoin level is therapeutic, and complete blood count is normal. The chart indicates that she has no fever and her blood pressure is normal. You leave to check on some other patients, but are called back at 2AM by the nurse who reports that the patient has been seizing for more than 30 minutes. Shorter episodes of seizure activity are expected during the video EEG monitoring period, but the nurse is concerned about the duration of the seizure activity. You rush to the patient's bedside, and find that there has been seizure activity recorded on the EEG for the past 35 minutes. The best description of your assessment and management at this point would be:

The patient is in status epilepticus and propofol could be given to stop the seizure

A 6-year-old male is brought to your clinic with complaints of high fever and a rash all over his face. The boy arrived in this country a week ago from his home in the Asian subcontinent. His mother states that he has a 4 to 5-day history of a cough and cold with a fever and suddenly developed a rash over his face today. She says that he had 3 vaccinations when he was an infant and none after that. On examination, the boy looks malnourished and very ill, with a 104 degree temperature. There was some congestion of the conjunctiva, and some red spots were observed inside his buccal cavity. The rash on his face was maculopapular. What can be said of this condition?

The rash on the face may spread to other parts of the body

The recommended time to screen for gestational diabetes in asymptomatic women with no risk factors for this condition is: (check one) A. in the first trimester B. at 16-20 weeks gestation C. at 24-28 weeks gestation D. at 35-37 weeks gestation

The recommended time to screen for gestational diabetes is 24-28 weeks gestation. The patient may be given a 50-g oral glucose load followed by a glucose determination 1 hour later

A 30-year-old female who had a deep venous thrombosis in her left leg during pregnancy has an uneventful delivery. During the pregnancy she was treated with low molecular weight heparin. Just after delivery her left leg is pain free and is not swollen. She plans to resume normal activities soon. Which one of the following would be most appropriate with regard to anticoagulation? (check one) A. Discontinuing treatment, with no further evaluation B. Discontinuing treatment if venous Doppler ultrasonography is negative for thrombus C. Continuing low molecular weight heparin for 6 more weeks D. Switching to low-dose unfractionated heparin for 6 weeks E. Switching to aspirin for 6 weeks

The risk of pulmonary embolism continues in the postpartum period, and may actually increase during that time. For patients who have had a deep-vein thrombosis during pregnancy, treatment should be continued for 6 weeks after delivery, with either warfarin or low molecular weight heparin.

Which one of the following platelet counts is the threshold for prophylactic platelet transfusion in most patients? (check one) A. 10,000/μL B. 25,000/μL C. 40,000/μL D. 50,000/μL E. 100,000/μL

The threshold for prophylactic platelet transfusion is 10,000/μL (SOR A). Platelet transfusion decreases the risk of spontaneous bleeding in such patients. A count below 50,000/μL is an indication for platelet transfusion in patients undergoing an invasive procedure.

A 5-month-old female is brought in with a 1-day history of an axillary temperature of 100.6°F and mild irritability. Findings are normal on examination except for a runny nose and a moderately distorted, immobile, red right eardrum. There is no history of recent illness or otitis in the past. The most appropriate management would be: (check one) A. azithromycin (Zithromax) for 5 days B. amoxicillin for 10 days C. amoxicillin for 5 days D. oral decongestants E. observation and a repeat examination in 2 weeks

The treatment for otitis media is evolving. Recommendations by the American Academy of Family Physicians and the American Academy of Pediatrics advocate a 10-day course of antibiotics for children under the age of 2 years if the diagnosis is certain. If the diagnosis is not certain and the illness is not severe, there is an option of observation with follow-up. For children over the age of 2 years, the recommendation is still to treat if the diagnosis is certain, but there is an option of observation and follow-up if the illness is not severe and follow-up can be guaranteed. Amoxicillin is the first-line therapy; the recommended dosage is 80-90 mg/kg/day in two divided doses, which increases the concentration of amoxicillin in the middle ear fluid to help with resistant Pneumococcus. Azithromycin, because of a broader spectrum and potential for causing resistance, is not considered the treatment of first choice. Treatment regimens ranging from 5 to 7 days are appropriate for selected children over the age of 5 years.

A 12-year-old white male asthmatic has an acute episode of wheezing. You diagnose an acute asthma attack and prescribe an inhaled β2-adrenergic agonist. After 2 hours of treatment, he continues to experience wheezing and shortness of breath. Which one of the following is the most appropriate addition to acute outpatient management? (check one) A. Oral theophylline (Theo-Dur) B. Oral corticosteroids C. An oral β-adrenergic agonist D. Inhaled cromolyn (Intal) E. Inhaled corticosteroids

The treatment of choice for occasional acute symptoms of asthma is an inhaled β2-adrenergic agonist such as albuterol, terbutaline, or pirbuterol. If symptoms do not respond to β-agonists, they should be treated with a short course of systemic corticosteroids. Theophylline has limited usefulness for treatment of acute symptoms in patients with intermittent asthma; it is a less potent bronchodilator than subcutaneous or inhaled adrenergic drugs, and therapeutic serum concentrations can cause transient adverse effects such as nausea and central nervous system stimulation in patients who have not been taking the drug continuously. Cromolyn can decrease airway hyperreactivity, but has no bronchodilating activity and is useful only for prophylaxis. Inhaled corticosteroids should be used to suppress the symptoms of chronic persistent 2 asthma. Oral β2-selective agonists are less effective and have a slower onset of action than the same drugs given by inhalation.

Which one of the following is an absolute contraindication to electroconvulsive therapy (ECT)? (check one) A. Age >80 years B. A cardiac pacemaker C. An implantable cardioverter-defibrillator D. Pregnancy E. There are no absolute contraindications to ECT

There are no absolute contraindications to electroconvulsive therapy (ECT), but factors that have been associated with reduced efficacy include a prolonged episode, lack of response to medication, and coexisting psychiatric diagnoses such as a personality disorder. Persons who may be at increased risk for complications include those with unstable cardiac disease such as ischemia or arrhythmias, cerebrovascular disease such as recent cerebral hemorrhage or stroke, or increased intracranial pressure. ECT can be used safely in elderly patients and in persons with cardiac pacemakers or implantable cardioverter-defibrillators. ECT also can be used safely during pregnancy, with proper precautions and in consultation with an obstetrician.

Patients with which one of the following conditions are at increased risk for complications from electroconvulsive therapy for depression? (check one) A. Pregnancy B. Seizure disorder C. Cardiac pacemaker implantation D. Depression unresponsive to oral medications E. Recent cerebral hemorrhage

There are no absolute contraindications to electroconvulsive therapy (ECT), but more complications are seen in patients with a history of recent cerebral hemorrhage, stroke, or increased intracranial pressure. The efficacy of ECT may be reduced in patients who have not responded to oral antidepressants

An asymptomatic 3-year-old male presents for a routine check-up. On examination you notice a systolic heart murmur. It is heard best in the lower precordium and has a low, short tone similar to a plucked string or kazoo. It does not radiate to the axillae or the back and seems to decrease with inspiration. The remainder of the examination is normal. Which one of the following is the most likely diagnosis? (check one) A. Eisenmenger's syndrome B. Mitral stenosis C. Peripheral pulmonic stenosis D. Still's murmur E. Venous hum

There are several benign murmurs of childhood that have no association with physiologic or anatomic abnormalities. Of these, Still's murmur best fits the murmur described. The cause of Still's murmur is unknown, but it may be due to vibrations in the chordae tendinae, semilunar valves, or ventricular wall. A venous hum consists of a continuous low-pitched murmur caused by the collapse of the jugular veins and their subsequent fluttering, and it worsens with inspiration or diastole. The murmur of physiologic peripheral pulmonic stenosis (PPPS) is caused by physiologic changes in the newborns pulmonary vessels. PPPS is a systolic murmur heard loudest in the axillae bilaterally that usually disappears by 9 months of age. Mitral stenosis causes a diastolic murmur, and Eisenmenger's syndrome involves multiple abnormalities of the heart that cause significant signs and symptoms, including shortness of breath, cyanosis, and organomegaly, which should become apparent from a routine history and examinatio`

The probability of pregnancy after unprotected intercourse is the highest at which one of the following times? (check one) A. 3 days before ovulation B. 1 day before ovulation C. The day of ovulation D. 1 day after ovulation E. 3 days after ovulation

There is a 30% probability of pregnancy resulting from unprotected intercourse 1 or 2 days before ovulation, 15% 3 days before, 12% the day of ovulation, and essentially 0% 1-2 days after ovulation. Knowing the time of ovulation therefore has implications not only for "natural" family planning, but also for decisions regarding postcoital contraception

Which one of the following is necessary to make a diagnosis of polymyalgia rheumatica? (check one) A. Joint swelling B. Early morning stiffness C. Reduction of symptoms with high-dose NSAID therapy D. An erythrocyte sedimentation rate ≥60 mm/hr E. Bilateral shoulder or hip stiffness and aching

There must be bilateral shoulder or hip stiffness and aching for at least one month in order to make the diagnosis of polymyalgia rheumatica. Joint swelling occurs occasionally, but neither swelling nor early morning stiffness is necessary to make the diagnosis. Polymyalgia rheumatica does not respond to NSAIDs. The erythrocyte sedimentation rate should be ≥40 mm/hr.

A 47-year-old female presents to your office with a complaint of hair loss. On examination she has a localized 2-cm round area of complete hair loss on the top of her scalp. Further studies do not reveal an underlying metabolic or infectious disorder. Which one of the following is the most appropriate initial treatment? (check one) A. Topical minoxidil (Rogaine) B. Topical immunotherapy C. Intralesional triamcinolone (Kenalog) D. Oral finasteride (Proscar) E. Oral spironolactone (Aldactone)

These findings are consistent with alopecia areata, which is thought to be caused by a localized autoimmune reaction to hair follicles. It occasionally spreads to involve the entire scalp (alopecia totalis) or the entire body (alopecia universalis). Spontaneous recovery usually occurs within 6-12 months, although areas of regrowth may be pigmented differently. Recovery is less likely if the condition persists for longer than a year, worsens, or begins before puberty. The initial treatment of choice for patients older than 10 years of age, in cases where alopecia areata affects less than 50% of the scalp, is intralesional corticosteroid injections. Minoxidil is an alternative for children younger than 10 years of age or for patients in whom alopecia areata affects more than 50% of the scalp. While topical immunotherapy is the most effective treatment for chronic severe alopecia areata, it has the potential for severe side effects and should not be used as a first-line agent. Finasteride inhibits 5 ß-reductase type 2, resulting in a decrease in dihydrotestosterone levels, and is used in the treatment of androgenic alopecia (male-pattern baldness). Similarly, spironolactone is sometimes used for androgenic alopecia because it is an aldosterone antagonist with antiandrogenic effects

What antihypertensive medication will help with managing kidney stones?

Thiazide diuretics. They may also reduce urinary calcium excretion and therefore limit the occurrence of calcium oxalate nephrolithiasis.WORSEN: LOOP diuretics

Which DM II medication is used in conjunction with sulfonylurea, metformin, or insulin to sensitize peripheral tissue to insulin?

Thiazolidinediones

Management of hyperthyroidism (thyrotoxicosis)

Thioureas (methimazole or propylthiouracil). These drugs take several weeks to take full effect and are taken for 12-24 months.

A 23-month-old child is brought to your office with a 2-day history of a fever to 102°F (39°C), cough, wheezing, and mildly labored breathing. He has no prior history of similar episodes and there is no improvement with administration of an aerosolized bronchodilator. Which one of the following is now indicated? (check one) A. Bronchodilator aerosol treatment every 6 hours B. Corticosteroids C. An antibiotic D. A decongestant E. Supportive care only

This child has typical findings of bronchiolitis. The initial infection usually occurs by the age of 2 years. It is caused by respiratory syncytial virus (RSV). Bronchodilator treatment may be tried once and discontinued if there is no improvement. Treatment usually consists of supportive care only, including oxygen and intravenous fluids if indicated (SOR B). Corticosteroids, antibiotics, and decongestants are of no benefit. RSV infection may recur, since an infection does not provide immunity. Up to 10% of infected children will have wheezing past age 5, and bronchiolitis may predispose them to asthma.

A 72-year-old male has had persistent interscapular pain with movement since rebuilding his deck 1 week ago. He rates the pain as 6 on a 10-point scale. A chest radiograph shows a thoracic vertebral compression fracture. Which one of the following would be most appropriate at this point? (check one) A. Complete bed rest for 2 weeks B. Markedly decreased activity until the pain lessens, and follow-up in 1 week C. Referral for vertebroplasty as soon as possible D. NSAIDs and referral for physical therapy

This patient has suffered a thoracic vertebral compression fracture. Most can be managed conservatively with decreased activity until the pain is tolerable, possibly followed by some bracing. Vertebroplasty is an option when the pain is not improved in 2 weeks. Complete bed rest is unnecessary and could lead to complications. Physical therapy is not indicated, and NSAIDs should be used with caution.

An 8-year-old female is brought to your office because she has begun to limp. She has had a fever of 38.8°C (101.8°F) and says that it hurts to bear weight on her right leg. She has no history of trauma. On examination, she walks with an antalgic gait and hesitates to bear weight on the leg. Range of motion of the right hip is limited in all directions and is painful. Her sacroiliac joint is not tender, and the psoas sign is negative. Laboratory testing reveals an erythrocyte sedimentation rate of 55 mm/hr (N 0-10), a WBC count of 15,500/mm 3 (N 4500-13,500), and a C-reactiveprotein level of 2.5 mg/dL (N 0.5-1.0). Which one of the following will provide the most useful diagnostic information to further evaluate this patient's problem? (check one) A. MRI B. CT C. A bone scan D. Ultrasonography E. Plain-film radiography

This child meets the criteria for possible septic arthritis. In this case ultrasonography is recommended over other imaging procedures. It is highly sensitive for detecting effusion of the hip joint. If an effusion is present, urgent ultrasound-guided aspiration should be performed. Bone scintigraphy is excellent for evaluating a limping child when the history, physical examination, and radiographic and sonographic findings fail to localize the pathology. CT is indicated when cortical bone must be visualized. MRI provides excellent visualization of joints, soft tissues, cartilage, and medullary bone. It is especially useful for confirming osteomyelitis, delineating the extent of malignancies, identifying stress fractures, and diagnosing early Legg-Calvé-Perthes disease. Plain film radiography is often obtained as an initial imaging modality in any child with a limp. However, films may be normal in patients with septic arthritis, providing a false-negative result.

A primigravida at 38 weeks gestation is concerned that her fetus is getting too large and wants to know what interventions could prevent complications from a large baby. On examination her uterine fundus measures 41 cm from the pubic symphysis. Ultrasonography is performed and an estimated fetal weight of 4000 g (8 lb 13 oz) is reported. Which one of the following management options is supported by the best evidence? (check one) A. Induction of labor B. Cesarean section C. Awaiting spontaneous labor D. Weekly ultrasonography to follow fetal growth

This estimated fetal weight is at the 90th percentile for a term fetus. Unfortunately, the accuracy of fetal weight estimates declines as pregnancy proceeds, and the actual size may be as much as 15% different from the estimate. Delivery of a large infant results in shoulder dystocia more often than delivery of a smaller infant, but most large infants are delivered without complications. Intuitively, it would seem logical to induce labor when the fetus seems to be getting large, but this intervention has been studied in controlled trials and the only difference in outcome was an increase in the cesarean rate for women who underwent elective induction for this indication. Recently, there has been an increase in requests from patients to have an elective cesarean section near term to avoid the risks of labor, including pain, shoulder dystocia, and pelvic relaxation. The American Congress of Obstetricians and Gynecologists (ACOG) recommends consideration of cesarean delivery without a trial of labor if the estimated fetal weight is 4500 g in a mother with diabetes mellitus, or 5000 g in the absence of diabetes. Even at that size, there is not adequate data to show that cesarean section is preferable to a trial of labor. Frequent ultrasonography is often performed to reduce anxiety for both patient and physician, but the problem of accuracy of weight estimates remains an issue even with repeated scans at term.

During rounds, you notice a new rash on a full-term 2-day-old white female. It consists of 1-mm pustules surrounded by a flat area of erythema, and is located on the face, trunk, and upper arms. An examination is otherwise normal, and she does not appear ill. Which one of the following is the most likely diagnosis? (check one) A. Erythema toxicum neonatorum B. Transient neonatal pustular melanosis C. Acne neonatorum D. Systemic herpes simplex E. Staphylococcus aureus sepsis

This infant has the typical "flea-bitten" rash of erythema toxicum neonatorum (ETN). Transient neonatal pustular melanosis is most common in African-American newborns, and the lesions lack the surrounding erythema typical of ETN. Acne neonatorum is associated with closed comedones, mostly on the face. As the infant described is not ill, infectious etiologies are unlikely.

A 45-year-old female presents with a complaint of pain and swelling in her right index finger of 2 days' duration. She reports that 5 days ago she had artificial nails applied, which she removed yesterday due to the pain. She used hydrogen peroxide on the finger, but it did not help. She denies any systemic symptoms or fever. On examination there is erythema and swelling in the lateral nail fold of the right index finger, with purulent material noted. Which one of the following would be the most appropriate treatment for this patient? (check one) A. Removal of the proximal nail fold B. Topical corticosteroids C. Topical antibiotics D. Topical antifungals

This is a common presentation for acute paronychia, which typically is caused by local trauma to the nail fold or cuticle, with resulting inoculation and infection. Topical antibiotics, with or without topical corticosteroids, is one treatment option. Other options include warm compresses, oral antibiotics, and incision and drainage; however, incision and drainage is not always necessary. Removal of the proximal nail fold is used to treat chronic paronychia that is not responsive to other treatments. Topical corticosteroids can be used alone for chronic paronychia, but if used for acute paronychia, they should be combined with antibiotics since acute paronychia is typically caused by a bacterial infection. Topical antifungals are a treatment option for chronic paronychia, which can be associated with a fungal infection, but not for acute paronychia.

You have recently begun caring for a 25-year-old white female who has multiple complaints. You have seen her 3 times for walk-in office visits over the past month. She has shown appreciation for your work during the encounters, but has been critical of your care when talking to the office staff. At times she has been kind and charming, and at other times she has been rude and verbally abusive to your staff. She has a string of multiple relationships in the past, none of which has lasted very long. During times of intense stress, she has sometimes engaged in self-mutilation. She frequently changes jobs and living arrangements. Which one of the following strategies would be most appropriate in the care of this patient? (check one) A. Strive to develop a close relationship with the patient B. Ignore verbal attacks on staff members C. Prescribe lorazepam (Ativan) D. Schedule frequent office visits for follow-up E. Provide detailed, technical explanations for any therapies provided

This patient demonstrates features of borderline personality disorder. These patients often demonstrate instability in interpersonal relationships and self-image, and may be impulsive. They can present with a wide range of symptoms, including depression, anger, paranoia, extreme dependency, self-mutilation, and alternating idealization and devaluation of their physicians. Their lives are often chaotic. Treatment strategies include maintaining a caring but somewhat detached professional stance. A close personal relationship is typically not therapeutic for these patients. Angry outbursts will often have to be tolerated, but limit-setting is necessary with respect to appropriate behaviors. SSRIs, atypical antipsychotics, and mood stabilizers may be of help at times, but anxiolytics are often abused and may be associated with self-mutilating behaviors. These patients tend to respond best to clear, simple, non-technical explanations related to their medical care.

HBsAg Post Exposure Prophylaxis: Vaccinated vs. Unvaccianted

VACCINATED - Test for HBsAg first UNVACCINATED - HBIG + Vaccine

A 50-year-old female presents with a 3-week history of a moderately pruritic rash, characterized by flat-topped violaceous papules 3-4 mm in size. The lesions are located primarily on the volar wrists and forearms, lower legs, and dorsa of both feet. Ten days after the rash first appeared she went to the emergency department and was treated for "possible scabies," but the treatment has made little or no difference. Which one of the following treatments is indicated at this time? (check one) A. Clobetasol (Cormax, Temovate) 0.05% ointment B. Permethrin 5% cream C. Dipyridamole (Persantine) D. Triamcinolone 0.1% cream

This patient has classic lichen planus, with pruritic, symmetrically distributed papular lesions. The violaceous flat-topped papules, usually 3-6 mm in size, are distinct and so characteristic in appearance that a biopsy is usually not necessary to make the diagnosis. First-line treatment is with high-potency topical corticosteroids such as clobetasol, as mid-potency topical agents such as triamcinolone are ineffective. Topical calcineurin inhibitors, including tacrolimus, can be used in cases not responding to topical corticosteroids. While scabies can masquerade as a variety of other dermatoses, retreatment with a scabicide is not indicated in this patient.

A 67-year-old female presents with progressive weakness, dry skin, lethargy, slow speech, and eyelid edema. Of the following medications currently taken by this patient, which one is most likely to be causing her symptoms? (check one) A. Donepezil (Aricept) B. Lithium C. Lisinopril (Prinivil, Zestril) D. Alendronate (Fosamax) E. Glyburide (DiaBeta, Micronase)

This patient has classic signs of hypothyroidism. Of the drugs listed, only lithium is associated with the development of hypothyroidism. In patients taking lithium, it is recommended that in addition to regular serum lithium levels, thyroid function tests including total free T4, and TSH be obtained yearly. ref: Griswold KS, Pessar LF: Management of bipolar disorder.

A 36-year-old male complains of clear rhinorrhea, nasal congestion, and watery, itchy eyes for several months. Tests in the past have suggested that he has an allergy to dust mites. Which one of the following is most likely to provide the most relief from his symptoms? (check one) A. Oral antihistamines B. An oral leukotriene-receptor antagonist C. Intranasal antihistamines D. Intranasal corticosteroids E. Furnace filters and mite-proof bedding covers

This patient has classic symptoms of allergic rhinitis. Intranasal corticosteroids are considered the mainstay of treatment for mild to moderate cases. In multiple studies, intranasal corticosteroid sprays have proven to be more efficacious than the other options listed, even for ocular symptoms. Air filtration systems and bedding covers have not been shown to reduce symptoms.

A 34-year-old female who delivered a healthy infant 18 months ago complains of a milky discharge from both nipples. She reports that normal periods have resumed since cessation of breastfeeding 6 months ago. She takes ethinyl estradiol/norgestimate (Ortho Tri-Cyclen) for birth control. A complete review of systems is otherwise negative. The most likely cause of the discharge is (check one) A. a medication side effect B. breast cancer C. a hypothalamic tumor D. hypothyroidism

This patient has galactorrhea, which is defined as a milk-like discharge from the breast in the absence of pregnancy in a non-breastfeeding patient who is more than 6 months post partum. It is more common in women ages 20-35 and in women who are previously parous. It also can occur in men. Medication side effect is the most common etiology. The most common pharmacologic cause of galactorrhea is oral contraceptives. Oral contraceptives that contain estrogen can both suppress prolactin inhibitory factor and stimulate the pituitary directly, both of which can cause galactorrhea. Other medications that can cause galactorrhea include metoclopramide, cimetidine, risperidone, methyldopa, codeine, morphine, verapamil, SSRIs, butyrophenones, dopamine-receptor blockers, tricyclics, phenothiazines, and thioxanthenes. Breast cancer is unlikely to present with a bilateral milky discharge. The nipple discharge associated with cancer is usually unilateral and bloody. Pituitary tumors are a pathologic cause of galactorrhea due to the hyperprolactinemia that is caused by the blockage of dopamine from the hypothalamus, or by the direct production of prolactin. However, patients often have symptoms such as headache, visual disturbances, temperature intolerance, seizures, disordered appetite, polyuria, and polydipsia. Patients with prolactinomas often have associated amenorrhea. These tumors are associated with marked levels of serum prolactin, often >200 ng/mL. Hypothalamic lesions such as craniopharyngioma, primary hypothalamic tumor, metastatic tumor, histiocytosis X, tuberculosis, sarcoidosis, and empty sella syndrome are significant but infrequent causes of galactorrhea, and generally cause symptoms similar to those of pituitary tumors, particularly headache and visual disturbances. It is rare for primary hypothyroidism to cause galactorrhea in adults. Symptoms that would be a clue to this diagnosis include fatigue, constipation, menstrual irregularity, weight changes, and cold intolerance.

A 30-year-old male complains of the gradual onset of anterior right knee pain on climbing the stairs. On examination there is no effusion, but there is tenderness over the medial retinaculum. There is good ligament strength, and range of motion is normal. When the knee is extended from 90° flexion to full extension, the patella deviates laterally. Which one of the following would be the best initial treatment for this condition? (check one) A. Bracing B. Taping C. NSAIDs D. Arthroscopic surgery E. Physical therapy

This patient has patellofemoral stress syndrome. It is often called runner's knee or anterior knee pain. The patellofemoral joint comprises the patella and femoral trochlea. The best initial treatment is physical therapy. Bracing, taping, and medications are unlikely to have better outcomes. Arthroscopic surgery is not indicated.

A previously healthy 60-year-old male is diagnosed with multiple myeloma after a workup for an incidental finding on routine laboratory work. He has no identified organ or tissue damage and is asymptomatic. Which one of the following would be appropriate treatment of this patient's condition? (check one) A. No treatment B. Chemotherapy C. Autologous stem cell transplantation D. Radiation

This patient has smoldering (asymptomatic) multiple myeloma. He does not have any organ or tissue damage related to this disease and has no symptoms. Early treatment of these patients does not improve mortality (SOR A) and may increase the likelihood of developing acute leukemia. The standard treatment for symptomatic patients under age 65 is autologous stem cell transplantation. Patients over 65 who are healthy enough to undergo transplantation would also be appropriate candidates. Patients who are not candidates for autologous stem cell transplantation generally receive melphalan and prednisolone with or without thalidomide. Radiotherapy can be used to relieve metastatic bone pain or spinal cord compression.

What is the most common cause of cute Bronchitis

VIRAL-Rhinovirus, coronavirus Bacterial causes: M. pneumoniae, C. pneumonia, Bordetella pertussis

Prophylactic treatment of cluster headaches

Valproate, cyproheptadine, lithium, CCB and oral corticosteroids.

Treatment of absence seizures?

Valproic acid or ethosuximide

You see a 6-year-old male for the third time in 3 months with a persistently painful hand condition. He has been treated with oral amoxicillin, followed by oral trimethoprim/ sulfamethoxazole (Bactrim, Septra), with no improvement. A physical examination reveals retraction of the proximal nail fold, absence of the cuticle, and erythema and tenderness around the nail fold area. The thumb and second and third fingers are affected on both hands. The patient is otherwise healthy. First-line treatment for this condition includes: (check one) A. warm soaks three times a day B. avoidance of emollient lotions C. a topical corticosteroid cream D. an oral antifungal agent

This patient has symptoms and signs consistent with chronic paronychia. This condition is often associated with chronic immersion in water, contact with soaps or detergents, use of certain systemic drugs (antiretrovirals, retinoids) and, as is most likely in a 6-year-old child, finger sucking. Findings on examination are similar to those of acute paronychia, with tenderness, erythema, swelling, and retraction of the proximal nail fold. Often the adjacent cuticle is absent. Chronic paronychia has usually been persistent for at least 6 weeks by the time of diagnosis. In addition to medication, basic treatment principles for the condition include avoidance of contact irritants, avoiding immersion of the hands in water, and use of an emollient. Topical corticosteroids have higher efficacy for treating chronic paronychia compared to oral antifungals (SOR B), particularly given the young age of the patient. A topical antifungal can also be tried in conjunction with the corticosteroid.

A 72-year-old African-American male with New York Heart Association Class III heart failure sees you for follow-up. He has shortness of breath with minimal exertion. The patient is adherent to his medication regimen. His current medications include lisinopril (Prinivil, Zestril), 40 mg twice daily; carvedilol (Coreg), 25 mg twice daily; and furosemide (Lasix), 80 mg daily. His blood pressure is 100/60 mm Hg, and his pulse rate is 68 beats/min and regular. Findings include a few scattered bibasilar rales on examination of the lungs, an S3 gallop on examination of the heart, and no edema on examination of the legs. An EKG reveals a left bundle branch block, and echocardiography reveals an ejection fraction of 25%, but no other abnormalities. Which one of the following would be most appropriate at this time? (check one) A. Increase the lisinopril dosage to 80 mg twice daily B. Increase the carvedilol dosage to 50 mg twice daily C. Increase the furosemide dosage to 160 mg daily D. Refer for coronary angiography E. Refer for cardiac resynchronization therapy

This patient is already receiving maximal medical therapy. The 2002 joint guidelines of the American College of Cardiology, the American Heart Association (AHA), and the North American Society of Pacing and Electrophysiology endorse the use of cardiac resynchronization therapy (CRT) in patients with medically refractory, symptomatic, New York Heart Association (NYHA) class III or IV disease with a QRS interval of at least 130 msec, a left ventricular end-diastolic diameter of at least 55 mm, and a left ventricular ejection fraction (LVEF) ≤30%. Using a pacemaker-like device, CRT aims to get both ventricles contracting simultaneously, overcoming the delayed contraction of the left ventricle caused by the left bundle-branch block. These guidelines were refined by an April 2005 AHA Science Advisory, which stated that optimal candidates for CRT have a dilated cardiomyopathy on an ischemic or nonischemic basis, an LVEF ≤0.35, a QRS complex ≥120 msec, and sinus rhythm, and are NYHA functional class III or IV despite maximal medical therapy for heart failure.

A 23-year-old gravida 1 para 0 at 36 weeks gestation presents to the office complaining of ankle swelling and headache for the past 2 days. She denies any abdominal pain or visual disturbances. On examination you note a fundal height of 35 cm, a fetal heart rate of 140 beats/min, 2+ lower extremity edema, and a blood pressure of 144/92 mm Hg. A urine dipstick shows 1+ proteinuria. A cervical examination reveals 2 cm dilation, 90% effacement, -1 station, and vertex presentation. Which one of the following is the most appropriate next step in the management of this patient? (check one) A. Laboratory evaluation, fetal testing, and 24-hour urine for total protein B. Ultrasonography to check for fetal intrauterine growth restriction C. Initiation of antihypertensive treatment D. Immediate induction of labor E. Immediate cesarean delivery

This patient most likely has preeclampsia, which is defined as an elevated blood pressure and proteinuria after 20 weeks gestation. The patient needs further evaluation, including a 24-hour urine for quantitative measurement of protein, blood pressure monitoring, and laboratory evaluation that includes hemoglobin, hematocrit, a platelet count, and serum levels of transaminase, creatinine, albumin, LDH, and uric acid. A peripheral smear and coagulation profiles also may be obtained. Antepartum fetal testing, such as a nonstress test to assess fetal well-being, would also be appropriate. Ultrasonography should be done to assess for fetal intrauterine growth restriction, but only after an initial laboratory and fetal evaluation. Delivery is the definitive treatment for preeclampsia. The timing of delivery is determined by the gestational age of the fetus and the severity of preeclampsia in the mother. Vaginal delivery is preferred over cesarean delivery, if possible, in patients with preeclampsia. It is not necessary to start this patient on antihypertensive therapy at this point. An obstetric consultation should be considered for patients with preeclampsia.

An 8-year-old male is brought to your office for evaluation of recurrent headaches. His mother explains that the headaches occur at least twice a week and often require him to miss school. The patient says he sometimes feels nauseated and that being in a dark room helps. His mother states that she had migraines as a child. The child's only other medical issue is constipation. A head CT ordered by another physician was negative. Which one of the following would be best for preventing these episodes? (check one) A. Sumatriptan (Imitrex) B. Ibuprofen C. Carbamazepine (Tegretol) D. Propranolol (Inderal) E. Amitriptyline

This patient most likely is suffering from recurrent migraine headaches; at the described frequency and intensity, he meets the criteria for prophylactic medication. Ibuprofen or acetaminophen could still be used as rescue medications, but a daily agent is indicated and propranolol is the best choice for this patient (SOR B). Sumatriptan is not approved for children under the age of 12 years. Carbamazepine has significant side effects and requires monitoring. Amitriptyline is a commonly used agent, but it could worsen his constipation.

A 7-year-old female with a history of asthma is brought to your office for a routine follow-up visit. She has a history of exercise-induced asthma, but also has had exacerbations in the past that were unrelated to exercise. In the past month, she has premedicated herself with albuterol (Proventil, Ventolin) with a spacer before recess 5 days/week as usual. She has also needed her albuterol to treat symptoms (wheezing and/or shortness of breath) once or twice per week and had one exacerbation requiring medical treatment in the past year. She has had no nighttime symptoms. Albuterol as needed is her only medication. After reinforcing asthma education, which one of the following would be most appropriate? (check one) A. Referral to an asthma specialist B. Addition of a low-dose inhaled corticosteroid C. Addition of a long-acting β-agonist D. Elimination of premedication with albuterol, restricting use to an as-needed basis E. No changes to her regimen

This patient's asthma is well-controlled according to the 2007 NHLBI asthma guidelines. The "rule of twos" is useful in assessing asthma control: in children under the age of 12, asthma is NOT well-controlled if they have had symptoms or used a β-agonist for symptom relief more than twice per week, had two or more nocturnal awakenings due to asthma symptoms in the past month, or had two or more exacerbations requiring systemic corticosteroids in the past year. For individuals over 12 years of age, there must be more than two nocturnal awakenings per month to classify their asthma as not well controlled. Exercise-induced asthma is considered separately. A β-agonist used as premedication before exercise is not a factor when assessing asthma control. Since this patient does not exceed the rule of twos, her asthma is categorized as well-controlled and no changes to her therapy are indicated. Asthma education should be reinforced at every visit.

Systolic vs. Diastolic HF

This patient's history and clinical examination suggest heart failure. The most important distinction to make is whether it is diastolic or systolic, as the drug treatment may be somewhat different. Physical findings and chest radiographs do not distinguish systolic from diastolic heart failure. An echocardiogram is the study of choice, as it will assess left ventricular function. In diastolic dysfunction, the left ventricular ejection fraction is normal or slightly elevated. Diastolic failure is more common in elderly females and patients with hypertension, and less common in patients with a previous history of coronary artery disease. Diuretics and angiotensin receptor blockers (ARBs) are useful treatments. Because of their effects on diastolic filling times, tachycardia and atrial fibrillation often cause decompensation in patients with diastolic heart failure. At this time, cardiac catheterization is not indicated, and a stress test will not provide useful information. If the patient had systolic failure, a workup for ischemic disease would be needed, but most cases of diastolic dysfunction are not caused by ischemia. While hyperthyroidism can cause tachycardia and atrial fibrillation, the more immediate issue in this patient is the heart failure, which requires diagnosis and treatment. A pulmonary embolus can cause shortness of breath but usually has an acute onset, so a D-dimer level would not help at this time.

A 21-year-old primigravida at 28 weeks gestation complains of the recent onset of itching. On examination she has no obvious rash. The pruritus started on her palms and soles and spread to the rest of her body. Laboratory evaluation reveals elevated serum bile acids and mildly elevated bilirubin and liver enzymes. The most effective treatment for this condition is: (check one) A. triamcinolone (Kenalog) cream B. cholestyramine (Questran) C. diphenhydramine (Benadryl) D. doxylamine succinate E. ursodiol (Actigall)

This patient's symptoms and laboratory values are most consistent with intrahepatic cholestasis of pregnancy. Ursodiol has been shown to be highly effective in controlling the pruritus and decreased liver function (SOR A) and is safe for mother and fetus. Topical antipruritics and oral antihistamines are not very effective. Cholestyramine may be effective in mild or moderate intrahepatic cholestasis, but is less effective and safe than ursodiol.

What is a Chylothorax?

Thoracic duct disruption causing an cholesterol complex accumulation which is associated with lymphoma and thoracic surgery.

Contraindications to thrombolytic therapy in acute stroke include which one of the following? (check one) A. Age >80 B. Resolving transient ischemic attack C. Blood glucose >200 mg/dL D. Deficit present for >1 hour

Thrombolysis is now an approved treatment for acute stroke. The critical time frame is 3 hours after the onset of the deficit. Beyond that time span, the use of thrombolytic agents is contraindicated. Advanced age per se is not a contraindication to thrombolytic therapy. Contraindications include blood glucose levels <50 mg/dL or >400 mg/dL, resolving transient ischemic attack, and hemorrhage visible on a CT scan

How do you treat a stroke?

Thrombolytic therapy (tPa given within 3 hours of symptoms), antiplatelets for ischemia strokes and TIA

What are common causes for Venous thromboembolism

Thrombophilias- Factor V Leiden (Activated Protein C resistance), surgery, trauma, age over 45, pregnancy, OCP or HRT, medical conditons

A child picks up his toy and has a thumb-finger grasp; he looks at a source of noise and tries to imitate sounds. He rolls over from stomach to his back and bangs 2 cubes, but he does not have a grasp reflex. He also responds to simple commands and responds to his name. He understands "no" and plays interactive games; he feeds himself using 2 or 3 fingers, and he waves goodbye. You know that he is probably older than 8 months of age based on what action??

Thumb finger grasp

Which one of the following is true concerning postpartum depression? (check one) A. It has no effect on cognitive development of the child B. It is directly related to the desired gender of the infant C. It is usually transient, lasting about 10 days D. Thyroid function should always be assessed in women with postpartum depression

Thyroid function must be evaluated in women with postpartum depression since both hyperthyroidism and hypothyroidism are more common post partum. Postpartum depression may impair cognitive and behavioral development in the child. It is not related to the desired gender of the child, breastfeeding, or education level of the mother. It should be differentiated from the short-term "baby blues" that resolve within about 10 days. Sertraline is considered first-line treatment for postpartum depression in women who are breastfeeding.

How do you treat Diabetic peripheral neuropathy?

Tight control of hyperglycemia

Why do we use antibiotics for bordetella pertussis?

To decrease infectivity; no effects on severity or duration

Why do you do a steroid taper?

To monitor the symptoms to see if they reoccur.

A 12-month-old Caucasian boy presents for a well-baby visit. Prenatal and past medical histories are unremarkable. Growth and development are normal. Immunizations are up to date. There are no acute complaints. Examination is normal. As part of your routine anticipatory guidance, you discuss poisoning prevention. What instruction should you give for poisoning prevention at home? Toxic liquid cleansers should be placed at the far rear of the cabinet under the bathroom sink Ipecac should be kept at home to induce vomiting in the case of ingestion of a toxic substance Child resistant containers insure against ingestion of toxic substances Toxic substances should be kept out of sight and out of reach Loose prescription medication should be stored in relabeled containers

Toxic substances should be kept out of sight and out of reach Accidental poisonings occurred an estimated 1.2 million times among children younger than 6 years of age in the United States in 2001. Fortunately, fatalities were rare. Child resistant packaging and safer medications have reduced the number of fatalities, as has rapid and more specific therapy. Toxic substances should be kept out of sight and out of reach; if possible, they should be locked away.

How do tension pneumothorax present?

Tracheal and/or mediastinal shift towards contralateral hemithorax (pushes everything to the opposite side)

A full-term male infant was delivered by emergency cesarean section due to obstructed labor. Birth weight of the baby was 3.5 kg. He developed respiratory distress at 2 hours of age. Physical examination reveals that the infant is dyspneic, and respiratory rate is 68/min with subcostal and intercostal retractions. There is no cyanosis. Septic screening is negative. He is managed with IV fluids, and oxygen saturation is maintained with 40% oxygen. Respiratory distress settles after 24 hours. The 1st image shows 4 hours of age, and the 2nd shows 24 hours of age. What is the most likely clinical diagnosis? Meconium aspiration syndrome Hyaline membrane disease Congenital pneumonia Transient tachypnea of the new born (TTNB) Persistent fetal circulation

Transient tachypnea of the new born (TTNB) The most likely diagnosis is transient tachypnea of the newborn (TTNB), as evidenced by the fact that the respiratory distress could be successfully managed by 40% oxygen and IV fluids as well as it disappearing within 24 hours. The diagnosis is further supported by the chest X-ray (4 hours) findings of pleural effusion on the left side in the 1st film done at 4 hours of age and complete clearance of the effusion after 24 hours, which is seen in the 2nd X-ray. TTNB is a relatively mild, self-limited disorder manifesting as respiratory distress. It commonly occurs in term and near term babies. It occurs due to slow absorption of fetal lung fluid, and it manifests with tachypnea, chest retractions, and respiratory grunting, which may respond to 40% oxygen, and respiratory distress should disappear within 2 - 3 days.

Treatment of cushing's disease/syndrome (hypercorticolism)

Transsphenoidal resection of the pituitary adenoma and hydrocortisol replacement

An 8-year-old male presents to your office 2 days after returning from a trip to Mexico with his family. He developed watery, nonbloody diarrhea on the day of departure. He has mild abdominal cramping, but no fever or vomiting. His mother had similar symptoms, which were milder and resolved with over-the-counter treatments. Which one of the following would be most appropriate to treat this patient's condition? (check one) A. Metronidazole (Flagyl) B. Ciprofloxacin (Cipro) C. Azithromycin (Zithromax) D. Mebendazole E. Metoclopramide (Reglan)

Traveler's diarrhea commonly occurs in travelers to Mexico and developing countries. It is usually caused by bacterial organisms such as Escherichia coli, Campylobacter, Shigella, and Salmonella. Viral and parasitic organisms are less common causes, unless the diarrhea persists for 2 weeks. Appropriate medications include antidiarrheal agents such as loperamide, bismuth subsalicylate, and antibiotics. Fluoroquinolones are effective in adults, but should not be used in an 8-year-old. Azithromycin isgenerally effective and safe in children. Metronidazole, mebendazole, and metoclopramide would not be likely to successfully treat bacterial traveler's diarrhera.

A 75-year-old male consults you after his family expresses concern about his loss of interest in his usual activities. They believe he has become increasingly withdrawn since the death of his wife 8 months earlier. You note he has lost 8 kg (18 lb) since his last office visit 6 months earlier. He does not drink alcohol. His physical examination is unremarkable for his age except for a blood pressure of 105/70 mm Hg. Detailed laboratory studies, including thyroid function tests, are all within normal limits. He tells you he would be fine if he could just get some sleep. His Mini-Mental State Examination is normal, but he is obviously clinically depressed. The most appropriate medication for his depression would be: (check one) A. trazodone (Oleptro) B. mirtazapine (Remeron) C. bupropion (Wellbutrin) D. amitriptyline E. nortriptyline (Pamelor)

Trazodone may be useful for insomnia, but is not recommended as a primary antidepressant because it causes sedation and orthostatic hypotension at therapeutic doses. Bupropion would aggravate this patient's insomnia. Tricyclic antidepressants may be effective, but are no longer considered first-line treatments because of side effects and because they can be cardiotoxic. Mirtazapine has serotonergic and noradrenergic properties and is associated with increased appetite and weight gain. It may be particularly useful for patients with insomnia and weight loss.

Where does pes anserine bursitis hurt?

medial proximal tibia

Which opioid side-effects don't change over time?

miosis and constipation

Treatment of hyperphosphatemia secondary to CKD

Treat with dietary phosphorus restriction and oral phosphate

Treatment of seborrheic dermatits

Ultraviolet radiation. Cradle cap is treated with olive oil compresses and baby shampoo or ketoconazole shampoo.

What are the classifications of an exudative effusion?

Uncomplicated effusion are free flowing sterile exudates that resolve with ABX Complicated effusion often froms a pleural "peel" and traps healthy lung tissue. Tx: thoracostomy or surgery Empyema Tx: thoracotomy and decortication

You have been treating a 43-year-old male for unipolar depression for 4 years. He has developed treatment-resistant depression, and despite having a good initial response to an SSRI, his symptoms are worsening. He has failed to improve despite escalated doses of multiple SSRIs and SNRIs. He is currently taking citalopram (Celexa), 60 mg daily. Of the following, the most effective adjunctive therapy would be augmentation with: (check one) A. lithium bicarbonate B. high-dose triiodothyronine C. an atypical antipsychotic, such as olanzapine (Zyprexa) D. an anticonvulsant, such as gabapentin (Neurontin)

Up to one-third of patients with unipolar depression will fail to respond to treatment with a single antidepressant, despite adequate dosing and an appropriate treatment interval. Lithium, triiodothyronine (T3 ), and atypical antipsychotics can all provide clinical improvement when used in conjunction with the ineffective antidepressant. The American Psychiatric Association and the Institute for Clinical Systems Improvement both recommend a trial of lithium or low-dose T 3 for patients who have an incomplete response to antidepressant therapy. A meta-analysis showed that a serum lithium level ≥0.5 mEq/L and a treatment duration of 2 weeks or greater resulted in a good response (SOR A). While thyroid supplementation as adjunctive therapy is effective, the recommended dosage is no higher than 50 μg/day (SOR B). Atypical antipsychotics can be used as add-on therapy, but are not as effective as lithium or T3 (SOR B). Anticonvulsant medications such as gabapentin have been shown to be effective in the management of bipolar affective disorder, but not as adjunctive therapy in the treatment of unipolar depression resistant to single-agent antidepressants.

A 21-year-old female complains of bulging veins in her right shoulder region, along with swelling and a "tingling" sensation in her right arm that has developed over the past 2 days. There were no unusual events other than her regular workouts with her swim team. Ultrasonography confirms an upper extremity deep-vein thrombosis of her right axillary vein. Which one of the following would be the most appropriate treatment? (check one) A. Intravenous heparin for 72 hours, followed by oral warfarin (Coumadin) for 3 months B. Low molecular weight heparin (LMWH) subcutaneously for 5 days only C. LMWH subcutaneously for at least 5 days, followed by oral warfarin for 3 months D. LMWH subcutaneously for at least 5 days, followed by oral warfarin indefinitely E. Oral warfarin for 3 months

Upper extremity deep-vein thrombosis (UE-DVT) accounts for 4% of all cases of DVT. Catheter-related thromboses make up the majority of these cases. Occult cancer, use of oral contraceptives, and inheritable thrombophilia are other common explanations. Another proposed risk factor is the repetitive compression of the axillary-subclavian vein in athletes or laborers, which is the most likely cause of this patient's UE-DVT. Taken as a whole, UE-DVT is generally associated with fewer venous complications, including less chance for thromboembolism, postphlebitic syndrome, and recurrence compared to lower-extremity deep-vein thrombosis (LE-DVT). However, the rates of these complications are still high enough that most experts recommend treatment identical to that of LE-DVT. Specifically, heparin should be given for 5 days, and an oral vitamin-K antagonist for at least 3 months.

You see a 78-year-old male in the hospital the day after his hip-replacement surgery. He has not voided in the past 12 hours. A urethral catheter is placed and 500 mL of urine is removed from his bladder. Which one of the following is most likely to improve the success rate of a voiding trial? (check one) A. Using a specialized catheter coudé instead of a standard catheter B. Leaving the catheter in place for at least 2 weeks C. Immediately removing the catheter to prevent a urinary tract infection D. Starting tamsulosin (Flomax), 0.4 mg daily, at the time of catheter insertion E. Starting antibiotic prophylaxis at the time of catheter insertion

Urinary retention is a common problem in hospitalized patients, especially following certain types of surgery. Starting an a-blocker at the time of insertion of the urethral catheter has been shown to increase the success of a voiding trial (SOR A). Voiding trial success rates have not been shown to be improved by leaving the catheter in for 2 weeks, immediate removal of the catheter, using a specialized catheter, or antibiotic prophylaxis.

Treatment of dandruff

Use shampoos containing selenium or sinc and ketoconazole

A 52-year-old male requests "everything you've got" to help him stop smoking. You review common barriers to quitting and the benefits of cessation with him, and develop a plan that includes follow-up. He chooses to start varenicline (Chantix) to assist with his efforts, and asks about also using nicotine replacement. Which one of the following would be accurate advice? (check one) A. Combining these medications has not proven to be beneficial B. The addition of transdermal nicotine, but not nicotine gum, has proven benefits C. The combination is highly efficacious D. Nicotine replacement doses need to be doubled in a patient taking varenicline E. The combination of nicotine and varenicline is potentially lethal

Varenicline works by binding to nicotine receptors in the brain, providing much lower stimulation than nicotine itself would. This has the effect of reducing the reinforcement and reward that smoking provides to the brain. However, this medication also blocks the benefit a patient would receive from nicotine replacement products. Studies have shown that using nicotine replacement products concurrently with varenicline leads to an increase in nausea, headaches, dizziness, and fatigue.

A 5-year-old boy presents with sudden onset of a stumbling gait following a 3-day history of a spreading rash. His mother notes that the rash began as red "bumps" but has progressively changed to fluid-filled blisters. The boy says the rash is itchy. A few days prior to onset he had a low-grade fever, abdominal pain, lethargy and a headache. Aside from his unsteady gait, his anxious mother has also noted that his speech seems slightly slurred. He has had no known exposure to sick children. The boy's older brother is present during the exam and is flapping his hands spastically, chewing on a rubber "P"-shaped toy, and jumping up and down facing the wall. The mother seems to keep checking on his whereabouts and what he's doing. It seems as the boy may be autistic. He has not exhibited any similar symptoms or been ill recently. On physical exam, the patient appears to have a clumsy, wide based gait as well as difficulty articulating certain words. He also has numerous vesicles with a cloudy fluid over his entire body. Intermixed with these are a few lesions that are already scabbed over, as well as numerous erythematous papular lesions. His temperature is 99.4°F. What is the most likely etiology of these findings? Parvovirus B19 Rubella Rubeola Varicella Coxsackie virus

Varicella Explanation The most likely etiology is varicella. Varicella, or chicken pox, presents initially as erythematous macules that develop quickly into papules and thin-walled vesicles surrounded by an erythematous base. Patients will have successive crops of lesions, which scab over in 5 to 7 days with occasional residual scarring. Mucosal involvement of areas such as the oropharynx and vagina is common. Marked pruritus may be seen. The incubation period is 14 to 21 days, and the prodrome consists of malaise, fever, and headache. The causative agent is the varicella-zoster virus.

What is in virchows triad?

Venous stasis Hypercoagubility Endothelial injury

Harsh, pansystolic murmur that is heard best at the left sternal border in the 3rd and 4th intercostal spaces.

Ventricular septal defects

What is the preferred method of diagnosing psychogenic non-epileptic seizures?

Video EEG

Nausea and Vomiting of Pregnancy Tx

Vitamin B6 Doxylamine

A 4-month-old female infant has been gaining weight well and achieving normal milestones. She was a full-term infant born via vaginal delivery without complication. Her past medical history is significant only for a mild URI the previous month, and she is current with her vaccines. She has been exclusively breastfed since birth. Her mother eats a well-rounded diet that includes meat and dairy. What supplementation does the infant require?

Vitamin D

What should patients taking phenytoin be placed on prophylactically?

Vitamin D

First action for hypercalcemia of malignancy

Volume correction witih normal saline

Treatment of viral meningitis?

no specific therapy unless herpes simplex virus is suspected ad then treat with acyclovir

A 3-year-old boy presents for a well-child visit. He is recovering from laryngotracheitis which was managed with over-the-counter cold medications. He has started daycare and seems to be adapting well. He enjoys a varied diet and his parents have no specific concerns. Past medical history and family history are unremarkable. Physical examination is normal. What developmental milestone is appropriate for his age? Tells a story Throws a ball overhand Washes his hands Engages in interactive play with other children Goes to the toilet alone

Washes his hands A child of 3 years of age would be expected to speak in short sentences, but he would not be able to tell a story. Although gross motor skills are developing, the strength and coordination to throw a ball overhand would not usually be expected until age 4. Children at this age generally engage in parallel play more so than group play. Some children at this age will be potty-trained, but rarely will they be able to go to the bathroom alone.

A 6-month-old girl presents for a well-baby visit. Prenatal and past medical histories are unremarkable. Growth and development are normal. Immunizations are up to date. There are no acute complaints. Examination is normal. As part of your routine anticipatory guidance, you discuss bath time safety. What instruction should you give regarding bath time safety? The girl's 7-year-old sister should attend to the girl in the bath if her mother is otherwise occupied Water heater maximum temperature should be set no higher than approximately 120°F to prevent scalding If the child must be left unattended in the bath, an inflatable tube or bath seat should be provided Water heater maximum temperature should be set no higher than approximately 140° to prevent scalding The tub may be refilled by an older sibling provided that the water heater maximum temperature is set no higher than approximately 120°F

Water heater maximum temperature should be set no higher than approximately 120°F to prevent scalding Scald injuries and drowning are 2 of the major risks at bath time for the young infant. The higher the water temperature, the shorter the duration of skin contact required to cause a serious burn. Very high temperatures cause burns on contact (scalding). Water heater maximum temperature should be set at approximately 120°F to prevent scalding. Because the water heater thermostat setting may not accurately regulate the temperature of the outflowing water, the supervising adult should test water temperature as well before placing the child in the bath.

Treatment of dyshidrosis

Wet Burrow solution. Fissures treated with collodion. PUVA for generalized disease

A 65-year-old male who has been in good health presents to your office with a 2-day history of a sensation of pressure and hearing loss in his left ear. A physical examination and a thorough neurologic examination are both unremarkable. Both tympanic membranes are normal. An audiogram shows a 30-decibel hearing loss at three consecutive frequencies in the left ear, with normal hearing on the right. Placing a vibrating tuning fork in the midline of the forehead reveals sound lateralizing to the right ear. Which one of the following would be most appropriate at this point? (check one) A. CT B. A CBC, metabolic profile, and thyroid studies C. Nifedipine (Procardia) D. Acyclovir (Zovirax) E. Oral corticosteroids

When a patient presents with sudden hearing loss it is important to distinguish between sensorineural and conductive hearing loss. Patients should be asked about previous episodes, and the workup should include both an assessment for bilateral hearing loss and a neurologic examination. Sudden sensorineural hearing loss is diagnosed by audiometry demonstrating a 30-decibel hearing loss at three consecutive frequencies, with no other cause indicated from the physical examination. Evaluation for retrocochlear pathology may include auditory brainstem response, MRI, or follow-up audiometry. Routinely prescribing antiviral agents, thrombolytics, vasodilators, vasoactive substances, or antioxidants is not recommended. Oral corticosteroids may be offered as initial therapy, and hyperbaric oxygen therapy may be helpful within 3 months of diagnosis. The guidelines also strongly recommend against routine laboratory tests or CT of the head as part of the initial evaluation.

A 4-year-old is brought to the emergency department with abdominal pain and is noted to have 3+ proteinuria on a dipstick. Three days later the pain has resolved spontaneously, and a repeat urinalysis in your office shows 2+ proteinuria with normal findings on microscopic examination. A metabolic panel, including creatinine and total protein, is also normal. Which one of the following would be most appropriate at this point? (check one) A. Renal ultrasonography B. A spot first morning urine protein/creatinine ratio C. An antinuclear antibody and complement panel D. Referral to a nephrologist

When proteinuria is noted on a dipstick and the history, examination, full urinalysis, and serum studies suggest no obvious underlying problem or renal insufficiency, a urine protein/creatinine ratio is recommended. This test correlates well with 24-hour urine protein, which is particularly difficult to collect in a younger patient. Renal ultrasonography is appropriate once renal insufficiency or nephritis is established. If pathogenic proteinuria is confirmed, an antinuclear antibody and/or complement panel may be indicated. A nephrology referral is not necessary until the presence of kidney disease or proteinuria from a cause other than benign postural proteinuria is confirmed.

A positive flexion abduction external rotation (FABER) test that elicits posterior pain indicates involvement of which joint? (check one) A. Sacroiliac B. Shoulder C. Ankle D. Wrist E. Knee

When the flexion abduction external rotation (FABER) test elicits pain posteriorly, it indicates sacroiliac involvement. Anterior pain indicates hip involvement.

A 75-year-old male has not seen a physician in 25 years and presents with advanced Parkinson's disease. The best initial treatment would be: (check one) A. Referral to a neurosurgeon for thalamotomy B. Amantadine (Symmetrel) C. Benztropine (Cogentin) D. Pramipexole (Mirapex) E. Carbidopa/levodopa (Sinemet)

While anticholinergics such as benztropine and amantadine may provide some improvement of symptoms, these effects wane within a few months. Such medications are not a good option in this patient with advanced disease. Dopamine agonists provide some improvement in motor complications, but are mainly used to delay the introduction of levodopa in younger patients, to avoid levodopa-related adverse reactions. Carbidopa/levodopa is better for initial therapy in older patients, and those who present with more severe symptoms. Slow-release versions of this combination may decrease motor fluctuations. Stereotactic thalamotomy is used to ameliorate tremors that have become disabling. This procedure has been replaced by other surgical options such as pallidotomy and high-frequency, deep-brain stimulation of specific nuclei

Leukoplakia

White spot on mucus membranes that stems from chemical irritation and can progress to SCC

An asymptomatic 55-year-old male visits a health fair, where he has a panel of blood tests done. He brings the results to you because he is concerned about the TSH level of 12.0 µU/mL (N 0.45-4.5). His free T4 level is normal. Which one of the following is most likely to be associated with this finding? (check one) A. Atrial fibrillation B. Reduced bone density C. Systolic heart failure D. Elevated LDL cholesterol E. Type 2 diabetes mellitus

With subclinical thyroid dysfunction, TSH is either below or above the normal range, free T3 or T4 levels are normal, and the patient has no symptoms of thyroid disease. Subclinical hypothyroidism (TSH >10 µU/mL) is likely to progress to overt hypothyroidism, and is associated with increased LDL cholesterol. Subclinical hyperthyroidism (TSH <0.1 µU/mL) is associated with the development of atrial fibrillation, decreased bone density, and cardiac dysfunction. Neither type of subclinical thyroid dysfunction is associated with diabetes mellitus. There is insufficient evidence of benefit to warrant early treatment of either condition. Ref: Wilson GR, Curry RW Jr: Subclinical thyroid disease

Men or women: who has slower gastric transit times?

Women

A 19-year-old white female presents for an initial family planning evaluation. Specifically, she is interested in oral contraception. She is not presently sexually active, but has a steady boyfriend. She has no contraindications to oral contraceptive use. She has mild acne vulgaris. You discuss possible side effects and benefits of combined oral contraceptives, including improvement of her acne. Which one of the following is also associated with oral contraceptive use? (check one) A. Increased risk of ovarian cancer B. Decreased risk of ovarian cysts C. Increased risk for ectopic pregnancy D. Increased incidence of dysmenorrhea

Women who take combination oral contraceptives have a reduced risk of both ovarian and endometrial cancer. This benefit is detectable within a year of use and appears to persist for years after discontinuation. Other benefits include a reduction in dysfunctional uterine bleeding and dysmenorrhea; a lower incidence of ovarian cysts, ectopic pregnancy and benign breast disease; and an increase in hemoglobin concentration. Many women also benefit from the convenience of menstrual regularity. All combination oral contraceptives raise sex hormone-binding globulin and decrease free testosterone concentrations, which can lead to improvement in acne

What central acting weight loss drug currently approved by the FDA ?

Xenical (Orlistat) at 120 mg 3 times/day as a prescription drug and 60 mg tid (3 times a day) over the counter (marketed as Alli).

A 72-year-old white male in otherwise good health complains of generalized pruritus that worsens in the winter. The itching is most intense after he bathes. He recently noticed a rash on his abdomen and legs as well. On examination you note poorly defined red, scaly plaques with fine fissures on the abdomen. No eruption is present at other pruritic sites. Which one of the following is the most likely cause of this problem? (check one) A. Stasis dermatitis B. Lichen simplex chronicus C. Xerosis D. Rosacea E. Candidiasis

Xerosis is a pathologic dryness of the skin that is especially prominent in the elderly. It is probably caused by minor abnormalities in maturation of the epidermis that lead to decreased hydration of the superficial portion of the stratum corneum. Xerosis often intensifies in winter, because of the lower humidity and cold temperatures. Stasis dermatitis, due to chronic venous insufficiency, appears as a reddish-brown discoloration of the lower leg. Lichen simplex chronicus, the end result of habitual scratching or rubbing, usually presents as isolated hyperpigmented, edematous lesions, which become scaly and thickened in the center. Rosacea is most often seen on the face as an erythematous, acneiform eruption, which flushes easily and is surrounded by telangiectasia. Candidiasis is an opportunistic infection favoring areas that are warm, moist, and macerated, such as the perianal and inguinal folds, inframammary folds, axillae, interdigital areas, and corners of the mouth

Is edema normal in pregnancy

Yes, not part of criteria for pre-eclampsia

Which three drugs have been proven to reduce hip fractures in the context of osteoporosis?

Zoledronic acid, Risedronate, and Alendronate

You see a 23-year-old gravida 1 para 0 for her prenatal checkup at 38 weeks gestation. She complains of severe headaches and epigastric pain. She has had an uneventful pregnancy to date and had a normal prenatal examination 2 weeks ago. Her blood pressure is 140/100 mm Hg. A urinalysis shows 2+ protein; she has gained 5 lb in the last week, and has 2+ pitting edema of her legs. The most appropriate management at this point would be: (check one) A. Strict bed rest at home and reexamination within 48 hours B. Admitting the patient to the hospital for bed rest and frequent monitoring of blood pressure, weight, and proteinuria C. Admitting the patient to the hospital for bed rest and monitoring, and beginning hydralazine (Apresoline) to maintain blood pressure below 140/90 mm Hg D. Admitting the patient to the hospital, treating with parenteral magnesium sulfate, and planning prompt delivery either vaginally or by cesarean section

`This patient manifests a rapid onset of preeclampsia at term. The symptoms of epigastric pain and headache categorize her preeclampsia as severe. These symptoms indicate that the process is well advanced and that convulsions are imminent. Treatment should focus on rapid control of symptoms and delivery of the infant.

Which one of the following sets of additional findings would indicate that the patient suffers from severe anorexia nervosa? (check one) A. Hypertension, tachycardia, and hyperthermia B. Hypertension, tachycardia, and hypothermia C. Hypotension, tachycardia, and hypothermia D. Hypotension, bradycardia, and hyperthermia E. Hypotension, bradycardia, and hypothermia

all low

Treatment of bacterial meningitis in a neonate?

ampicillin and cefotaxime

Treatment of bacterial meningitis in an infant 3 up to 3 months of age?

ampicillin and cefotaxime (higher doses)

What is the initial anti-coagulating therapy for patients with a drug eluting stent?

aspirin 162-325 plus clopidogrel for 12 months. Then aspirin 75-165 indefinitely

Prophylactic medications for migraines include?

beta blockers, tricyclic antidepressants, calcium channel blockers, NSAIDs, valproic acid, or topiramate.

The FDA has approved what medication as the 1st drug for seasonal affective disorder (SAD).

bupropion (Wellbutrin)

Risk of what with dermatomyositis

cancer

Blowing, systolic murmurs that radiate to the left axilla and left back.

coarctation of the aorta

Schizophrenia Dx

delusions, hallucinations, disorganized speech, grossly disorganized or catatonic behavior, and negative symptoms (i.e., affective flattening, alogia, or avolition).

How do Amphetamines work?

elease of stores of dopamine and norepinephrine from the nerve endings

Clinical presentation of cocaine intoxication

hypertension, tachycardia, diaphoresis, anxiety, dilated pupils, agitation, nasal congestion, and psychosis

First line action for patient with chronic hoarseness and no other symptoms

laryngoscopy

Treatment for Giant Cell Arteritis is prompt to avoid

permanent blindness using prednisone, 60mg given daily, immediately and a temporal artery biopsy

Spinal stenosis vs. herniated disc: which is relieved with sitting and aggravated with standing?

stenosis. Herniated disc is aggravated by sitting and relieved by standing.

Treatment of gastroenteritis

symptomatic. Treatment with bactrim, ampicillin, or ciprofloxacin for severely ill patient

A 3-month-old female infant presents with a maculopapular rash on her trunk. You diagnose exanthema subitum, caused by human herpesvirus 6. Which of the following phrases correctly describes this disease?

there is a rapid onset of high fever

How long must a patient with diphtheria be isolated

until three pharyngeal cultures are documented negative.

Complication of Psuedotumor cerebri

visual loss

Which one of the following historical features is most suggestive of congestive heart failure in a 6-month-old white male presenting with tachypnea? (check one) A. Diaphoresis with feeding B. Fever C. Nasal congestion D. Noisy respiration or wheezing E. Staccato cough

ymptoms of congestive heart failure in infants are often related to feedings. Only small feedings may be tolerated, and dyspnea may develop with feedings. Profuse perspiration with feedings, is characteristic, and related to adrenergic drive. Older children may have symptoms more similar to adults, but the infant's greatest exertion is related to feeding. Fever and nasal congestion are more suggestive of infectious problems. Noisy respiration or wheezing does not distinguish between congestive heart failure, asthma, and infectious processes. A staccato cough is more suggestive of an infectious process, including pertussis.


Conjuntos de estudio relacionados

Texas Life and Health: Field underwriting procedures

View Set

Information Literacy lesson 3 exam

View Set

3. Základní a ostatní činnosti bank otázky

View Set

CH.1 - LIMITS, ALTERNATIVES, CHOICES

View Set

Middle Ages Quiz 1: Feudalism, The Church, and Medieval Life

View Set

Accounting Chapter 17 Learnsmart/Homework

View Set

DSM-5 Study Guide Questions and Answers

View Set

Unit 4 English Which sentence uses numbers correctly?

View Set